Unz评论•另类媒体选择$
美国主流媒体大都排除了有趣,重要和有争议的观点
 博客浏览詹姆斯·汤普森档案馆
惊呆了:被代数骗了吗?

书签 全部切换变革理论添加到图书馆从图书馆中删除 • B
显示评论下一个新评论下一个新回复了解更多
回复同意/不同意/等等 更多... This Commenter This Thread Hide Thread Display All Comments
同意不同意谢谢LOL轮唱
这些按钮可将您的公开协议,异议,感谢,LOL或巨魔与所选注释一起注册。 仅对最近使用“记住我的信息”复选框保存姓名和电子邮件的频繁评论者可用,并且在任何八个小时的时间内也只能使用三次。
忽略评论者 关注评论者
搜寻文字 区分大小写  确切的词  包括评论
列表 书签

黑天鹅谷歌

纳西姆·尼古拉斯·塔勒布 (Nassim Nicholas Taleb) 在推特上发表了一系列关于情报研究的评论。

https://threadreaderapp.com/thread/1076845397795065856.html

他现在将这些内容汇总成一种格式,并附有链接和说明。

https://medium.com/incerto/iq-is-largely-a-pseudoscientific-swindle-f131c101ba39

对他的文章不缺乏信心。 这里有很多东西要讨论,接下来的内容涵盖了我认为的要点。 我添加了一些指向相关出版物的链接,但您可以在我的搜索栏中输入任何概念和作者姓名以获取更多详细信息。

1 智商很大程度上是一种伪科学的骗局

鉴于塔勒布批评情报研究人员使用的统计数据不佳,温和的评论是最好更精确。 我假设他的意思是一半以上的情报研究结果是错误的,而且是出于恶意的原因。 如果这是他的观点,那么他实际上是错误的。

2 智商是陈旧的,主要衡量非常低的智力,或者是纸牌洗牌者或那些不适合现实生活的人的次等智力。 “它最多可以解释某些任务中 13% 到 50% 的性能”。 它基于糟糕的数学,并受到种族主义者和骗子的推动。

似乎塔勒布对很多人的看法都很差。 “洗牌员”大概包括了所有为明星交易员保管账簿和处理交易的幕后工作人员。 Taleb 解释说,他对 IQ 背后的数学表示怀疑的原因是,他可以根据特定假设通过计算机生成相关性,这些相关性看起来就像一些报告的关于智力和学业成绩的发现。 他暗示,如果他能在简单的基础上做到这一点(创建一个神话测试,只测量低于 IQ 100 的表现,并逐渐增加高于该水平的噪音),那么 Frey 和 Detterman (2004) 报告的实际观察结果无效。 这不是一个令人信服的论据。 一个简单得多的解释是,将一项人口范围的衡量标准(一般智商)与仅由精选的优秀学生(SAT)进行的学术测试进行比较,但仍然很好地展示了两者之间的联系。 这是一个现实生活中的发现,据说是塔勒布喜欢的那种。

3 如果你想检测某人在某项任务上的表现,比如高利贷、打网球或随机矩阵理论,让他/她完成这项任务; 我们不需要概率挑战的心理学家对现实世界的功能进行理论考试。

事实上,心理学家已经明白这一点。 亨特、施密特和昆泽尔指出,检验一个人能否胜任工作的最好办法就是让他们去尝试。 然而,这在时间和金钱上都是昂贵的,因为你必须监督他们预防灾难,给他们详细的指示并仔细监控他们的表现,所有这些至少需要两周时间才能对申请人的能力进行合理的评估。 您不能对所有申请人都这样做,否则会占用所有员工进行实际业务工作所需的时间。 上述研究人员表明,就结果而言,智力测试几乎是次佳的,而且速度更快、成本更低。 添加一个诚实的测试,你有一个有效的选择系统。

4 不同的总体有不同的方差,甚至不同的偏度,这些比较需要更丰富的模型。

同样,大多数心理测量学家都同意这一点,并且几十年来一直为人所知。 至少,他们喜欢看到正确绘制的数据,因此实际发现是可见的,并且可以通过不同的统计方法对其进行分析。 这里没有什么新的或有见地的。

5 一种适用于左尾而不是右尾的度量(IQ 随着它变得更高而去相关)是有问题的。

Lubinski 和 Benbow 在大样本的前瞻性研究中表明,智商在最高水平仍然具有预测性,并且在每个更高的水平上继续发挥作用。 塔勒布的观点显然是错误的。

6 它(智商)可以衡量一些被认为有用的任意选择的心理能力(在测试环境中)。 但是,如果您对智力采取波普尔-哈耶克 (Popperian-Hayekian) 的观点,您会意识到要衡量它,您需要了解未来生态系统所需的心理技能,这需要对所述未来生态系统具有可预测性。 它还需要具备面向未来的技能(因此需要有心理偏见才能生存)。

智力测试项目不是随意的。 选择它们来代表从实际任务和现实生活问题中提取的各种能力。 它们与专门基于美国社会现实生活任务的测试高度相关,例如 Wonderlic Personnel Test。 几十年来,琳达·戈特弗雷德森 (Linda Gottfredson) 多次展示了这一切。 至于“未来生态所需的心理技能”,这是智能行为的一个很好的例子,生存也是如此。 在苏格兰的一项人口研究中,伊恩·迪瑞 (Ian Deary) 表明,在 11 岁时测试的智力可以预测寿命到老年。 聪明的人能够比不那么聪明的人活得更久。 塔勒布又错了。

7 现实生活从来不会提供有清晰答案的清晰问题(大多数问题没有答案;也许智商最糟糕的问题是它似乎选择了不喜欢说“没有答案,不要浪费”的人时间,找点别的”。)

如果这是一个相关的反对意见,那么苏格兰 11+ 要求的清晰答案就不会显示出与寿命和几十年的成就有任何关系。 同样,SMPY 参与者所需的清晰答案不会预测他们的中年成就(并且随着后续行动的继续,可能会预测数十年的成就)。 向后数字是一个清晰的答案任务。 它浪费的时间很少,但却是一般能力的一个很好的预测指标。 清晰的测试答案还与神经影像学评估的许多大脑结构和功能测量相关(海尔,2017 年)。 此外,鉴于所有谜题都需要脑力,这些选定的项目可能会挖掘解决更普遍和紧迫性质的谜题的一般能力。

8 需要某种类型的人将智慧的注意力浪费在课堂/学术问题上。 这些是毫无生气的官僚,他们可以鼓起无用的动力。 有些人只能关注真实的问题,而不是虚构的教科书问题。

塔勒布对他的侮辱非常自由。 它可能会影响那些已经采取反智商立场的人。 可以在两分钟内获得粗略的能力衡量标准,这不会影响集中度。 当然,许多人更喜欢实用而不是学术,并且可能最专注于现实生活中的问题。 这是可测试的,再一次,在广泛的人群和广泛的现实生活问题上,智力测试保持预测效用。 Detterman 显示了许多相关性。

9 智商不检测凸性(通过类似于偏差方差的论点,您需要犯很多无关紧要的小错误,以避免产生重大的后果。参见 Antifragile 以及任何没有凸性的“智能”度量是无意义的edge.org/conversation/n...)

Taleb 就他所说的“凸性”提出了有趣的观点。 在他的 Edge 文章中,他指出“机会”并不是长期收益的好解释。 现在我们有一些我们可以达成一致的事情。 塔勒布所说的“凸性”意味着,在他看来,研究的进展是通过“收益(因为它们需要很大)和错误(小或无害)之间的显着不对称性,并且正是由于这种不对称性,运气和反复试验才能产生结果”。 这是一个凸函数,因此得名。 美好的。 这可能是也可能不是,并且不清楚如何直接测试这个假设(因此不科学),也不清楚为什么这个提议意味着智力测量是“无效的”。 一项测试可能是查看 IQ 与期权交易/金融投资之间的相关性。 后者是具有相当重要意义的现实世界测试,如果结果为零,则会加强他的论点。

幸运的是,这里有一份相关的出版物。

https://sci-hub.shop/10.1016/j.jfineco.2011.05.016

我们分析 IQ 是否影响交易行为、绩效和交易成本。 该分析将股票收益、交易和限价订单数据与对几乎所有应征年龄的芬兰男性进行的智力 (IQ) 测试的二十年分数结合起来。 控制多种因素,我们发现,高智商的投资者受处置效应影响较小,避税交易更积极,更有可能在股票创下一个月高位时提供流动性。 高智商投资者还表现出卓越的市场时机、选股技巧和交易执行能力。

作者发现,通过更好地选择股票和降低交易成本,高智商的受试者每年比低智商的受试者表现好 4.9%。 鉴于实际回报率平均为 7%,这是一个巨大的差异,它会随着时间的推移而累积,并为更聪明的投资者带来高得多的个人净资产。 顺便说一下,智力是在征兵年龄衡量的,早在有很多投资历史之前,所以更有可能是因果关系。

https://www.sciencedirect.com/science/article/pii/S0304405X1100211X

10 看到浅薄的模式并不是一种美德——会导致天真的干预主义。 一些心理学家回信给我:“智商选择模式识别,对现代社会的运作至关重要”。 不。除非模式很重要,否则看不到模式是现实生活中的一种美德。 要在生活中做得好,您需要深度和能力来选择自己的问题并独立思考。

能够看到其他人无法看到的模式的能力传统上被视为智慧的标志。 有趣的是,塔勒布承认有些问题是肤浅的。 他怎么知道? 大概他可以看穿它们,并且发现它们很容易。 好的。 物品难度各不相同。 深度、选择问题和独立思考的能力是智力的标志。 这是与心理测量学一致的一点,尽管他可能没有意识到这一点。

11 功能商数:如果你把 IQ 重命名,从“智能商数”到 FQ“功能商数”或 SQ“Salaryperson Quotient”,那么一些东西将是真实的。 它最能衡量成为一个好奴隶的能力。

塔勒布的论点似乎是智商测试只适用于从事单调工作的普通人。 不完全是义务。 这是智力测试不衡量创造力的古老观点的一个版本。 很容易断言,但证据似乎反对它,只要你用质量和数量来衡量创造力。 Rex Jung 创造性地、仔细地研究了这个问题。

昆泽尔表明,即使在远非单调乏味的工作中,大学入学考试也可以预测成功。

12 “智商”最能预测军事训练中的表现,w/correlation~.5,(这是循环的,因为招聘不是随机的)。

两个批评。 首先,军事训练数据本身很有趣,但就调查结果的普遍性而言,关键问题是相当多的任务与非军事任务相同。 例如,车辆维修是相同的任务,因此我们所拥有的关于智商/培训环节的详细信息远比通常在商用汽车修理厂中收集的信息要详细得多。

其次,远非“圆形”,观察到的 5 的相关性实际上因范围限制而减弱。 Taleb 会犯一个简单的统计错误吗? 不可能的。 由于美国军方被允许筛选低能力的候选人,他们提供了对塔勒布早些时候断言的精确测试,即测试只对低能力的人有效。 5 的相关性是在能力较高的人身上实现的。 如果我们假设至少需要 100 IQ,那么真正的相关性可能是 7

13 我在这里没有备份的心理参考:简单地说,这个领域很萧条。 到目前为止,约有 50% 的研究没有重复,而效果较弱的论文。 不算差转移到现实。

为什么他不提供参考? 可能是因为,事实上,心理测量学的主要发现与心理学的其他领域一样好或更好。 只是很多人讨厌结果。

14 弗林效应应该警告我们不仅智商在某种程度上取决于环境,而且它是循环的。

好吧,弗林效应并没有提高数字跨度或数学分数,所以这里有一个关于测量问题的被忽略的故事。 不清楚FE是ag效应。 很少有心理测量学家怀疑环境会影响能力。 智商不是循环的,但可以由所有文化中都存在的非常简单的心理任务决定,并且对更复杂的任务具有长期预测能力。

总结

塔勒布以极大的信心做出了全面的断言,并充斥着侮辱性的语言。 这些断言很可能会影响那些对智力不确定的人,以及那些认为对自己有把握的人必须知道他们在说什么的人。 这是可以理解的:一个不确定的人意识到他们需要做更多的阅读和思考才能感到自信,并且仁慈地假设只有一个有必要阅读的知识渊博的人才会敢于自信地说话。

然而,塔勒布并没有在他的文章末尾给出科学参考,而是自信地断言他不需要这样做,因为这个领域被打破了,因为…… 凸度。 这似乎是统计分析的基本要素,而不是他关于研究策略的有趣想法之一。 这很有趣,因为即使在 Taleb 称之为他自己的领域,作为一名金融工具交易员,也很容易找到一项仔细、长期、大样本的研究,显示情报对投资行为的有益影响。 在自己的主场,他1-0落后。

另一个失误是忽略了情报研究人员(特别是詹森)几十年来为改进情报措施而进行的辩论,以便它们符合 SS Stevens 提出的要求。 数字跨度就是这样一个衡量标准。 数字符号也是如此,如果广泛测量,词汇也是如此。 简单和复杂的反应时间是其他例子。 总体而言,Taleb 并未提供推动该领域发展的新见解或原创见解。 但他的目标似乎没有建设性,甚至没有提供信息。

我不知道为什么一个有能力的人对能力的衡量如此不屑,但只能假设他很清楚自己的能力,并且认为自己高于这些世俗。 他没有给出参考文献,但提到了他即将出版的一本书。 最好坚持事实。

塔勒布吹嘘他明显不擅长的领域是否意味着我们应该忽视他对其他领域的贡献? 可能不会。 公众人物有时会偏离他们的职权范围。 这是一种由公众奉承带来的职业危害,自罗马时代就已为人所知。 然而,如果他在超出他的深度时能如此夸张,那么以稍微批判性的眼光回到他的其他作品是明智的。 当我读到他关于概率的想法时,我对他关于风险的一些声明做出了积极的假设,这是非常谨慎的理由,我无法反驳他的数学研究。 也许我被代数愚弄了。 也许我不是唯一的。

塔勒布将自己描述为一个流浪者,也就是一辆婴儿车,是那种四处游荡的人。 没有问题。 天鹅很迷人,但美丽的形状可能会让我们误入歧途。

 
• 类别: 科学 •标签: 经典卡, 房源搜索, IQ, 心理测验, 种族和智商 
隐藏444条评论发表评论
忽略评论者...跟随Endorsed Only
修剪评论?
    []
  1. dearieme 说:

    我想有人被告知他的蹒跚学步的孩子很小,可能会花很多时间大喊“尺寸不是一切”。

    我想知道哪个智商测试侮辱了塔勒布先生。

    • 同意: Stan d Mute
  2. mijj 说:

    从无知和完全不愿意在这个主题上查找任何东西说起......难道智商测试的“智力”衡量的是一个泡沫占星师品质的信仰概念,这难道不是一个核心问题吗?

    • 回复: @Anon
    , @Svigor
  3. 著名的瑞士作家罗尔夫·多贝利 (Rolf Dobelli) 想了一两年,他会和尼古拉斯·塔勒布 (Nicholas Taleb) 成为朋友。 这导致了 Taleb 对 Dobelli 的极其不健全、不公平和不合理的人身攻击。 当我想到塔勒布时,我想到了马克思(极短的点火跨度)和巴枯宁:“如果我只有一个愿望,他喊道,那就是我对我来说神圣的尴尬感会一直保持在完美的状态塑造直到我的最后一天! – 安静,这就是让我生气的地方。” – 没有古兰经的穆罕默德。***

    *** 行情来自 – 汉斯·马格努斯·恩岑斯伯格 (Hans Magnus Enzensberger) 的“进步史中的三十七首民谣” – : – 我们的年龄在 126 页非常有趣的页面上(从帕多瓦的制表师 Giovanni de Dondi 开始***, 1318 – 1389 并以 Ernesto Guevara de la Serna 1928 – 1967(“人民战争中的漏洞”)结尾——介于两者之间:图灵(所有民谣中最好的)和美国发明家奥利弗·埃文斯——关于他的诗包含以下几行我最常想到的这本书:关于这个事实,磨坊在磨坊主中幸存下来。

    *** G. 德 D. 1318 – 1389

    来自帕多瓦的乔瓦尼·德·唐迪 (Giovanni de Dondi)
    度过了他的一生
    制作手表

    一款前所未有的腕表,无与伦比
    数百年。 (…)

    • 回复: @Jett Rucker
  4. Polynices 说:

    感谢您如此简洁地总结了所有内容。 我对塔勒布的无知感到沮丧,但无法清楚地说明原因。 他的许多批评者实际上都太愚蠢了,无法理解他在说什么,因此阅读有关他的“论点”并解释他的错误的明智批评是件好事。

    • 回复: @Anon
    , @Alfred1860
  5. “智力测试项目不是随意的。”

    这是 100% 错误的。

    • 回复: @DFH
    , @Anon
    , @Anon
  6. 谢谢你,汤普森教授。

    Taleb 通常读起来很有趣,而且充满洞察力。 他的咆哮通常很有趣,并且经常包含一些真相。 有时他们是准时的。 他们需要根据具体情况进行仔细检查。

    他在_黑天鹅_中有一个很好的部分,他区分了“专家是专家”和“不是专家的专家”,最终得出的结论是“有些领域没有专家”。 这是本书的一个很好的部分,值得思考。

    “有些领域没有专家”是一个具有挑衅性的假设,而且很可能是正确的——尤其是在选股等情况下。 你在上面提到的他最新的论战成果可能不是他最好的作品。 谢谢收听。

    也感谢您在处理来自“花生画廊”的评论时付出的时间和努力。

    • 同意: Wizard of Oz
    • 回复: @m___
    , @Bill
    , @HallParvey
  7. DFH 说:
    @RaceRealist88

    1. 如果没有连接心理状态和生理状态的规律,我们就不能有物理状态存在的证据
    2. 没有连接心理和生理状态的定律【你的前提】
    3.我们不能有物理状态存在的证据[reducio]

    • 回复: @RaceRealist88
  8. @DFH

    你如何定义“物理状态”?

    没有心理物理学或心理定律。 由于不存在心理物理学定律,因此精神不可还原为物理。 由于心理不可还原为物理,因此心理/心理状态无法还原为任何物理的东西——“基因”、生理学、大脑状态/结构。

    因此,心理特征——包括智力——无法归结为身体状态

    为 (3) 提供理由。

    • 回复: @DFH
    , @Loretta
  9. Anonymous[270]• 免责声明 说:

    天哪,这么多人关注智力不安全的塔勒布! 他一定在避风港。

    • 回复: @Wizard of Oz
  10. DFH 说:
    @RaceRealist88

    “物理状态”

    非精神。 从现在开始,我将使用它来更清楚。

    为 (3) 提供理由。

    不管你认为证据是什么,我认为如果说 A 成为 B 的证据需要 A 和 BI 之间存在联系,那么如果两件事之间没有联系,一个就不能成为另一个的证据,我认为这是没有争议的。

    更直接地说,如果我们的精神状态与任何潜在的非精神状态之间没有联系,那么我们的精神状态就无法为非精神状态提供任何证据。 所以没有理由认为非精神状态存在,或者,如果存在,它们与我们的(明显的)感知完全一样。

    • 回复: @RaceRealist88
  11. Anonymous [又名“红色任务”] 说:

    如果你读过他的书,你会发现塔勒布在那里同样夸张。

    他在相当多的时候是对的,但是通过他乏味的人身攻击来获取有用的材料是非常烦人的。

    • 回复: @Anon
  12. @DFH

    “非精神”

    有点模糊……你能详细说明一下吗?

    “身体”是指身体; “状态”是一个条件。

    “没有理由认为非精神状态存在”

    以“大脑状态”为例。 中枢神经系统的“快照”。 我们知道那是存在的。

    你如何看待戴维森的论点?

    • 回复: @DFH
  13. JLK 说:

    我已经在其他线程中更详细地介绍了这一点,这里不再赘述。

    Taleb 对组之间的差异差异提出了一个不错的观点(除了男性/女性差异之外,很难找到其他数据)。 否则,这似乎是试图混淆辩论,而且在这方面相当笨拙,尤其是对于一个拥有他所谓的智力的人而言。

    毫无疑问,这背后有一些政治因素,亚洲人对哈佛的诉讼,亚洲学生在 SAT 和 NMSQT 上更令人印象深刻的表现,以及 Unz Meritocracy 的文章,尽管事实上他可能是在他成为一个非人的路上。 想想塔勒布是华尔街人群中的阿拉伯人。 以下是新泽西州 2018 年 NM 半决赛选手的名单:

    https://worldscholarshipforum.com/wp-content/uploads/2017/09/358901119-Semifinalists-in-the-2018-National-Merit-Scholarship-Program.pdf

    新泽西州是一个有趣的案例,因为犹太人和亚裔人口都占总人口的 6% 左右。 我没有尝试列出数字,有些名字含糊不清,但快速的眼球测试告诉我亚洲人的出拳远远超过他们的体重。 考虑到双重加权的语言部分和缺乏空间成分都对亚洲的优势不利,这是令人惊讶的。

    将此与 Unz's Meritocracy atHarvard 的文章联系起来,这份名单上的犹太人名字似乎比我参加考试时 80 年代的人数少得多。 其中很大一部分可能是由于他们在 80 年重新调整考试中心时发生的口头部分的上限被截断(SAT-V 增加了 1995 分)。 1999 年有一篇关于普林斯顿争议的文章我在另一个帖子中引用了这一点,该帖子指出其他机构的犹太人入学率下降。 这可能与 1995 年重新定心一致。 通过某种机制,哈佛、耶鲁、宾夕法尼亚和哥伦比亚等某些学校似乎已经确信无论如何都将犹太人的入学率保持在以前的水平。 我认为他们在决定这样做时错过了一点,那就是犹太人的分数首先人为地高,因为这些测试是为了提升女性,而像犹太人一样,她们往往更擅长语言处理。 通过截断顶端而不是重新平衡测试以赋予数学和空间更多的权重,ETS 和 NMSQT 使右尾的女性人数尽可能多,同时减少了犹太人乘坐他们的尾随。

  14. Nobody 说:

    我一直在阅读 Taleb 关于智商的这些反驳,希望有人最终能从数学上解决他。 再次失望。 相反,我们看到更多的循环引用回到了最初的统计方法存在争议的领域,以及对他的语气的大量浪费性吹嘘。 科学史对你们这些骗子不会比他更友善。

    • 同意: onebornfree
  15. 嗨,汤普森先生,

    非常感谢为 Taleb 提供了一些逐点反驳,包括一些让读者自己查找相关研究的名字。 这比其他一些亲智商博主提供的要好得多。

    对我来说,塔勒布的批评是我不关心或不能很好地遵循的混合。 但是,我对他的说法很感兴趣,即 IQ 的预测能力适用于较低的 IQ 分数,但不适用于较高的 IQ 分数。 根据您的文章,我在 SMPY 中做了一些阅读。 我对 Taleb 所说的“预测性”的理解与 Lubinski 和 Benbow 的理解有所不同。 例如:

    智商可以预测你能否成功获得博士学位?

    来自 Lubinski 和 Benbow:

    例如,在我们对 20 岁青少年进行的 12 年随访研究中,SAT-M 或 SAT-V 分数为 30 或以上的参与者中有 500% 获得了博士学位,而得分为 50 或以上的参与者中有 700% 获得了博士学位。 在美国获得博士学位的基本利率是 1%……
    到 33 岁时,大约 25% 的队列 1 获得了博士学位,相比之下,队列 30 超过 2%,队列 50 超过 3%。

    所以我们可以看到,高智商获得博士学位的比率要高得多,而且这个比率与智商呈正相关。 然而,(1)即使在顶尖人群中,这个人是否拥有博士学位也基本上是一个掷硬币的问题,并且(2)这些数据并没有告诉我们关于谁的任何信息 尝试或想要 获得博士学位,尤其是在 1% 的基本利率中。 那么它有什么好处呢?

    (显然,Lubinski 和 Benbow 写的远不止获得博士学位,但这是一个例子。)

    军队的能力倾向测试经常被引用作为智商预测能力的证据。 正如您所注意到的,由于军队只接收智商低于平均水平的人,因此这是对右手的考验。 但同样,只有 5? 我不明白为什么亲智商对此印象深刻?

    我还想知道军方是否在军官晋升中使用智商/能力测试,尤其是在最高级别。 在我看来,如果 IQ 测试在曲线的右侧具有预测性,那么选择海军上将和将军的最佳方法就是对他们进行 IQ 测试。 这种思路有什么问题?

  16. 在一个稍微不同的话题上,我注意到 Sailer 已经转发了 Pinker 关于 IQ 研究不可复制性的推文,但 Pinker 的推文链接到一个页面,该页面只有会议发言人的简历。 他没有链接到证据:

    这个谷歌链接转到:

    http://www.isironline.org/2015-albuquerque-new-mexico-september-18-20/

    同样,您声明可复制性,但您不会删除任何研究人员的姓名或提供链接。

    事实上,心理测量学的主要发现与心理学的其他领域一样好,甚至更好。 只是很多人讨厌结果。

    也许你可以编辑你文章的这一部分?

    • 回复: @utu
    , @Anonymous
    , @James Thompson
  17. utu 说:
    @Chrisnonymous

    如果“心理中的可复制性危机不适用于智商”平克先生,您为什么认为本文中的汤普森博士没有引用一个数字来说明相关性、成功的可预测性、遗传性以及智商分数应该执行的其他任何内容如此美妙? 他没有记住,因为他不记得这些数字中的任何一个,因为它们被放在非常宽的括号中,如果他引用了其中任何一个,它们可能会受到质疑。

    • 回复: @Wizard of Oz
    , @CanSpeccy
  18. Hail 说: • 您的网站

    一项人口范围的衡量标准(一般智商)正在与仅由一些更聪明的学生(SAT)进行的学术考试进行比较,但仍然很好地展示了两者之间的联系。 这是一个现实生活中的发现,据说是塔勒布喜欢的那种。

    这就提出了一个问题,即在塔勒布是无所不能的独裁者的世界中,大学招生将如何处理:

    如果 SAT 和 IQ 相关,
    如果智商必须不惜一切代价进行斗争,因为它无论如何都没用,
    那么SAT也是无用的伪科学骗局,
    因此,大学录取应该按照以下方式进行判断:

    与申请人的全面面试,其中一个 Taleb 批准的团队面对面地评估申请人,以确定申请人是否只是一个纸上的洗牌者(或更糟),或者非常适合现实生活(在这种情况下,录取信正在路上)。

    • 回复: @Wizard of Oz
  19. Anonymous [又名“RFN”] 说:

    作为工程经理发言

    塔勒布屁股向后

    你需要让智商在 120 到 140* 之间的人来填补你的部门。

    智商低于 115 的几乎没有人会成功。

    * - 我不反对智商更高的人,但很难找到他们。

    (我不能引用研究论文。以上是基于个人经验。)

    • 回复: @res
  20. Anonymous [又名“亚马逊”] 说:
    @Chrisnonymous

    我全神贯注地看着这一切,而对该领域的具体情况知之甚少。 我没有受过教育的印象是,尽管文章中有连贯的逐点回应(谢谢),但作者和 Taleb 仍然没有完全交谈。

    我想强调之前的评论者的观察,即支持 IQ 的人群坚持认为心理测量学没有受到复制危机的影响,从而为 IQ 辩护。 是的,智商是心理测量学的一个子集,但塔勒布的批评不是关于大 5 等的,尽管我明白为什么这会留在他夸夸其谈的语言中。

    无论如何,感谢作者,我将在一段时间内将其与任何其他未来报告一起使用,以便我可以通过所有参考资料和必要的数学来弄清楚。

    • 回复: @Che Guava
  21. J 说: • 您的网站

    智商测试具有预测性。 为什么我要让塔勒布的诡辩术迷惑我?

    • 回复: @Johnny Horton
  22. 他讨论的一个方面是最令人反感的:他以极其随意的方式抛弃了技术统计和/或数学术语,例如凸性、传递性、单调性、风险度量。 它的目的是让读者眼花缭乱,但如果您受过训练来理解这些术语,它只会显得具有欺骗性。 他的讨论风格与这些术语的严格统计/数学定义没有牢固的联系。 关于科学主题的流行写作可以做得很好,也可以以欺诈方式完成。 他的文章对我来说闻起来像欺诈。 有点像 Stephen Jay Gould 的东西,但更明显是欺诈。

    • 同意: res, utu, Philip Owen
    • 回复: @Wizard of Oz
    , @Bill
    , @utu
  23. 关于 Taleb 关于 IQ 是循环的观点:

    看起来,拥有高智商肯定会让一个人在足够稳定的社会环境中取得更大的成功,从而产生普遍可预测的结果。 然而,在这样的环境之外,智商几乎没有那么有用,实际上整个概念变得越来越没有意义。

    这就是我们应该如何解释智商的“环境”方面。 不是环境因素直接提高或降低个人的分数; 是环境的性质决定了现象首先出现的程度。 智商分数反映了社会可预测性在个别情况下的体现程度。 分数越低,个人的社交惰性越强。 他没有意识到他周围世界的丰富意义,仍然是一个流浪者,或者一堆原材料。 分数越高,社会认可的意义领域就越与个人自己的思想一致。

    我相信如果维特根斯坦在这里权衡,他会同意这一点。

    • 回复: @DFH
    , @Anonymous
  24. J 说: • 您的网站

    我想说的不是另一个犹太人的阴谋,而是 2018 年的年轻美国犹太人与 1980 年代的素质不同。 通婚率超过70%。

    • 回复: @Franklin Ryckaert
  25. DFH 说:
    @RaceRealist88

    有点含糊

    它很宽泛,但非常精确。

    以“大脑状态”为例。 中枢神经系统的“快照”。 我们知道那是存在的。

    您可能对此有一种看法,但是,如果您认为精神和身体之间没有联系,那么您就没有任何理由认为它确实存在,就像我之前论证的结论一样。 这就是你的主张导致的荒谬。

    • 回复: @RaceRealist88
    , @Gssere
  26. J 说: • 您的网站

    PS:除了美国人口普遍下降。

  27. Loretta 说:
    @RaceRealist88

    那么脑损伤是如何让人变得更笨的呢? 求一个朋友。

    • 回复: @Anonymous
    , @RaceRealist88
  28. 完全无法理解理性人对智商测试的大惊小怪。
    智商衡量智商,仅此而已。
    据我所知,没有人能够以可以衡量的方式定义智力,因此,智商不能衡量智力。
    智商测试是相关的,因为任何雇主都知道谁要求求职者接受测试。
    因此,智商测试也与判断任何人(包括移民)融入欧洲工作文化的程度有关。
    这些事情当局极力保密,但荷兰铁路和警察学校都知道,智商低的人很可能失败,无论是在简单的铁路工作中,还是在完成警察培训方面。
    警察学校降低了最低智商,以让更多有移民背景的学生完成学业。
    讨论的背景当然是清楚的,如果智商存在显着差异,那么很多抱怨种族主义和歧视就变成了无稽之谈。

  29. Anonymous[364]• 免责声明 说:

    优秀的文章。 塔勒布对他的主张非常粗心 看似自信。 这是一个糟糕的组合。

    一个诚实的人会仔细检查核心问题并提出他们的理由。 一个不诚实的人会把每一个红鲱鱼和半生不熟(已经被反驳)的“挑战”扔到墙上。 他们甚至不希望“有什么东西会粘住”——他们希望纯粹的数量和信心会欺骗那些不了解细节的人。

    • 回复: @Anonymous
    , @Bill
  30. Anonymous[364]• 免责声明 说:
    @Chrisnonymous

    所以我们可以看到,高智商获得博士学位的比率要高得多,而且这个比率与智商呈正相关。 然而,(1)即使在顶尖人群中,无论这个人是否拥有博士学位,这基本上是一个掷硬币的问题,并且(2)这些数据并没有告诉我们谁试图或想要获得博士学位,特别是在1% 的基准利率。 那么它有什么好处呢?

    你在开玩笑?

    • 回复: @Chrisnonymous
  31. m___ 说:
    @charles w abbott

    “花生画廊。”

    被铸造。

    • 回复: @Sparkon
    , @m___
  32. @dearieme

    我想知道哪个智商测试侮辱了塔勒布先生。

    耶稣,好。 这是我想知道的第一件事。 第二件事是,他就像男人走自己的路。 MGTOW,或任何格式。 他们不喜欢今天的女人,所以他们要离开了,女人是狗屎,他们不需要她们。 或者讨厌选美比赛和比赛中获胜者圈子的丑女权主义者,因为那里有漂亮的女孩。 Teleb 是一个智商测试不及格的人,他讨厌那些表明他不如自己高评价的人。 他充满敌意,但我知道有一种测试集在只考虑优点时有效。

    我,我显然是个白痴,几乎没有通过代数 1,老实说,我几乎没有通过高中,因为女孩、摩托车、高尔夫和锅更有趣。 我完成了 C 和 D,期间。 我读了很多,我们每天都有三份报纸敲门,但我不是学生,也不是学者。 我的两个兄弟姐妹是,但不是我,呵呵

    但是海军用一些衡量我效用的套装测试了我,即武装部队入学考试。 艾菲? 无论如何,在我签署合同之前,在一次用笔和纸的坐下,也许一两个小时,他们决定我可以作为飞行甲板老鼠,为弹射座椅、液氧和飞机空调提供服务。 AME 他们称之为。 我很出色,我基本上是一个好孩子,三年内取得了 E5,在他们杀了我之前,在将近 6 年里光荣地退出了。

    在我的服役过程中,我遇到了 999 万名水手和飞行员,我不得不说,1000 人中有 1975 人完全适合显然复杂的技术和航空壮举,并具备执行它们的疯狂或勇气。 可以肯定的是,一旦您掌握了基础知识,海军航空队就没有太多时间进行 OJT。 他们的测试非常准确,尽管我在 XNUMX 年注册的那个时间范围内读到过一段时间,当时我在那里接受了白痴服务。 也许我是其中之一,我不知道。

    那么他们怎么知道,在一个纯粹的基于功绩的系统中(因为今天功绩已经死了,所以我是故意的),他们怎么知道我会在训练和学习中达到他们的标准,他们怎么知道我有足够的疯狂在飞行甲板上跳舞,在 18 岁时提供工作弹射座椅和可呼吸的氧气? 因为仅仅看我高中的记录,你就会认为(我不否认),“失败者”。 这将是一个公平的假设。 军方的测试和招募是怎么知道我能做到的? 因为即使是现在,我也不知所措。 任何人? 仅仅从这些测试中,他们怎么知道数以百万计的新兵? 他们非常非常好。 所以我只是在考虑 Teleb fulla shit。

    • 回复: @Jim Christian
    , @Anon
  33. @Jim Christian

    哦,顺便说一句,他们怎么知道我不会在海上被关闭两年而不会发疯而发疯? 还是我们中的任何人?

  34. 让我们切入正题,塔勒布:他不想接受智力存在种族差异,黑人——总的来说但总是——处于最底层,他怀疑自己的人民,不管他们是谁是的,也不要得分那么好。 所有这些回避问题而不是命名它是乏味的。

  35. APilgrim 说:

    Nassim Nicholas Taleb 是黎巴嫩人,大概是穆斯林。 黎巴嫩的平均智商为 82。 https://brainstats.com/average-iq-by-country.html 穆斯林国家的平均智商为 81,比非穆斯林国家的平均智商低半个标准差。 阿拉伯国家的平均智商为 84。 http://www.mankindquarterly.org/archive/issue/50-3/2

    • 回复: @Anonymous
    , @stretch23
    , @mikesmith
  36. Anonymous[364]• 免责声明 说:
    @Anonymous

    “An 诚实”,该死的。 英语不是我的母语,但在智商讨论中它仍然是一个严重的错误。

    • 回复: @Anon
  37. DFH 说:
    @Intelligent Dasein

    然而,在这样的环境之外,智商几乎没有那么有用,实际上整个概念变得越来越没有意义。

    你有任何证据证明这一点吗?

    • 回复: @Intelligent Dasein
  38. Anon[379]• 免责声明 说:
    @mijj

    如果您完全无法抗拒发表评论的冲动,那么您的开场白就得到了很好的判断。

  39. Anonymous[364]• 免责声明 说:
    @Intelligent Dasein

    看起来,拥有高智商肯定会让一个人在足够稳定的社会环境中取得更大的成功,从而产生普遍可预测的结果。 然而,在这样的环境之外,智商几乎没有那么有用,实际上整个概念变得越来越没有意义。

    这是没有意义的。 在所有其他条件相同的情况下,如果在荒岛上跳伞,一个高智商的人将有更好的生存机会。 他们会 弄清楚 更好更快。

  40. Anon[379]• 免责声明 说:
    @Polynices

    “他的许多批评者”???? 他的许多追随者当然更是如此。

  41. Anon[379]• 免责声明 说:
    @RaceRealist88

    欢迎回到精神无政府主义者。

  42. @Anonymous

    我想你可能已经把他搞定了。 尽管我试图阅读他的第一本书《黑天鹅》中的一些令人兴奋的段落,但我形成的压倒性印象是一个傲慢但缺乏安全感的非常聪明的废话艺术家。

    • 同意: Bill
  43. @JLK

    抱歉,如果我说得有点模糊,但你能解释一下高端的截断是如​​何继续让女性受到青睐而不是犹太人吗? 也许我不完全理解截断所涉及的内容。

    • 回复: @JLK
  44. Sparkon 说:
    @m___

    被铸造。

    D不要成为Flub-a-Dub。 根据谷歌的 N-gram 查看器,自 1870 年左右以来,“花生画廊”这个词就一直被普遍使用。 该术语指的是(杂耍表演)剧院中的廉价座位,在那里吵闹的顾客有时会在舞台上扔花生以表示不赞成。

    大多数布布管一代是由 Howdy Doody 节目引入的。

    Howdy Doody 节目于 1943 年首次出现在广播中,并于 1947 年过渡到电视。

    Howdy Doody 及其花生画廊的受欢迎程度导致 United Features Syndicate 的高管使用这个名字 花生米 为 Charles M. Schulz 的联合组织 小伙子们 漫画,据说让舒尔茨终生懊恼

    Flub-a-Dub 是节目中的另一个角色。

  45. @Nobody

    已经要求 Ron 安排他的一位数学伙伴对 Taleb 的数学开膛,如果不是他自己的话,我对您的评论表示同情。 但我认为你对那些发现 Taleb 不了解文学的作者有点苛刻。

  46. Anonymous [又名“阿姆斯汀”] 说:
    @APilgrim

    纳西姆是希腊东正教黎巴嫩少数民族和希腊族。 他是 75% 的希腊人,25% 的亚美尼亚人。

    • 回复: @Anonymous
    , @Hail
  47. Bill 说:
    @charles w abbott

    塔勒布在谈论他擅长的事情时非常出色。然而,他的专业知识似乎并没有超出价外期权的定价。

  48. Anonymous[863]• 免责声明 说:
    @Anonymous

    “Nassim”和“Taleb”是希腊名字吗?

    • 回复: @APilgrim
  49. AaronB 说:

    人们可以通过查看一个国家或一个人的具体成就模式来构建一个信息密集得多的国家或个人潜在表现的画像。

    智商是一个抽象的数字,因此与检查具体成就相比,所传达的信息必然少得多。 它抽象了某些关键特征——即它失去了信息密度。

    假设是说,日本和中国一些城市的智商相似,那么我们可以期待他们的表现模式相同。 我们现在知道这是误导。

    IQ 会“拉平”地图上的轮廓和凹凸,以生成统一的图形,但会丢失信息密度和细节。

    抽象优先于具体总是会导致信息密度和细节丰富性的损失——只要有可能,具体是 时刻 优先于摘要(按设计选择)。

    那么智商什么时候有用,什么时候没用呢? 在我们没有一个人的具体成就记录的情况下——当我们需要确定 潜力 没有此人在现实世界中的表现的历史,可以将智商用作试探性工具。

    这实际上就是它的设计目的——从不作为 替代 为具体的成就记录。 现实以其丰富的具体细节,总是应该胜过具有选择性的抽象形象。 智商是我们知识中的权宜之计,一种启发式方法,是一个人在现实生活中展示能力之前的临时工具。 智商旨在帮助识别潜在人才——从不权威地断言其存在或不存在。

    多年来,智商以某种方式被具体化了——人们现在自信地断言,那些未能表现出特定能力的国家在未来肯定会这样做,因为一个抽象的数字旨在帮助在没有成就记录的情况下帮助识别潜在的人才比 成就的实际历史记录。

    或者其他民族在未来肯定无法展示才华,因为这个抽象的数字比众所周知的历史记录更具权威性,该记录表明国家随着时间的推移在具体成就方面经历了急剧的上升和下降。

    这就是将启发式具体化的意义——信息密度和具体细节的戏剧性损失,现实中可观察模式的可配置性的戏剧性损失(以及最终否认这些模式),失去了有利于柏拉图思想的具体时间感,以及一种贫乏的现实感。

    我们还可以看到,当涉及到国家比较时,智商是相当没有意义的——通过大量不同时间条件的具体成就的历史记录传达了一个更现实的画面——以及更精细、更具体的画面。

    相反,众所周知的“上升和下降”模式使短期预测以外的任何事情都变得非常成问题。 建立柏拉图式的民族观念的尝试是非历史性的。

    同样,一个人的智商分数远不如他的成就记录——甚至是长时间的谈话——在评估他的智力方面有用。 而一个高智商的人,如果谈话总是迟钝(我个人经历过很多次),或者反复表现出能力失败,可以说不是很聪明。

    当然,还有一个问题是,能力只能根据动机和目标来衡量,除非所有个人和群体在这些方面都可以说是相同的(这会奇怪地与 HBD 假设相矛盾,即群体在所有特征上都不同),测量能力有问题……房间里的大象没人想解决……

    无论如何,这只是冰山一角……

    在一段时间内,向抽象思维和广泛概括的转变在历史上是有用的,但今天,我们所处的位置是,构建一种更智能的现实方法意味着远离抽象和一般,转向具体,而精确度的损失将通过细节的丰富性和对现实中实际模式的更好一致性来补偿,而无法建立永恒法则和发现柏拉图思想将被应对现实的现实灵活性所抵消。

    • 同意: wayfarer
    • 巨魔: DFH
    • 回复: @utu
    , @Sylsau
  50. @utu

    我怀疑存在虚拟或逻辑复制,而不是迂腐相同的复制,比如将 25 具 25 至 28 岁、BMI 均在 30 至 35 岁之间的新鲜苏格兰男性尸体切成薄片。

    • 回复: @utu
  51. @Hail

    嘿,他和他的爸爸编制了一个算法交易系统,仅仅从被其模拟所说服的投资者那里,他们就赚了几十万。 现在,当他们为纳西姆·尼古拉斯·塔勒布(Nassim Nicholas Taleb)金融雄辩主席(俗称金钱谈判主席)提供 10 万美元时,这已经足够真实了。

  52. Polymath 说:

    塔勒布争论的基本观点是现实世界中与将细尾模型应用于具有肥尾现象的领域相关的误测和风险。 仅从信息论就可以看出,这样的模型肯定无法捕捉到很多东西。 可以从序数数据重新调整测量以反映这一点的论点是虚假的,因为测试本身的构建已经朝着使原始分数遵循钟形曲线的方向发展,因此重新调整仍然会提供相对缺乏信息的测量。

    • 回复: @James Thompson
  53. Anon[379]• 免责声明 说:
    @Anonymous

    但它是肛门或无知的,无论你的第一语言不能识别出如此明显的错别字/文字,还是校对者可能会称之为。

    • 回复: @Anonymous
  54. wayfarer 说:

    银河系中最高效/最有效的算术和代数教程!
    来源: https://themathpage.com/

  55. Bill 说:
    @Peter Johnson

    是的。 当他远离期权定价时,这似乎会发生。

  56. Bill 说:
    @Anonymous

    换句话说,像律师一样争论。 争论就像一个想要特定答案而不是真相的人。

    • 回复: @Philip Owen
    , @Dieter Kief
  57. 当您使用 Piaget 知情镜头时,智商实际上衡量的是有限的思考能力。

    本文解释了天花板效应导致的问题: https://dareassociation.org/bdev/bdb_archive/BDB_21.1-50-62.pdf

    因此,智商评估需要与不受这种天花板效应影响的评估一起使用 - 除非您对一个人的思维能力的有限情况感到满意。

    该论文的标题是:“使用层次复杂性(MHC)模型对韦克斯勒成人智力量表第四版(WAIS-IV)的言语理解指数进行分析:为什么阶段可能比言语更好地衡量“聪明人”智商?

    这可能是塔勒布在不知不觉中绊倒试图说的话? (而且基本上没有)。

    如果我认识他,我会亲自问他。

    • 回复: @Anonymous
    , @Wizard of Oz
    , @utu
  58. Anonymous[364]• 免责声明 说:
    @Anon

    我在纠正自己。

    • 回复: @Anon
  59. Anon[436]• 免责声明 说:
    @Jim Christian

    多年来,我也有非常可靠的观察和传闻证据,证明智商测试具有现实基础,使它们具有潜在的实用性,并且平均而言是预测指标。

    我妻子的弱点我当然知道。 我认为我对很多事情的记忆都更准确,但她的阅读速度比我快得多,而且似乎更容易掌握剧情或情节。 她是一个非常好的无笔记演讲者,可以娱乐和告知,并写了几本非小说类书籍。 她似乎尊重我的大脑。 我记得她的母亲是一位非常有经验的老师,她告诉我,我的妻子在很小的时候就出现了阅读障碍(实际上只有在成年后才使用科学名称才成为可靠的拼写者),所以她接受了智商测试(我两次思考)。 可以说它比我的高两个标准差就够了! 也就是说,假设是斯坦福比奈或韦克斯勒,但很可能是卡特尔,这将使我们大致相同。 我经验丰富的老师婆婆对智商测试的有效性毫无疑问,因为她能够预测她所教的大多数孩子的结果。

    碰巧我被跳进了比我领先 7 次的班级,13 岁时(因为我没有上过学),16 岁时进入一个平均年龄比我大 15 个月的班级,因为校长是前一年尝试了一个小型精选课程,其中有两个加速,在 2 岁时,因为我和 XNUMX 岁大的孩子一起被推进到顶级物理和拉丁语。 在每一种情况下,我都有经历过一个学期来赶上并成为班上的佼佼者或非常接近班上的佼佼者(在不成熟或被视为运动员的愿望接管之前)。 这对智商有效性有何影响?

    我回忆起在 9 岁和 10 岁进行的 IQ 测试,并记得它们非常相似,以至于我实际上可以玩第二个。 然而,当我在高中的最后一年时,我有一天可以访问学校的所有学术(和其他)记录,包括智商分数。 有一个终生的朋友,他的最高分数比我高出几个标准差,甚至更多,这可以说体现在他回国前在名牌大学成为杰出的粒子物理学家。在最高水平上追求另一个要求很高的职业。 (正如我在提出他的健康状况时所说的,并回忆起他写诗,记住别人在戏剧中的角色,并在暑假期间因原创项目获得科学奖,我在足球方面做得更好)。 其他人包括一些教授和领先的专业人士,但我毫不怀疑,除了我最聪明的同时代人之一(他的气质和兴趣非常不同:他实际上做了化学反应)实验)是真实的,因为我只是在有动力时做得更快。 我认识一些非常愚蠢的人,他们在智商测试中测试得很高,人们可能会过得很糟糕,然后是弗林效应,可能有多种原因。 也许需要一个智商高于平均水平的大脑才能充分利用智商分数。

    • 回复: @Prof Watson
  60. @dearieme

    “我想知道哪个智商测试侮辱了塔勒布先生”

    诉诸动机。

    • 回复: @dearieme
  61. @DFH

    “你可能有这样的看法,但是,如果你认为精神和身体之间没有联系,那就不能让你有任何理由认为它确实存在”

    这是真的。 我们可以看到大脑状态。 因此,存在物理状态。

    • 回复: @DFH
  62. @Loretta

    精神需要大脑,没有大脑就没有精神。 然而,这并不意味着精神可以还原为肉体。

    然而,Sophocles、Shakespeare、Tom Stoppard 或 Elfriede Jelinek 可以教会我们更多关于我们自己的知识,而不是神经科学。 神经科学关注我们的大脑或中枢神经系统及其功能模式。 没有大脑就没有头脑。 大脑是人类思维的必要条件。 没有它,我们将无法过上有意识的生活,而只会死而复生。 但它与我们有意识的生活并不相同。 人类意识的必要条件远非充分条件。 有腿是骑自行车的必要条件。 但这还不够,因为我必须掌握骑自行车的艺术,并且必须与我的自行车出现在同一个地方,等等。 相信一旦大脑被理解,我们就完全理解了我们的思想,就好像我们相信一旦我们的膝盖被理解,我们就会完全理解骑自行车。 (马库斯·加布里埃尔,我不是大脑:21 世纪的心灵哲学)

    • 回复: @Poco
    , @Loretta
  63. res 说:
    @Anonymous

    你采用什么方法来寻找 IQ 120 到 140 之间的人?

  64. Taleb 我们是一个聪明、有见地的人,但他有他的 IYI 时刻。 这似乎是一个。

    他关于凸性在某种程度上是正确的,但在债券数学中,这只是重要的,因为使用更简单的数学来模拟利率变化(久期)影响的曲线,因此当您偏离该点时,凸性就会成为一个问题你锚定测量。 在这方面,塔勒布是正确的,智商测试更准确地确定了那些接近平均水平的人; 通过围绕实际 IQ 的测试估计的 IQ 存在偏差,这是无法观察到的,并且当您偏离平均值时,该偏差会更高。 由于这个原因,塔勒布对垃圾 IQ 测试是错误的,因为围绕不可观察的实际值的一系列估计分散并没有错,只是不太准确。

    顺便说一句,在十进制值中添加前导零会很有帮助。

  65. @Chrisnonymous

    博士学位率是人口基数的 50 倍。 那不是抛硬币。 如果我们有一个人口基数,那么人们想要攻读博士学位的比率将是相关的,但完成率是最好的衡量标准。 博士的最高智力水平与一般人群之间的差异是巨大的。

    .5 在行为研究中具有相当高的相关性。 修正了范围限制,它会更高,可能是 6 甚至 7

    智商测试在所有级别的工作中都很有用。 对于更高的级别,为高能力候选人设计的更专业的测试会更好。 要求严格的职业会根据他们的要求制定自己的测试。

  66. 研究表明,男性思考逻辑,女性思考情感。 从个人经验来看,如果你试图与她进行合乎逻辑的交流,你就无法赢得与她的争论。 当然,有的男人接触到了女性的一面,也有的女人接触到了阳刚的一面。 女性有两个 XX,没有 Y 染色体。 更不用说不同的性激素了。

  67. DFH 说:
    @RaceRealist88

    我们可以看到大脑状态。

    我承认你可以有一种声称是“看到”脑部扫描的精神状态(即你“看着”脑部扫描的视觉体验)。 但是如果精神状态和非精神状态之间没有联系,那么精神状态就不能提供任何非精神状态的证据(即脑部扫描或大脑的存在)。 因此,说“我们可以看到大脑状态”只是在回避问题。

  68. @Anon

    我看一个人的成就,而不是智商测试结果。 除了考试成绩,影响成绩的因素还有很多。 在大学里,即使是 SAT 考试也被视为种族主义者。 社区激进主义的评级更高。 这有助于实现我们的多样性配额。 在今天的校园里,由于教授潜在的种族主义偏见,甚至成绩也令人怀疑。

  69. Anonymous[364]• 免责声明 说:
    @Miriam Spence

    当您使用 Piaget 知情镜头时,智商实际上衡量的是有限的思考能力。

    你能用你自己的话解释一下这个问题吗? 什么是伯爵知情镜头? 快速、肤浅地查看那篇论文并不能激发信心:

    MHC 是一个非精神主义的新皮亚杰主义者,
    和定量行为发展理论。
    它提供了一种标准的检查方法
    普遍的发展模式。 一个基本的
    假设是发展继续
    跨越大量的一般序列
    的行为。 这些序列存在于
    每个域,包括但不限于
    数学、逻辑、 科学的、道德的、社会的、
    和人际领域
    .

  70. @Miriam Spence

    我注意到链接的论文似乎没有明确提到 Raven's Matrices。 我对 Thompson 博士关于这如何影响论文的力量的看法很感兴趣。

    • 回复: @utu
  71. @Anonymous

    动物生活在荒岛上、海底、无踪的荒野、冰盖下以及全球其他任何地方。 动物根本没有推理能力,更不用说我们称之为智商的东西了。

    你应该考虑一下。

    • 回复: @Anonymous
  72. 你在参加罗恩吗? 我仍然希望你,或者你有数学能力的伙伴之一能多纳数学去除塔勒布。

  73. @DFH

    你有任何证据证明这一点吗?

    是的,我有 所有 证据。 我有整个 HBD 论点。

    黑人智商较低。 黑人几乎没有形成复杂社会的才能。 尽管如此,黑人还是在无数危险中在非洲荒野中生存——低智商、糟糕的组织能力等等。

    智商似乎不是复杂社会之外的成功所必需的。

  74. Rich 说:
    @Anonymous

    我不知道,我认识一些高智商的人,他们不会钉钉子、扔球或诱饵。 我认识一些高中辍学或勉强过日子的人,他们会钓鱼、打猎、修理发动机或举起重物,我预计他们在那个荒岛上会做得更好。

    把高智商的人丢在华尔街、实验室或会计办公室,他可能会做得更好。 我会给你的。

    • 回复: @Anonymous
    , @Svigor
  75. nsa 说:

    Wiki 指出,早在 1990 年代,Unz 声称 IQ 为 214。如果是这样,如何衡量超过 200 的数字? 或者这只是一个典型的虚假 lugenpresse 刺戳嘲笑 Unz 竞选加州州长? 像 Fischer 和 Carlsen 这样的国际象棋大师的 IQ 要求仅在 180 范围内,没有任何支持方法,即可能只是估计? 贝尔曲线的两端是否连接形成一种莫比乌斯连续体? 例如,爱因斯坦的姐姐/看守人声称有一天这位伟人在一只脚上放了两只袜子,然后用了剩下的一天时间寻找丢失的袜子。 当姐姐解开谜团时,艾尔从此不再穿袜子,从而解决了这个问题。

  76. utu 说:
    @Peter Johnson

    他以极其随意的方式抛弃了技术统计和/或数学术语,例如凸性、传递性、单调性、风险度量。

    毫无疑问,他是想炫耀和迷惑。 但他的攻击精神是健全的。

    • 回复: @Dave Bowman
  77. dearieme 说:
    @RaceRealist88

    只有傻瓜才会忽略这种可能性,即诉诸动机可能与理解塔勒布先生在该主题上的情绪化密切相关。 毕竟,完全有可能对 IQ 测试抱有怀疑,而不会对它们歇斯底里。

  78. Mike1 说:

    我研究过 Taleb 参与的公司,他们的策略是高杠杆。 不完全是开创性的。
    我喜欢这个人,但“你必须过我的生活”的套路已经过时了。
    您说的非常长期回报约为 7% 是正确的,但过去 3.145 年的数字为 500%(标准普尔 XNUMX 年回报率(股息再投资和通货膨胀调整后)。即使如此高,也需要当前的估值。

    • 回复: @Anonymous
  79. Anonymous[364]• 免责声明 说:
    @Intelligent Dasein

    好吧,即使你听起来像是生活在冰盖下。

    关键是,即使在一个新的、不可预测的环境中,一个聪明的人也会做得更好。

  80. RW 说:

    这就像听一个没有物理学背景的人谈论气候变化一样。

  81. Anonymous[370]• 免责声明 说:
    @Rich

    这就是为什么我说:“所有其他条件都相同”。 我们谈论的是同样的知识、经验、韧性、意志力等。即使在荒岛上,智力也是一个巨大的优势,但它不是唯一的优势。

  82. JLK 说:
    @Wizard of Oz

    抱歉,如果我说得有点模糊,但你能解释一下高端的截断是如​​何继续让女性受到青睐而不是犹太人吗? 也许我不完全理解截断所涉及的内容。

    语言是女性和犹太人的强项,但犹太人的平均语言水平更高,并开始在分布曲线的右侧尾部占主导地位。 1995 年重新定心截断了右侧尾部。 一下子,以前在 SAT-V 上得分 720-800 的每个人都是平等的。 这对犹太人和非犹太人的伤害比对女性和非犹太人的伤害更大,因为 720+ 的范围远远超出了犹太人的数量不成比例,并且没有太多女性受到影响。 在重新居中之前,即使是非犹太男性在这个范围内的表现也优于女性。

  83. @Intelligent Dasein

    黑人几乎没有形成复杂社会的才能。

    他们有吗?
    扬·范西纳(Jan Vansina),“稀树草原王国,中非国家直到欧洲被占领的历史”,伦敦,1966年
    扬·范西纳(Jan Vansina),《伍特的孩子们,库巴族的历史》,1978年,麦迪逊
    “非洲考古、食品、金属和城镇”,编辑。 Shaw、Sinclair、Andah 和 Okpoko,伦敦和纽约 1993

  84. Anonymous[370]• 免责声明 说:
    @Mike1

    我研究过 Taleb 参与的公司,他们的策略是高杠杆。 不完全是开创性的。

    声音 有风险.

  85. Svigor 说:
    @Rich

    你的意思是那些从来没有钉过钉子的人,或者不想这样做,或者有一些相关的残疾? 因为我怀疑你是否真的认识身体健全、智商高的成年人,他们不会钉钉子等等。

    • 回复: @Rich
  86. @James Thompson

    这是一个我一直希望得到明智答案的问题。 他们是否对导致高智商人士在生活(甚至只是职业)中失败的原因进行了很好的研究?

  87. DFH 说:
    @Intelligent Dasein

    黑人智商较低。 黑人几乎没有形成复杂社会的才能。 尽管如此,黑人还是在无数危险中在非洲荒野中生存——低智商、糟糕的组织能力等等。

    智商似乎不是复杂社会之外的成功所必需的

    这些都没有表明更高的智商在这种环境中没有/不会有助于生存(并不是说非洲在很大程度上是一个非常难以生存的环境,抛开疾病的威胁)。

  88. Gssere 说:
    @DFH

    我喜欢他是多么愚蠢,以至于他一直在说“精神不能还原为身体”,而这实际上并不是所提出的要求。 我们的大部分心理活动必须是非物质的,没有人“见过”意识。

    任何年纪大到喝过几杯啤酒的人都知道,精神状态和身体状态之间存在直接联系。 认为它们彼此之间完全没有联系是荒谬的。

    很高兴他接受脑部扫描数据是真实的。 我不知道他是否熟悉脑部扫描研究,其中在受试者有意识地意识到(即决定按下按钮)之前在大脑中观察到物理变化(运动皮层的激活)。 我不想相信物理决定论,但这是一个非常可怕的发现。

    • 回复: @jilles dykstra
  89. m___ 说:
    @m___

    每一代人都有它的死角。 你是对的更正,这个词对于我们“评论者”来说是合适的,至少是浮动的。

  90. utu 说:
    @Wizard of Oz

    你没有注意到标题中的“言语智商”吗?

  91. @Chrisnonymous

    由于 Taleb 认为他可以放弃一个世纪的心理测量学,因此有很多参考资料可供参考。
    尝试按照博客中的建议搜索 Detterman。
    这是一个开始 /www.unz.com/jthompson/dettermans-50-year-of-seeking
    这是另一个捷径: https://www.unz.com/jthompson/intelligence-all-that-matters-stuart/
    另一个捷径: https://www.unz.com/jthompson/intelligence-in-2000-words
    然后阅读 Jensen:g 因子
    然后阅读《智能》杂志。

    那会给你一个开始。

  92. utu 说:
    @Miriam Spence

    很好的观点和 Featherston 等人的论文。 非常有趣,但它会超出像绿野仙踪这样头脑简单的 IQ 主义者的头脑,他们已经抱怨 Raven 矩阵没有包含在论文中。 无论如何,塔勒布可以将这篇论文纳入他的论点。

    • 回复: @Miriam Spence
    , @Wizard of Oz
  93. Agent76 说:

    14年2013月XNUMX日,关于学校的真相

    您是否曾经想过,孩子从幼儿园到高中要花13年的时间为生活做准备,但是当他们出去时,他们却没有任何真正的技能吗?

    • 回复: @jilles dykstra
    , @Anon
  94. Anonymous[370]• 免责声明 说:
    @Anon

    不过还是谢谢关心。 🙂

  95. utu 说:
    @AaronB

    好帖子。 你不会接触到智商主义者,因为他们是物化的受害者。 将需要严重的反编程。 一旦你到了物化的啦啦地,就很难再回来了。

    • 回复: @AaronB
  96. @Gssere

    我们的大部分心理活动必须是非物质的,没有人“见过”意识。

    没有心理活动这样的东西,除非大脑中的身体活动被称为心理活动。
    罗杰·彭罗斯(Roger Penrose),“皇帝的新思想,关于计算机,思维和物理定律”,1989年,牛津

  97. @Wizard of Oz

    你应该首先定义你认为生活中的失败。
    Faurisson 是失败还是成功?

    • 回复: @Wally
    , @Wizard of Oz
  98. utu 说:
    @Poco

    比你链接到 Michael Egnor 的文章。

  99. AaronB 说:
    @utu

    谢谢。 不幸的是你是对的。

  100. Agent76 说:

    12 年 2017 月 12 日,这个 XNUMX 岁的孩子在智商测试中超越了阿尔伯特·爱因斯坦、斯蒂芬·霍金

    我们中的大多数人都为在课堂上获得满分而感到高兴,并且多年来一直在吹牛。 但是,这里有一个女孩尚未进入十几岁,但通过在Mensa的智商测试中获得罕见的成绩而使印度和英国感到自豪。 让我们知道更多这个女孩。

    http://dhunt.in/2hXaR

  101. @Agent76

    据说为生活做好了准备

    准备过什么样的生活?
    在以《托拉》或《古兰经》为最高智慧的社会中,这种准备是美好生活的最佳基础。

    在遥远的国家,以正确的方式看待自己的社会发生在我身上。
    在斯里兰卡,我们看到一个乞丐和尚走在高速公路上,令我惊讶的是我们的导游说“这是一种轻松的生活”。
    他们可以进入任何房子并获得食物和床等。

    在中世纪,不太富裕的小贵族的小儿子们通过学习骑马、打猎、打架等为生活做好了准备。

    • 回复: @Agent76
  102. Rich 说:
    @Svigor

    你是美国人吗? 在美国,我们有一个词来形容非常聪明的孩子,“书呆子”。 通常,他们非常聪明,但身体虚弱。 当然,也有少数例外。 作为一个孩子,我测试得很好,可以退出普通班,去和高智商的人一起上课。 他们中的一些人直到今天仍然是我的朋友。 我们中只有两个人是体面的运动员,可以参加任何高中球队,而我几乎是我们团队中唯一一个用我的双手工作的人,因为我的大家庭非常参与机械行业,他们确保我学到了东西。

    这只是我的经验,我不想伤害任何人的感情。

    • 回复: @jilles dykstra
  103. Forbes 说:
    @dearieme

    我从一开始就读过塔勒布的所有书,看过他的许多不同的外表,虽然有些人可能会说,“他不乐意受愚人”,但我认为更好的格言是,“我并不总是是的,但我总是很确定。”

  104. 让我印象深刻的是,几乎没有什么衡量标准是完美的。 尽管如此,有些还是相当不错的。

    你可以像个领主一样喝醉了,仍然可以开车回家; 尽管如此,从统计上讲,让人们在血液酒精浓度为 0.3 的情况下咆哮是个坏主意。

    同样,智商在许多方面与智力有着非常明确的联系。 我无视你发现很多智商七十的人并不明显是愚蠢的; 或者智商为 150 的人不是很敏锐。

    当然,人们关心的不是智商可能不是一个完美的衡量标准; 不同种族的分数不一样,尤其是黑人分数很低。 让他们感到不安的是准确性,而不是不准确性。

    • 回复: @FvS
    , @Wally
  105. Loretta 说:
    @RaceRealist88

    所以心灵是一种副现象,完全依赖于大脑的存在及其独特的形而上学状态……但是大脑的健康和构建质量永远无法决定其附属思想的质量……因此相关的心灵表现可以永远不会有成效地衡量。 ???

  106. @J

    正如其他人指出的那样,理查德·费曼 (Richard Feynman) 做了一个智商测试,他得了 126 分。门萨有多少人被证明与费曼一样富有成效和成功? 这也引出了一个问题——你如何定义“成功的人生?”。 一个是门萨会员的人是亚马逊的员工,会比不是门萨会员的杰夫贝索斯更成功,可以测试说 130 吗? 这是塔勒布论证的主旨。 从这个简单的例子来看,智商不是预测性的。 一个人的成功可能远远超过具有同等智商的人对生活结果的预期平均值。 因为一个人一生可能出现的结果是“肥尾”,人们不得不质疑智商的效用。 智商也不考虑现实世界的条件,比如智商高得多的人可能更擅长某些特定任务,但他们对自己的看法是,你作为雇主愿意支付的任务低于他们对自己的看法,而在智商测试中得分较低的人可能会相当好地完成任务,并且满足于完成任务,从而在一段时间内提供稳定和可预测的输出——从长远来看,这更有价值. 许多企业主都见过这种行为,这就是“资格过高”一词存在的原因。 最后,对于一个正在经历人生的人来说,最好忽略 IQ 的整个概念,因为它无法准确预测您的特定结果,并且不会将先入为主的观念放在您的脑海中,并允许自己尝试和失败,尝试并成功,可以让你有机会超越先入为主的观念。 在这方面,智商的概念是面对人类存在过于复杂而无法减少的天真的理性主义的产物,而通过这种复杂性进行测试的唯一方法是反复试验 - 并通过一些智慧完成(即凸修补、试验、投资、探索)。 生命是路径依赖的。

    • 同意: utu
    • 回复: @utu
    , @Anonymous
    , @DFH
  107. @JLK

    这是惊人的考虑到 双权重的言语 部分和 缺乏空间感 两个组件 反对亚洲人 长处

    这些观点直接取自 Ron Unz 的文章。 你相信他的文章是事实的准确来源吗? 除了他在上述项目上的错误之外,他错过了更重要的一点,即关于 PSAT 的一切都是为了增加亚洲移民的代表性而优化的(这在一定程度上也抑制了犹太人的数量,因为亚洲人集中在大多数与犹太人相同的州) .

    负重口头部分的双重加权于 1997 年结束。

    国家优异奖学金选择测试 [PSAT] 有 3 个同等权重的部分:口头(准智商测试)、数学和“写作技巧”,这是一项源自 TSWE 的多项选择语法测试。 测试中增加了写作以减少性别差距( http://www.fairtest.org/revised-psat-debuts-october )。 它是考试中负荷最少的部分,并且很容易准备。 亚洲人在 TSWE 上的得分超过了白人,在 PSAT 中添加一个类似的部分对他们有帮助。

    该测试确实有一个空间组件:数学部分加载视觉空间能力。 可以测试的数学上限非常低,因此(对于那些有很多额外数学课的人)获得高数学分数的容易程度是将亚洲 PSAT 分布的很大一部分提升到国家优异奖范围的原因. 数学是一门累积的学术科目,通过多年的学习和实践,可以系统地和可预测地提高考试成绩。 数学的 1/3 权重名义上高于 ACT 和高中 GPA,实际上几乎相同。

    不同州的分界线对亚洲人有利,因为在竞争较弱的州,他们在轻松竞争中占主导地位,而分界线高的州则拥有大量的亚洲移民人口中心和完整的测试工业综合体。

    PSAT 是亚洲人更有可能准备的测试,而白人历来将其视为 SAT 热身的低风险零风险形式。 粗略地说,白人考试准备和以大学入学为重点的活动在 PSAT 前后升级,而对于亚洲“老虎孩子”来说,这是多年前开始的努力的第一个展示。 犹太人是中间人,但更接近类似白人的准备模式。 无论如何都会有很高的亚洲代表性,但是准备工作和 PSAT 的结构相结合,将其推向了高位。

    • 回复: @JLK
    , @Ron Unz
  108. stretch23 说:
    @APilgrim

    塔勒布是马龙派基督徒(或者至少这是他的背景。

    • 回复: @Antiwar7
  109. utu 说:
    @Johnny Horton

    最后,对于一个正在经历人生的人来说,最好忽略 IQ 的整个概念,因为它无法准确预测您的特定结果,并且不会将先入为主的观念放入您的脑海中,并允许自己尝试和失败,尝试并成功,可以让你有机会超越先入为主的观念。 在这方面,智商的概念是天真的理性主义在人类生存面前过于复杂而无法减少的产物。

    确切地。 不过有一件事:天真的理性主义者没有必要那么天真。 他们了解后果,他们实际上可能想将“先入为主的观念”置于不同民族的头脑中。

    • 回复: @Gssere
  110. @Intelligent Dasein

    实际上,真正的实验将是立即将具有当前教育水平的第一世界个人运送到非洲环境中,让他们从非洲的贫困和基础设施水平开始,然后看看未来的生存率/增长率是多少如果生存率/增长率是可怕的(我怀疑它会是),那么你有很多选择,要么将 IQ 定义为不能真正预测智力,要么,高 IQ 可能与低存活率/增长率或社会相关为了生存和成长,智商超过 150 的人的数量必须低于一定百分比,以纠正缺乏实用的启发法以在特定环境中生存/成功之类的问题——在这种情况下,智力很可能成为社会应该具备的品质不偏向于无限,以确保长期生存。

    正如塔勒布所指出的,人们还可以看看罗马帝国之前的北欧。 为什么几千年来阿尔卑斯山以北没有优越的文明/文化,然后在 1600 年之后突然间,北欧变得占主导地位。 1600 年之前的地中海人可能很容易争辩说,日耳曼人根本不聪明,只是适应了寒冷的气候才能生存——这基本上就是你现在提出的论点。 然而现在谁拥有欧洲最大的经济体? 智商不是预测性的。 生命是路径依赖的。

  111. HallParvey 说:
    @charles w abbott

    “有些领域没有专家”是一个具有挑衅性的假设,而且很可能是真的——尤其是在选股等情况下。 你在上面提到的他最新的论战成果可能不是他最好的作品。 谢谢收听。

    选股是预测未来。 最好留给专家。 有内幕的人。 像玛莎斯图尔特。

    您可以通过他们选择的成功来识别那些属于内部人员群体的成员。 然后是经纪人,他们以玩家下注的方式赚钱。 真正的专家。 他们不是赌徒。

  112. Gssere 说:
    @utu

    暗示激进的平等主义者几个世纪以来都没有传播他们自己的思想病毒,而且它是完全无害的。

    喜欢这个可笑的表情包

  113. Anonymous[370]• 免责声明 说:
    @Johnny Horton

    一个是门萨会员的人是亚马逊的员工,会比不是门萨会员的杰夫贝索斯更成功,可以测试说 130 吗? 这是塔勒布论证的主旨。 从这个简单的例子来看,智商不是预测性的。

    错误的。 这是预测性的 统计学 几乎所有的智力努力,但它不是 保证. 它在最大资金积累类别中的预测性要低得多,但那是你的红鲱鱼。

    此外,门萨会员资格或多或少需要 132 智商分数,因此您想象中的贝索斯非常接近。 他可能会通过一些运气和毅力到达那里。

    • 回复: @Johnny Horton
  114. @Rich

    在美国,我们有一个词来形容非常聪明的孩子,“书呆子”。

    世界上确实有很多嫉妒。
    在我看来,这里很少有书呆子。
    智商测试只是衡量智商,仅此而已。

    • 回复: @Rich
  115. JLK 说:
    @academic gossip

    感谢您的实质性答复。

    这些观点直接取自 Ron Unz 的文章。 你相信他的文章是事实的准确来源吗?

    我独立得出我的结论。 如果 Unz 提到过与空间组件有关的任何事情,我都错过了。

    除了他在上述项目上的错误之外,他错过了更重要的一点,即关于 PSAT 的一切都是为了增加亚洲移民的代表性而优化的(这在一定程度上也抑制了犹太人的数量,因为亚洲人与犹太人集中在大多数相同的州) .

    仔细观察事实会消除这个论点,我将在下面进一步解释。

    国家优异奖学金选择测试 [PSAT] 有 3 个同等权重的部分:口头(准智商测试)、数学和“写作技巧”,这是一项源自 TSWE 的多项选择语法测试。 测试中增加了写作以减少性别差距( http://www.fairtest.org/revised-psat-debuts-october )。 它是考试中负荷最少的部分,并且很容易准备。

    选择索引并没有太大变化。 维基百科将这三个部分解释为“批判性阅读+数学+写作技巧分数”。 无法绕过三个中两个的高口头权重。

    该测试确实有一个空间组件:数学部分加载视觉空间能力。

    你有引文来支持这个吗? “负载”通常被认为意味着最大化的相关性。 如果 PSAT-M 仍然有一些 80 年代的旧问题,那么它就没有加载到空间中。 它可能具有对数学和空间的混合加载。 这仍然意味着东亚空间强度约为选择指数的 1/6,而语言约为 2/3。

    可以测试的数学上限非常低,因此(对于那些有很多额外数学课的人)获得高数学分数的容易程度是将亚洲 PSAT 分布的很大一部分提升到国家优异奖范围的原因.

    错误的思维。 如果天花板很低,亚洲人实际上在高端被截断,就像犹太人在 SAT-V 上被截断一样。

    数学是一门累积的学术科目,通过多年的学习和实践,可以系统地和可预测地提高考试成绩。

    言语也是如此,尽管大部分内容可能是从课堂外的阅读中收集的。 空间智能在早期受到语言学家的批评,因为据说它是可训练的。 他们基于受试者在练习他们正在测试的相同 3D 练习后变得更好——比如一遍又一遍地给某人相同的 WORDSUM-IQ 集。 现在,像 Jensen 这样的专家普遍认为,最好的 g-loaded 测试几乎可以完全在空间语言平面内制定,而数学在很大程度上是多余的。

    数学的 1/3 权重名义上高于 ACT 和高中 GPA,实际上几乎相同。

    测试的目的是识别在大学及以后表现良好的人。 口头语言对社会的效用被高度高估了。 考虑到今天的孩子发短信更多,阅读更少,其与 g 可能也在下降。

    不同州的分界线对亚洲人有利,因为在竞争较弱的州,他们在轻松竞争中占主导地位,而分界线高的州则拥有大量的亚洲移民人口中心和完整的测试工业综合体。

    我不能辩论这个,因为我不知道。 Unz 认为,在低门槛州的犹太人身上发生了完全相反的效果,他已经完成了跑腿工作。

    PSAT 是亚洲人更有可能准备的测试,而白人历来将其视为 SAT 热身的低风险零风险形式。

    您是否有确凿的数据来支持该声明,还是一种刻板印象? 他们是否为 PSAT 提供 Kaplan 课程? 如果他们这样做,我想富裕的白领父母的孩子更有可能接受他们,跨越种族。 无论如何,备考能提高分数的能力就有限了。

    顺便说一下,我看到你引用了 Fairtest。 几个月前,我简要地检查了该网站,发现他们的许多论点都是不诚实的。 它让我想起了这个网站上伊斯兰民族研究的一些材料,表面上写得很好,但在思想上空洞。 经典高语言,低空间。

    • 同意: Ron Unz
    • 回复: @academic gossip
  116. Gssere 说:
    @Johnny Horton

    除了欧洲人几个世纪以来完全统治了北美和非洲的土著人民。

    从“生存”的角度来看,我很确定有能力完全征服另一个社会(例如,完全消灭它们),但反过来说不是真的是适应性的。 因此,您可能不想将“适应性”作为绝对积极的一面。 过去 400 年至少记录了 IQ > Technology > more IQ > sh**在第三世界,他们对此无能为力。 这可能以西方的灾难告终是一个正在进行的实验。 也许生活在丛林中是林迪和整个时间最稳定的生存模式。 谁知道?

    智商最高的人也往往有最少的孩子,所以我想有一个论点,你可以有足够的智商来满足某个阈值是好的,更多的稍微好一点,但太多是坏的。

    您的评论有趣的是,我们已经知道当您将智商相对较高(阅读:欧洲混合)的人置于非洲条件下时会发生什么。

    https://en.wikipedia.org/wiki/Americo-Liberians

    哦,哇,占主导地位的多数地位,但皮肤黑乎乎的。 一定是暴露于白人的宪法政府形式导致了它。

    也就是说,任何时候有人争辩说 IQ 在个人层面上不相关,你不应该太担心,他们是对的,但他们永远不会承认 IQ 人们想要留下深刻印象的实际点,那就是在群体、集体、文明层面,这是绝对相关的。

    我看到很少(他们自己的智商很高)“智商无关紧要”的人报名参加使用本地工具与新几内亚部落的人进行狩猎和捕鱼比赛,以证明我们所谓的“智力”的随意性。 我的主要怀疑解释是,他们往往是智障,但在攀登西方文明地位等级方面的智力受到限制,所以他们玩了一个附带游戏,他们在等级制度上大便,尽管完全没有能力没有电、没有药品、没有自来水、没有超市、没有节育措施和Netflix。

  117. Ron Unz 说:
    @academic gossip

    这些观点直接取自 Ron Unz 的文章。 你相信他的文章是事实的准确来源吗? 除了他在上述项目上的错误之外,他错过了更重要的一点,即关于 PSAT 的一切都是为了增加亚洲移民的代表性而优化的(这在一定程度上也抑制了犹太人的数量,因为亚洲人与犹太人集中在大多数相同的州) ....g-loaded 语言部分的双重加权于 1997 年结束。

    我看到我们众多狂热的犹太激进主义者中的一个再次出现,一如既往的无能不诚实。

    正如您所注意到的,PSAT 由三个同等权重的部分组成,其中一个是口头的,另一个是写作的。 显然,写作测试与语言能力密切相关,而不是能力的数学或空间子组件。

    即使对于宣传员来说,假装写作测试与语言能力无关也是一种愚蠢的伎俩。 事实上,由于相当一部分亚洲学生来自相对较新的移民背景,甚至可以说写作测试比简单的口头测试更不利于他们。

    剩下的测试部分是数学。 大多数心理测量学家认为能力的三个主要子成分是语言、数学和空间,而后两者根本不是一回事。 通过在没有证据的情况下断言 PSAT 数学子测试实际上是一个空间子测试,你是在假装 PSAT 对亚洲人的不喜欢远没有它明显的那么受欢迎。

    所以就像我说的,PSAT 是两部分语言和一部分数学,缺乏直接的空间部分,因此对犹太人来说是理想的,对亚洲人来说是不利的。

    鉴于精英常春藤大学长期以来一直有严格的亚洲配额,因此如今的亚洲学生努力工作以争夺数量有限的名额也就不足为奇了,甚至非常关注完全由前几代人创建的备考系统犹太活动家,显然是故意在 SAT 上作弊的手段。

    诚然,由于精英常春藤大学现在承认大量完全不合格的犹太人,犹太学生几乎不再努力工作或准备。

    与此同时,你对遭受不公平的亲亚裔歧视的白人外邦申请者的明显同情在你的每一句话中都体现出来。 因此,奇怪的是,您没有注意到几乎完全由顶级犹太行政人员控制的常春藤大学录取犹太学生的比率通常比具有类似能力的白人外邦人高出大约 1,000%。

    • 回复: @Anonymous
    , @JLK
    , @jilles dykstra
    , @JLK
  118. DFH 说:
    @Johnny Horton

    如果您的智商低于 110(至少),那么尝试成为一名律师、医生或教授可能是在浪费您的时间。 知道这一点会很有用。

  119. Anonymous[370]• 免责声明 说:
    @Ron Unz

    学术八卦

    手柄说明了一切。

  120. Maus 说:
    @Poco

    #76 波科
    感谢您分享犁的文章。 作为一名托马斯主义者,我发现神经科学研究证明了心灵的非物质性,这让我感到很欣慰。 一种批评是作者的声明,即物质对象永远不会“关于”某事。 符号学表明并非如此。 多米尼加约翰·波因索 (John Poinsot) 在他的 Tractatus de Signis (1632) 中探讨了物体与理解它的智力之间的关系如何作为具有三元性质的意义能指真正存在。 波因索是他那个时代最重要的阿奎那阐释者,他在符号方面的工作提醒我们这个时代,启蒙运动之前的“黑暗时代”毕竟不是那么黑暗。

  121. JLK 说:
    @Ron Unz

    与此同时,你对遭受不公平的亲亚裔歧视的白人外邦申请者的明显同情在你的每一句话中都体现出来。 因此,奇怪的是,您没有注意到几乎完全由顶级犹太行政人员控制的常春藤大学录取犹太学生的比率通常比具有类似能力的白人外邦人高出大约 1,000%。

    正如我昨天提到的,看起来除了普林斯顿大学(就在 ETS 旁边)之外的大多数常春藤大学都决心在 1995 年 SAT-V 截断之后保持犹太人的代表性基本不变。

    我怀疑普林斯顿大学的人明白,截断只是纠正由于测试的高语言权重和缺乏空间成分而导致的长期犹太人比例过高,这可能都是为了让女性尽可能保持竞争力。 媒体停止对普林斯顿大学衰落的尖叫这一事实让我认为这是在闭门造车的情况下解释的。 很难相信哈佛人不知道这一点。

  122. 塔勒布称自己为流浪者

    是不是打错字了,因为他肯定看起来像 喷火器..

    即使作为一个年轻的骆驼牧民,他也有那种火红的外表。

    • 回复: @Mike Tre
  123. 塔勒布不知情的咆哮都只是一个稻草人论点的变体——也就是说,g (IQ) 必须是一门“伪科学”,除非它可以准确地预测每个特定个体在每项特定努力中的结果。

    没有人一开始就提出这个要求。 尤其是,一般认知能力从未被视为 足够 成功赚大钱的条件(这似乎是塔勒布的主要关注点)。 我的意思是,Who 中最具决定性的遗传学家认为职业道德、个性等也不是重要因素。

    此外,塔勒布无法远程证明具有更好预测能力的任何事情。 他衡量能力的竞争性建议是让人们尝试现实世界的任务,看看他们是怎么做的。 我猜塔勒布希望每个人都能处理一些脑部手术或公司 IPO,然后评估他们在事后的表现。 非常平等,但也非常愚蠢。

    • 回复: @Hail
    , @utu
    , @James Thompson
  124. utu 说:
    @Wizard of Oz

    我举了一个例子,说明 Moray House 测试分数在 11 到 80 岁之间的稳定性。
    https://www.gwern.net/docs/iq/2004-deary.pdf

    两个样本之间的相关性为 r=0.63。 对受限范围的修正产生 r=0.73。 然后针对衰减(可重复性误差)校正此相关性,得到 r≈0.80。 (见第 136 页)。

    所以我们有三个数字,只有一个是纯经验的。 其他两个通过更正得到改进,这些更正取决于人们可能质疑的假设。 无论如何,在这门科学中,您必须记住三个数字 0.63、0.73、0.80,并且您必须准备好捍卫后两个的有效性。

  125. utu 说:
    @Gssere

    尽管没有电、药、自来水、超市、节育和Netflix,完全无法生存

    试着解释你论点的这一部分。 您可以使用尽可能多的智商。

  126. Anonymous[370]• 免责声明 说:
    @Gssere

    伟大的职位。

    智商最高的人也往往生育最少的孩子……

    在现代西方国家,这完全是人为的,并且 (((故意)))。 主要是通过灌输、错误的激励和社会安全网变成了入侵者的温暖孵化器。 在一个有凝聚力的、体面的智商国家,你可以拥有一个强大的网络,而不会产生不良影响,因为女性总是更喜欢 真实 获奖者。

  127. Ron Unz 说:

    实际上,我真的没有太多关注潜在的 Taleb 争议本身,但这里有一个简单的问题……

    Taleb 喜欢获得有利的 MSM 报道,并让人们向他支付大笔咨询费或与他一起投资。 他的所作所为不是对自己有很大好处吗?

    同时,如果他说相反的话,即智商结果非常准确,并告诉我们关于人的重要事情,纽约时报和其他 MSM 媒体不会问他“哦,所以你同意詹姆斯沃森的观点??!!” 这不会让他陷入困境吗?

    毕竟,塔勒布不是诺贝尔奖获得者,通常被评为 XNUMX 世纪下半叶最有影响力的科学家之一。 他只是一个卖书和提供金融服务建议的名人知识分子。 他的整个职业生涯不会很快被毁掉吗?

    不管塔勒布相信什么或不相信什么,他不就是在说强烈符合他个人利益的事情吗?

  128. @Anonymous

    你的评论:

    “错误的。 它对几乎每一项智力活动都具有统计预测性,但不能保证。”

    因此,正如投资组合的风险模型无法预测您的股票何时会下跌一样,风险模型实际上毫无用处。 这就是对冲存在的原因,这就是保险存在的原因,这就是杠铃投资策略存在的原因。 不能盲目信任风险模型,因为不仅不能保证成功,而且可以保证失败会在某个概率点发生。 股票市场是俄罗斯金融轮盘赌的一种形式。

    这是塔勒布论证的核心要点。 就像股票风险模型一样,智商无法预测某个特定的人是成功还是失败——无论是在生活中还是在特定的努力中。 您需要其他经验信号来帮助您进行选择,并从最坏情况的假设开始,然后从那里建立起来以正确对冲您的选择。 这可以追溯到 Taleb 的另一点——不了解概率就不应使用统计数据。 许多使用统计方法的人没有意识到正在玩的游戏是俄罗斯轮盘赌的一种形式。

    智商不能用作尺子。 在最好的情况下,它是一个高通滤波器——这就是美国军方使用它的方式。

    • 回复: @Wizard of Oz
    , @Stan d Mute
  129. Hail 说: • 您的网站
    @Ron Unz

    他的所作所为不是对自己有很大好处吗?

    这就提出了一个问题,他为什么“等”了这么久。

    我没有追踪这件事的起源(似乎起源于圣诞节前一周); 有谁知道是什么引发了他的反智商狂潮?

  130. Stefan Molyneux “反驳” Taleb:

    此致onebornfree

  131. @Gssere

    因此,让我们首先假设智商确实是智力的衡量标准。

    您建议我们如何利用这些信息为我们谋利?

    – 我们是否限制来自平均智商等于或高于我们全国平均水平的地区的移民? 仅针对申请者的智商测试怎么样? 用他们的母语测试智商怎么样?

    – 我们是否将公共教育限制在种族智商平均水平高于特定标准的人群中,以保护公共资金并最大限度地提高财政资源和成果? 或者我们是否提供公共教育到一个我们知道智商不会改变的年龄点,管理智商测试,并且只提供公共资金到那个点?

    – 我们是否对那些得分低于特定智商的人实施节育措施——因为人们认为智商确实与遗传有关? 通过这种方式,我们确保平均智商不断攀升。

    – 所有职业——医生、律师、会计师,是否应该通过单一的、全国认可的智商测试来获得全国排名,并在专业执照上强制规定这些信息,以便我可以选择我能负担得起的最聪明的专业人士?

    – 所有银行家、工程师、技术人员、技术人员都应该在全国范围内接受测试、排序和发布,以便银行、企业和客户购买他们能负担得起的最聪明的劳动力和服务吗?

    我只是把这些扔出去。 我想看看使用智商作为衡量标准对我们有什么真正的好处。

    • 回复: @onebornfree
  132. Hail 说: • 您的网站
    @Hypnotoad666

    一个单一的稻草人论点——即,g (IQ) 必须是一门“伪科学”,除非它可以准确地预测每个特定个体在每项特定努力中的结果。

    没有人一开始就提出这个要求

    这里有很多模仿的可能性。

    史蒂夫赛勒重新发布了一个匿名的“硬拉在很大程度上是一种伪科学的骗局” 讽刺:

    硬拉是一种伪科学的力量测量方法。 […] 在比赛中,硬拉的重量只会升高到腰部高度,然后直接向下返回。 在现实生活中,街头体力劳动者可能需要将重量放在腰部以上的卡车床上。

  133. @Bill

    估计的正好。 这就是我被阻止的原因。

  134. @utu

    我必须承认,这不是一篇简单的论文。

    潜在的一点是智商或多或少衡量了一个 18 岁的皮亚杰最终的能力。

    但后来的研究最终发现,在皮亚杰所描述的 18 岁之后,会出现许多更复杂、更有趣的想法。

    开发了各种评估来评估这一点。

    智商评估并非旨在评估更复杂、更细致的思维,尽管与智商总是存在一些相关性。

    • 回复: @James Thompson
  135. Anonymous[370]• 免责声明 说:

    就像股票风险模型一样,智商无法预测某个特定的人是成功还是失败——无论是在生活中还是在特定的努力中。

    看起来你真的不知道你在做什么。 一种 的 130+ IQ 人在智力方面的努力将永远胜过他们的低智商同龄人。 他们中的一些人会选择一生都在冲浪,但这是理所当然的。 这就是为什么我们使用 统计 永远不要使用一人一篮子所有东西的“风险模型”。 哈哈!

  136. Hail 说: • 您的网站
    @Anonymous

    纳西姆是希腊东正教黎巴嫩少数民族和希腊族。 他是 75% 的希腊人,25% 的亚美尼亚人

    误导:

    安条克希腊基督徒也被称为 Rûm,是来自黎凡特地区的一个讲阿拉伯语的民族宗教东方基督教团体。 他们要么是安条克希腊东正教教会的成员,要么是梅尔基特希腊天主教会的成员,他们在黎凡特 [...] 名称“希腊语”主要是指在礼仪中使用通用希腊语

    塔勒布先生被更恰当地描述为阿拉伯基督徒; 他拥有令人印象深刻的黎巴嫩法语社会显赫祖先的个人血统。

    穆斯林和所有人(以及部分将他们推向基督教黎巴嫩的以色列人)太糟糕了,或者塔勒布今天可能是基督教黎巴嫩的大人物。

  137. @Bill

    用哈贝马斯术语重新表述你的论点甚至可能更清晰一点:在科学话语的背景下应用战略论点是没有用的。 ——

    – 换句话说:从科学论证链(合理论证链——这就是科学论述的全部内容)中挑选一个方面并将其置于日常语境中并因此表明,在这方面没有任何意义感觉。 在这种情况下,这种方法本身毫无意义,但非常有用,尤其是当律师试图说服陪审团时,比方说。

    证明科学论证是对是错的正确方法是遵循处于危险之中的科学规则,看看这门科学是否正确,根据自己的主张和标准。

    除非你想表明——例如,像尼采和海德格尔(以及他们的后继者德里达)那样,科学世界观本身就是错误的。 但这是一项尚未在 twitter 上掌握的努力。

  138. APilgrim 说:
    @Anonymous

    塔利卜这个名字是什么意思? 意义与历史。 在阿拉伯语中的意思是“知识的探索者,学生”。 Abu Talib 是穆罕默德的叔叔,在他的父母和祖父母去世后抚养他长大。

    Nassim 名字的含义 词源:新鲜空气(阿拉伯语)。

    尼古拉斯的名字:来自希腊语 Νικολαος (Nikolaos),意思是“人民的胜利”,来自希腊语 νικη (nike) “胜利”和 λαος (laos) “人民”。

    因此,作者似乎有一个混合的种族名称。

  139. utu 说:
    @Hypnotoad666

    塔勒布不知情的咆哮都只是一个稻草人论点的变体——也就是说,g (IQ) 必须是一门“伪科学”,除非它可以准确地预测每个特定个体在每项特定努力中的结果。

    稻草人确实存在。 您可以在 UR 页面与他们会面。 他们确实相信智商测试分数是衡量人类的决定性指标。 他们将智力与这种一维标量度量混为一谈。 如果您试图争辩说也许我们应该考虑更多维度,他们会与您抗争。 如果您向他们指出所谓的 g 因子在数学上是不可避免的,同时也不是唯一的,因为它取决于用于构建协方差矩阵的测试组,他们会与您作对。 如果指出声称的 IQ 测试分数预测能力的相关性因各种更正而膨胀,他们会与您作对。 他们确实生活在他们自己的具体化信仰的世界中。

    我确实同意这里的其他人的观点,即塔勒布是一个胡说八道的人,他试图用在这种情况下不适用的数学术语来使人们眼花缭乱并让人们闭嘴。 他是个恶霸,而且似乎是一个令人讨厌的人。 或许这只是他创造的一个角色,帮助他营销和销售书籍和他的服务。 他是名人。 但我确实相信他在这个论点中的立场是正确的。

    • 同意: AaronB
    • 回复: @JLK
    , @Mike Tre
  140. 如果你想检测某人在某项任务上的表现,比如高利贷、打网球或随机矩阵理论,让他/她完成这项任务; 我们不需要概率挑战的心理学家对现实世界的功能进行理论考试。

    招聘人员也知道这一点,这就是为什么他们倾向于专注于招聘那些已经有工作并证明他们可以胜任工作的人,

    今天毕业生的问题是,有太多的方法可以伪造它,尤其是如果你是少数族裔的话。

  141. JLK 说:
    @utu

    但我确实相信他在这个论点中的立场是正确的。

    他以错误的方式表达了他的爱。 一旦社会接受智商对高附加值任务极其重要并且在很大程度上可以遗传,那么收入均衡和社会主义的论点就会变得更好。 幸运基因与功绩无关,贫富差距与汗水无关。

    这是精英们对遗传主义怀有敌意的主要原因。

    • 回复: @utu
    , @Wizard of Oz
  142. Rich 说:
    @jilles dykstra

    作为一个经过测试的高智商人,我很高兴地承认我们比其他人更聪明,可能应该掌控世界,但我的经验告诉我,高智商的人通常不太适合追求身体。 也许我错了,我会根据我所看到的,仅此而已。 现在我要告诉你,荷兰人,你们的人民,非常聪明。 我的妻子是荷兰人和德国人,她和她的姐妹们是我认识的最聪明的人之一,所以也许他们祖国的情况有所不同。 在这个曾经存在过的最伟大的国家,这个拯救世界并统治世界的国家,一直以来都对那些被称为书呆子的人略有反感。 我不是说这是对的,我自己就是个书呆子,我只是说事情就是这样。 “书呆子”真的不是一个基于嫉妒的术语,但也许它是美国的东西,很难理解。

    • 回复: @jilles dykstra
    , @onebornfree
  143. niteranger 说:
    @Nobody

    数学不一定等于真理。 理论物理学家一直在写魔法数学,几年后它发生了变化。 艾萨克牛顿是一位伟大的物理学家,但对生物学和自然科学有着真正孩子般的理解。

    塔勒布有很多问题,第一个是现实。 非洲人不能建造狗屎! 如果您的智商为 100,您在很大程度上不能成为科学家。 如果智商不重要,为什么它在他不断吐槽的现实世界中有效? 非洲人被告知不要因为埃博拉而吃丛林肉,他们会做什么……他们吃丛林肉。 南非黑人因为他们美妙的文化和信仰体系而强奸小女孩以摆脱艾滋病,这些人将建立一个充满活力的文明。

    塔勒布是一个属,但他并不知道一切。 他相信他可以将数学模型应用于他所反对的一切事物,但他却将其用于一切事物,因为他认为他的模型是正确的。 我不认为智商是很多人认为的那样,但是,有一些东西,因为如果它在某个范围内,您就有潜力从事某些职业或任务。 如果你没有它……你做不到。

    这并不完美,但塔勒布想把婴儿和洗澡水一起扔出去,为此,他的天才是错的,现实世界与他作对。

  144. Rich 说:
    @HallParvey

    不可以。当某些团体认为这有助于他们的议程时,“非洲裔”鸭子开始流行。 多发生更可能是人类起源的开始。 尽管它在我们以非洲为中心的社会中极不受欢迎。

    • 回复: @Anonymous
    , @Wizard of Oz
  145. 汤普森先生

    我是这个话题的中立观察者。 我没有特别的倾向。

    我所拥有的是对事物和观察的好奇心,因此寻求根据信息来理解和被理解。

    我的语气总是暗示性的,从不肯定。

    我只会尝试调和观察到的 IQ 分数的群体差异。 以下是我的几篇高度假设的文章,着眼于关于 PGS 和组间 IQ 分数差异的最坏情况:
    http://4-manifold.blogspot.com/2017/10/an-attempt-to-reconcile-group.html
    http://4-manifold.blogspot.com/2019/01/an-attempt-to-reconcile-group.html

  146. Wally 说:
    @jilles dykstra

    哦,吉尔斯! 你有史以来最好的金块。

    回想一下,学者 Germar Rudolf 是他追求博士学位的“失败者”。

    因为言论自由的常见敌人因为他的研究*而被吊销了他的博士学位,这科学地证明了“纳粹毒气室”宣传是不可能的。

    * 关于奥斯威辛集中营的化学和技术方面的鲁道夫报告/专家报告
    http://vho.org/GB/Books/trr/index.html

    更多关于鲁道夫: https://inconvenienthistory.com/columnists/1465

    https://www.amazon.com/Hunting-Germar-Rudolf-Essays-Witch-Hunt-ebook/dp/B01N0V1C67

    • 回复: @jilles dykstra
  147. @jilles dykstra

    为什么我要为我提出的调查的目的定义我的速记“生活中的失败”? 但我可能会通过添加“失望的期望”来扩大课程范围。

    学校里受欢迎的高智商孩子如何通过奖学金进入好大学,获得不错但令人失望的学位,谈论潜在的好工作,不真正专注或表现出认真的野心,嫁给有吸引力的配偶并生孩子,未能阻止同事/下属的欺诈,必须在 30 多岁后重新开始职业,离婚,再婚,再次离婚等? 使用“失败”作为速记——但没有包含和排他性的“定义”有什么问题吗?

    • 回复: @jilles dykstra
  148. @Ron Unz

    足够精明的赫尔克里·波洛奖的高级小灰色细胞顺利搅拌。

  149. Anonymous[370]• 免责声明 说:
    @Rich

    如果我错了,请纠正我(我真的不太在意,因为它不能改变当前的 IQ 分布)但非洲起源理论基于这样一个事实,即最古老的人类化石曾经起源于非洲。

    但他们最近没有发现 甚至更老 马其顿/保加利亚和中国的化石? 我确定我在 2018 年看过有关此的文章。

  150. @Chimela Caesar

    我看了你的第二个链接,注意到我们还远远没有了解遗传因果关系的细节。 你对浅肤色韩国人基因组的肤浅理解导致人们期望他们是黑人的例子很有说服力。

    • 回复: @Chimela Caesar
  151. @Rich

    相信智人起源于非洲以外的一个更简单的原因(没有忽略尼安德特人和丹尼索瓦人的部分,实际上,由于他们在非洲的缺席,往往证实了这一点)是非洲内部有更多的遗传多样性比世界其他地方。 这很明显是由于一小群相关人员离开非洲,甚至更小的群体在适应或未能适应变化的环境(可能主要是寒冷)时在瓶颈中生存。

    • 回复: @Rich
  152. Mike Tre [又名“MikeatMikedotMike”] 说:
    @Stan d Mute

    他有点像巴勃罗·埃斯科巴。

    • 回复: @Wally
  153. utu 说:
    @JLK

    抱歉,我无法理解你的逻辑。

    • 回复: @JLK
  154. Mike Tre [又名“MikeatMikedotMike”] 说:
    @Gssere

    “也就是说,任何时候有人争辩说 IQ 与个人水平无关,你不必太担心,他们是对的,但他们永远不会承认 IQ 人们想要留下深刻印象的实际点,这在群体、集体、文明层面,这是绝对相关的。 ”

    我认为这真的触及了智商问题的核心。 说得好。

  155. @Chimela Caesar

    亲爱的奇美拉

    感谢您向我发送您的博客文章。 你对几篇论文发表了有见地的评论,我可以看出你正在努力理解和解释撒哈拉以南非洲智力得分极低的难题。

    其他人也在尝试做同样的事情,主要是因为分数似乎太低而无法依赖。 第一点是 Jensen 在 1980 年在“心理测试中的偏见”中提出的一点,即如果整个群体在现实生活中的表现比测试或考试结果预测的要好,那么该测试或考试就低于-估计他们真正的才能,对他们有偏见。 这导致了一系列研究,试图表明非洲人的实际能力比测试预测的要高。 钱德拉·奇萨拉 (Chandra Chisala) 曾为该案辩护。 我不反对这样的发现会非常有说服力的观点,但我认为它还没有被证明,部分原因是非洲人口非常多,而且所有人口中总会有一些非常聪明的人。 Chisala 和其他人提出的另一个选择是,非洲可能存在认知精英。 我同意这是可能的,应该进行调查,但再一次不要认为到目前为止有证据表明这些团体处于世界最高水平。

    根据新的发现,我目前的观点是,这些水平可能是正确的,并且是遗传和环境影响的混合体,尤其是糟糕的教育系统。 无论是什么原因,分数都很低,目前可以预测经济进展不佳。

    在我的博客中搜索相关评论,但这里有一个部分解决了这个问题:

    https://www.unz.com/jthompson/africa-and-the-cold-beauty-of-maths

    继续写博客。

    • 回复: @anonymoys
  156. Mike Tre [又名“MikeatMikedotMike”] 说:
    @utu

    “稻草人确实存在。 您可以在 UR 页面与他们会面。 他们确实相信智商测试分数是衡量人类的决定性指标。 他们将智力与这种一维标量度量混为一谈。 如果你试图争辩说也许我们应该考虑更多的维度,他们会与你抗争。”

    他们(意思是,泡沫包裹着,超级绝缘的知识认证者)还会告诉你,在 UR,不要相信自己的眼睛,在你生活中的每一天,你都习惯于观察所有不同群体的人。

  157. wayfarer 说:
    @Anon

    感谢您发布此视频。

    Linda Ronstadt 让我想起了我的母亲,她的美丽、她的微笑、她的甜美、她完美无瑕的西班牙语、她与墨西哥的联系及其持久的 Vaquero 文化。 她为中央情报局工作,多语种,具有摄影记忆力,能够准确回忆过去半个多世纪以来看似无限量的细节,并且能够与大多数 Jeopardy 冠军展开激烈竞争.

    我期待着有一天再次和她在一起,在精神的魔法领域。

    来源: https://en.wikipedia.org/wiki/Linda_Ronstadt

  158. @Johnny Horton

    统计预测仍然是预测并且通常很有用。 如果有八个专业的毕业生正在考虑在创业公司工作的研究团队和A公司收购了那些谁得分131,133,135和137,而B公司得到了那些谁拿下123,125,127和129,你会疯体重你对它们的投资同样基于可能的基于研究的结果。 (显然,这不是一个精确的现实世界效用示例。它只是指出了有效的推理类型)。

  159. @JLK

    您的结论合乎逻辑,但是否有类似的经验证据?

  160. JLK 说:
    @utu

    抱歉,我无法理解你的逻辑。

    如果今天大部分的经济战利品都流向了天生具有聪明基因的人,那是否比封建时代财富留给天生富人的时代更公平?

    不管怎样,你生来就很幸运。

    • 同意: Jett Rucker
    • 回复: @Anonymous
    , @Wizard of Oz
    , @utu
  161. Anonymous[370]• 免责声明 说:
    @JLK

    不管怎样,你生来就很幸运。

    巨大的差异。 一个是毫无意义的标题,另一个是原始能力。 这是一个问题 正义 地球上的每一种社会动物(可能还有其他动物)都对它很敏感。

    不幸的是,在今天的腐朽体制中,大部分的经济战利品都流向了(((不值得)))和他们的走狗。 这就是为什么每个理智的人都变得焦躁不安的原因。 系统不自然。

  162. Rich 说:
    @Wizard of Oz

    在希腊发现的人类化石早于在非洲发现的化石。 “走出非洲”理论几乎被这一发现揭穿了,但由于西方新的种族政治,这个故事很少受到关注。 类似于美洲印第安人入侵之前在美洲发现的白种人骨骼。

    • 回复: @Wally
    , @Wizard of Oz
  163. Anonymous [又名“AB1”] 说:

    他给人的印象是一个彻头彻尾的白痴。

    我怀疑他很担心自己没有自己想象的那么聪明。

    • 回复: @Dave Bowman
  164. JLK 说:

    巨大的差异。 一个是毫无意义的标题,另一个是原始能力。 这是一个正义问题,地球上的每一种社会动物(可能还有更多)都对它很敏感。

    你已经习惯这样想了。 听着,我不是在鼓吹马克思主义,只是指出如果群众(他们被编程来思考同样的事情)会更加不安,如果他们真的意识到生活在富裕郊区的人实际上从出生到因为他们在学校更努力地工作,而不是应得的。

    这就像在出生时就被处理了一手永远不会改变的糟糕扑克牌,并且不得不和你的幸运朋友一周又一周地玩它而没有抱怨。

    不幸的是,在今天的腐朽体制中,大部分的经济战利品都流向了(((不值得)))和他们的走狗。 这就是为什么每个理智的人都变得焦躁不安的原因。 系统不自然。

    这是更广泛问题的一部分。 大多数人被引导接受了金钱从华尔街上的愚蠢的人转移到聪明的人的观念。 实际上,它是一种寄生商业,与市场效率或为生产性企业筹集资金几乎没有关系。 这是塔勒布说话的平台。

    明白我现在对精英们对遗传主义的真正担忧是什么意思了吗?

  165. @Wizard of Oz

    也许你的目光太快而没有注意到我在引用别人的话:

    另一篇新论文发现,GWAS 对智商的影响——主要是由欧洲人决定的——对非洲人后裔不起作用。 那总是一种可能性:我已经谈过了。 如果您查看身高等位基因的频率(由欧洲人的 GWAS 确定),您会预测俾格米人非常矮——但它们比这要短得多。 他们有自己的影响身高的私人等位基因,这使他们比你想象的还要矮。 或者,如果您试图通过导致欧洲人皮肤变浅的变异频率来估计韩国人的肤色,您会得出结论,他们像夜晚一样黑——但事实并非如此。 他们的肤色非常浅,但这是由东亚常见的浅肤色等位基因引起的,几乎与欧洲人常见的浅肤色等位基因完全脱节。

    所以你不能用那些 GWAS 命中来判断撒哈拉以南非洲人有多聪明,至少在今天不是。 您可以使用的只是智商测量和成就。 好像我们确定您的身高的唯一方法是使用标尺,而不是 GWAS 预测。

    – Gregory Cochran,在他的博客文章中。

    • 回复: @Wizard of Oz
  166. Anon[424]• 免责声明 说:

    感谢您的评论 。 啊,美国深厚的西班牙裔血统!

    我希望我们都在天堂相见,经过我们在地上的考验。

    ¡ 安代尔!

  167. @Wizard of Oz

    这是一个很好的问题。

    我将失败定义为没有实现自己的目标,我在“精英”大学的经历告诉我,精神疾病和性格/社交怪癖是导致高智商人士失败的两个最大因素。

  168. @Anonymous

    我不是在开玩笑,但我不认为智商是微不足道的。 我希望人们能够更好地表达他们的术语、主张和意义。 我倾向于同意支持智商的一方,但我不知道他们是否真的在回应 Taleb 的批评。

    当塔勒布说智商不能预测时,我认为他不是在谈论人口。 如果 ypu 有两个人,IQ 分别为 2 和 120,并询问哪个人更有可能获得博士学位,那么博士学位获得率告诉我们什么?

  169. @James Thompson

    当我提到抛硬币时,我的意思是我认为塔勒布对可预测性的定义。 如果我们正在考虑招收某人攻读博士学位,而我们只有他的智商分数可以做出决定,那么他的成功有多大的预测力?

    事实上,我认为智商可能是非常具有预测性的,但获得博士学位的基本比率告诉我们,与顶级群体的掷硬币相比,随机的人有 1% 的机会。

    为了获得准确的信息,我们应该比较那些渴望获得博士学位的人的比率。 我很确定顶级队列将接近 100%。

    我仍然认为我关于海军上将和将军的问题也很密切。 如果军队使用智商代理分配其入伍者,为什么不使用智商来选择其领导人? 我认为对这个问题的全面解释将回答 Taleb 的不可预测性批评。

    • 回复: @res
    , @James Thompson
  170. Antiwar7 说:
    @stretch23

    不,他是东正教基督徒。 马龙派教徒是天主教徒。

  171. Wally 说:
    @Colin Wright

    说过:
    “当然,人们关心的不是智商可能不是一个完美的衡量标准; 不同种族的分数不一样,尤其是黑人分数很低。 让他们感到不安的是准确性,而不是不准确性。”

    说得好,科林

  172. Wally 说:
    @Rich

    和:
    新证据表明来自欧洲的石器时代猎人被发现了美国
    https://www.lewrockwell.com/2018/01/no_author/new-evidence-suggests-stone-age-hunters-from-europe-discovered-america/
    “新的考古证据表明,美洲最早是由欧洲石器时代的人发现的—— 在美洲印第安人的西伯利亚祖先踏上新大陆之前 10,000 年设立的区域办事处外,我们在美国也开设了办事处,以便我们为当地客户提供更多的支持。“

  173. @Wally

    与此同时,对所谓的毒气室进行了更多的科学调查,所有的结论都表明它们都没有用于杀人。

    • 回复: @Wally
  174. @Rich

    “书呆子”真的不是一个基于嫉妒的术语,但也许它是美国的东西,很难理解。

    在小学时被昵称为教授,在荷兰语中教授是大学讲师,我经历了嫉妒。
    因为,当我离开小学时,我是唯一一个去体育馆的人,在一次家长会上,学校甚至因为我而受到指责。

    • 回复: @Philip Owen
  175. @Wizard of Oz

    显然,您对成功的定义是共同的美国梦、家庭、房子、收入等。
    Solsjenytsyn 是否成功,是的,在我看来。
    斯大林成功了吗?
    不完全是,以他自己的判断,没有世界规则。
    希特勒成功了吗?
    毫无疑问,他复活了德国。
    他失败是因为他低估了反德力量吗?
    也许。

  176. @Ron Unz

    诚然,由于精英常春藤大学现在承认大量完全不合格的犹太人,犹太学生几乎不再努力工作或准备。

    如果这是真的,会不会是美国军事进步远远落后于中国和俄罗斯的原因?
    有几个迹象表明美国确实远远落后,当然,这些事件是有争议的。
    如果这是真的,美国不会公开承认一架俄罗斯飞机可以禁用一艘最新的美国海军舰艇上的所有电子系统。
    这似乎发生在黑海。

  177. @Gssere

    我看到很少(他们自己的智商很高)“智商无关紧要”的人报名参加使用本地工具与新几内亚部落的人进行狩猎和捕鱼比赛,以证明我们所谓的“智力”的随意性

    这样的测试在科学上是不可能的。
    在这样的测试中,你不能将智力与经验分开。
    当然,人们可能想知道 IQ 测试是否没有类似的偏见。
    我想他们是。
    但对我来说没关系,在西方社会,智商分数是西方社会工作适合性的一个很好的指标。
    任何雇主,包括我在内,都知道这一点。

    • 回复: @Antiwar7
  178. 当我读到他关于概率的想法时,我对他关于风险的一些声明做出了积极的假设,这是非常谨慎的理由,我无法反驳他的数学研究。 也许我被代数愚弄了。 也许我不是唯一的。

    也许你无法反驳他基于数学的论点,但想必全世界有成千上万的人可以。 他们这样做了吗? 如果是这样,我相信你会引用它们。 事实上,是什么阻止了你自己开始这个主题? 我有(64 岁),并且发现这是一个启示。

    我已经四次阅读了塔勒布的前三本书,每次都发现了一些新的和令人兴奋的东西。 是的,他是夸张和侮辱性的,但这与这里的问题有什么关系? 这篇文章部分是热门作品吗?

    也许您对 NNT 对社会科学几乎没有尊重(究竟是什么使它们成为科学?)感到生气,主要是因为它们缺乏波普尔式的可证伪性。 提示命中部分。

    • 回复: @James Thompson
  179. @Johnny Horton

    为什么几千年来阿尔卑斯山以北没有优越的文明/文化,

    是什么让你这么认为?
    凯撒的宣传?
    参观奥地利哈雷的凯尔特博物馆,或阅读有关 Ötzi 的信息。
    对我来说,不存在残酷的警察国家是优越文明的证明。
    对我来说,莱茵河以东的野蛮人比罗马军队文明得多。

  180. Anonymous [又名“双工”] 说:

    “一种适用于左尾而非右尾(智商随着它变得更高而去相关)的衡量标准是有问题的。”

    我猜塔勒布在这里混淆了心理测量质量和相关性; 他吗?

  181. @JLK

    或许才干的比喻同样适用于两者,但也可能不是。

    建立一个系统,让人们有足够的动力去使用他们拥有的任何类型的资本,这是有道理的。 因此,通过法律确保继承权有很好的论据,至少是为了给在世的所有者提供足够的激励,使其富有成效和谨慎。 是否应该同样地争论,应该激励为一个人的问题寻找合适的基因。 当然,女性从不怀疑这种特别是来自大自然的激励。

  182. @Chimela Caesar

    谢谢。 我没有注意到它背后有 Greg Cochran 的可信度。

  183. @JLK

    下面回答你的一些观点。 稍后我会尝试回到其余部分(目前时间紧迫,而 RKU 正在再次管道......)

    PSAT 的所有内容都经过优化,以增加亚洲移民的代表性

    仔细看看事实会打消这个论点,

    尽管您到目前为止发表了评论,但论文看起来不错。

    选择索引并没有太大变化。 维基百科将这三个部分解释为“批判性阅读+数学+写作技巧分数”。 无法绕过三个中两个的高口头权重。

    然而,亚洲人在 SAT 写作(前身为 TSWE)中的得分超过了白人,该考试的 PSAT 写作部分旨在预测其分数,尽管它是“口头的”。

    该测试确实有一个空间组件:数学部分加载视觉空间能力。

    …它可能对数学和空间有混合负载。

    语言和视觉空间上有负载。 我不认为数学考试分数存在“数学”(因子或 PCA 组件),因为存在 g、语言、视觉空间、工作记忆和处理速度。 在大量人群的测试中,跨测试和样本,这不是一个强大的东西。

    可以测试的数学上限非常低,因此(对于那些有很多额外数学课的人)获得高数学分数的容易程度是将亚洲 PSAT 分布的很大一部分提升到国家优异奖范围的原因.

    错误的思维。 如果天花板很低,亚洲人实际上在顶端被截断了,

    不会。数学测试中可能获得的分数 (60) 是固定的。
    如果难度增加,数学考试的平均值和标准差,以及该州的全国优秀半决赛选手选拔截止点,都将下降; 并且通过准备考试的数学部分来获得额外的分数将变得更加困难。 在数学测试被博士资格考试中的 60 道问题取代的极端情况下,您可以很容易地看到这一点。 对选择指数的贡献变为零,因为学生不可能在这部分考试中获得任何分数,而亚洲孩子会突然面临一个实际上没有数学的考试,尽管其数学成分得到了加强。

    无论如何,备考能提高分数的能力就有限了。

    足够长的准备时间(以多种形式)可以做很多事情。

    最公平

    Fairtest 就是这样,但它关于这个问题的网页是权威的。 ETS 于 1997 年对 PSAT 进行了更改,以解决 Fairtest 的投诉。

    • 回复: @res
    , @JLK
    , @keuril
  184. anonymoys 说:
    @James Thompson

    汤普森教授,

    如果你写的是,“......在所有人群中总会有一些非常聪明的人。 Chisala 和其他人提出的另一个选择是,非洲可能存在认知精英。”确实如此,结论似乎是合理的,关于 IQ 的所有辩论不都是“非辩论”吗?

    我的意思是,非洲的问题并不明显,因为他们未能——出于多种原因——无法产生“认知精英”?

    承认如果​​非洲领导人要求中国帮助他们建立认知精英和冷酷的精英制度是不合理的——非洲,不到两代人就可能拥有一两个物理/化学领域的诺贝尔?

    当然,这是一厢情愿的想法,它不会发生,因为不幸的是,那些在非洲掌权的人太愚蠢了,无法理解拥有认知精英的重要性……

    • 回复: @Wizard of Oz
  185. 按平均智商计算的 STEM PHD 人数 = 0。

    高智商的博士数量 >0

  186. @Rich

    丰富 说: “作为一个经过测试的高智商人,我很高兴地承认我们比其他人都聪明,可能应该掌控世界,”

    先生,如果你真的像你错误相信的那样聪明,你就不会在这里幻想你和像你一样的人“可能应该如何管理世界”,仅仅因为你已经了解人类行为和经济的基本不变法则,并且已经得出结论,[任何人]对社会的自上而下的集中规划总是“严重”失败[正如斯科特亚当斯所说],并最终导致[通过战争和饥饿]大量人死亡。

    当然,除非最终结果是您真正想要的。 否则,它简称为“傲慢”🙂

    参见:Murray N. Rothbard 的“人的经济与国家”:
    https://mises.org/library/man-economy-and-state-power-and-market.

    此致onebornfree

    • 回复: @Rich
  187. Antiwar7 说:
    @Ron Unz

    他声称已经通过交易赚了足够的钱,不再需要。 他称之为他的“f*** 是 ** 钱”。

  188. Antiwar7 说:
    @jilles dykstra

    这就是为什么 Taleb 认为它应该被称为其他东西,比如 Salaryman Quotient。

  189. Rich 说:
    @onebornfree

    如果你反对我,那你就是反对你应该崇拜的一个高智商的人。相信我,我的一些老师也惊讶于这个来自皇后区的意大利裔美国孩子的智商有多高,所以我接受了测试和重新测试,但我的得分一直名列前茅。 如果我的天才让您难以理解,我很抱歉,但这就是我们这些高智商与低智商的区别之一。

    我关于我们应该如何管理世界的笑话并不是“傲慢”,这只是一个让你无法理解的笑话。 如果你是 Scott Adams 的粉丝,你会记得他的垃圾人角色,他是世界上最聪明的人。 如果没有,去查查他,也许它会帮助你理解。

    的问候。

    • 回复: @onebornfree
  190. Curle 说:

    我不确定你教什么,但我有一位法学院教授在他职业生涯的暮年告诉我,教员可以并且会可靠地仅通过参考 LSAT 分数来预测班级排名。 这是一所他们使用盲测的学校。

  191. Antiwar7 说:
    @James Thompson

    毫无疑问,塔勒布认为获得博士学位并不是什么了不起的成就。

    顺便说一句,我是 Lubinski 和 Benbow 纵向研究的一部分。

    • 回复: @res
  192. @Rich

    好吧,你是在开玩笑,你不会幻想管理整个世界。 很高兴知道。 知道了。 🙂

    是的,我非常熟悉 Scott Adams 的垃圾人角色。 死忠粉。 我每天阅读呆伯特漫画: 呆伯特网 ,并拥有大约十多本 Dilbert 系列书籍。 我个人最喜欢的呆伯特角色是沃利。

    此致onebornfree

  193. res 说:
    @Chrisnonymous

    如果军队使用智商代理分配其入伍者,为什么不使用智商来选择其领导人?

    因为它确实如此? 此页面包含大量有关美国陆军不同工作所需的 ASVAB 子测试分数的信息: https://www.military.com/join-armed-forces/asvab/asvab-and-army-jobs.html

    其中:
    MOS ARMY JOB TITLE 最低 ASVAB 线分数
    09S 美国陆军军官候选人 GT:110
    其中 GT 是线分数(子测试的组合)可能最像传统的 IQ 测试:
    GT – 一般技术:VE+AR

    从这个 Quora 答案(更多好的信息,包括阈值曾经是 120 的 IQ)对应于大约 108 的 IQ: https://www.quora.com/What-is-the-average-IQ-of-US-Army-Officers
    我认为降低的阈值是出于显而易见的原因。 我想知道来自不同组的军官候选人的平均 GT 分数如何比较。

    也就是说,您的部分观点仍然适用。 IQ 被用作初始筛选,但 AFAIK 不是作为将军级别的选择标准。 为此,我认为最好的屏幕似乎是一个初始阈值,然后是展示性能的要求。 这恰好是美国有多少工作世界(大学充当智商筛选器,好吧,从前无论如何)。 这是塔勒布至少有一个观点的部分(很小的一点,与他的大部分吹嘘相去甚远)。 虽然我怀疑初始筛选在培训费用(例如冲洗)和实践中实际失败的费用方面有很大的不同。

    看到对军队不同级别的平均 GT 分数的研究会很有趣。

  194. res 说:
    @academic gossip

    错误的思维。 如果天花板很低,亚洲人实际上在顶端被截断了,

    不会。数学测试中可能获得的分数 (60) 是固定的。
    如果难度增加,数学考试的平均值和标准差,以及该州的全国优秀半决赛选手选拔截止点,都将下降; 并且通过准备考试的数学部分来获得额外的分数将变得更加困难。

    您足够聪明,可以理解天花板在这里的重要性。 你为什么要谈论这个?

    • 回复: @academic gossip
  195. res 说:
    @Antiwar7

    毫无疑问,塔勒布认为获得博士学位并不是什么了不起的成就。

    我认为塔勒布的部分问题在于他的智商很高,并且认为他认为容易的事情对每个人来说都很容易。 同时也很清楚他的非智商缺陷和相对于智商越来越低的竞争对手的优势。

    顺便说一句,我是 Lubinski 和 Benbow 纵向研究的一部分。

    凉爽的。 你能对它从内部的外观添加任何评论吗? 例如,他们是否让您了解正在发表的研究? 有什么样的长期随访?

    • 回复: @Antiwar7
  196. Agent76 说:
    @jilles dykstra

    这有助于解释我从哪里来的生活。

    19 年 2015 月 XNUMX 日 你不必躲避上帝

  197. utu 说:
    @JLK

    这就是您设想您的财富再分配计划的方式。 先从聪明人那里拿钱,因为聪明是不配的。 我们应该根据智商征收累进税吗? 你会从 Ron Unz 开始吗? 但是从那些愚蠢的人那里拿钱呢? 你会如何证明?

    • 回复: @JLK
  198. JLK 说:
    @academic gossip

    然而,亚洲人在 SAT 写作(前身为 TSWE)中的得分超过了白人,该考试的 PSAT 写作部分旨在预测其分数,尽管它是“口头的”。

    你想在这里反驳什么? 文献并没有说亚洲人语言水平低; 它说他们最强的套装在视觉空间轴上。 由于他们的社会经济地位、父母的教育水平以及他们倾向于生活在该国信息丰富的城市/郊区,我预计第二代或第三代亚洲人在语言方面的表现略胜于白人。

    你正在击倒一个你自己建立的稻草人。

    语言和视觉空间上有负载。

    对空间高度怀疑。 我还在等待引用。

    我不认为数学考试分数存在“数学”(因素或 PCA 组件),因为存在 g、语言、视觉空间、工作记忆和处理速度。 在大量人群的测试中,跨测试和样本,这不是一个强大的东西。

    我不认为这个论点实际上对你的论文有效,但无论如何它是错误的,因为在文献中已经很好地确立了数学智能与语言智能和空间智能一起存在。 Jensen 认为最有效的一般智力测试或 g 可以通过省略数学成分并严格测试语言和空间来构建。 这意味着消除 SAT-M 并用 SAT-Spatial 替换它。 我已经同意他的观点,你的评论基本上承认了在这个问题上的观点。

    当然,按照这些思路构建的新 SAT 会产生巨大的社会和政治影响。 很难找到黑人、白人、亚洲人和犹太人之间空间智力的数据细分,除了几十年前巴克曼的论文给犹太人分配空间智商为 91。如果这是正确的,犹太人可能会从大约 25% 的学校下降像哈佛一样回到大约 1-3%,更接近他们在人口中的比例,亚洲人将是拥有约 30% 代表的人。 我个人怀疑犹太人的平均空间智商那么低,并怀疑下降会更温和,但会很大。 在考虑那些将此类辩论排除在公众视线之外并玷污詹森等研究人员和任何深入研究该主题的人的声誉的人的动机时,我们都应该牢记这一点。 特权往往受到嫉妒的保护。 在正常的信息环境中,高端出版物如 “经济学家” 敢于发表关于该主题的发人深省的文章,虚伪的反驳很快就会被能够看穿它们的观众扼杀。

    不会。数学测试中可能获得的分数 (60) 是固定的。 如果难度增加,数学考试的平均值和标准差,以及该州的全国优秀半决赛选手选拔截止点,都将下降; 并且通过准备考试的数学部分来获得额外的分数将变得更加困难。

    我不知道你是否真的相信这一点,或者你是否在不经意间提供了一个非常典型的例子,说明我们的教育系统是如何因推广缺乏推理技巧的高水平语言而受到损害的。

    你是对的,如果它变得更难,PSAT-M 的平均分数会下降。 然而,最有能力的学生将有更好的机会展示自己的东西,并与普通学生保持距离。 在 PSAT-M 的情况下,最有能力的那部分学生是不成比例的亚洲人。 在 PSAT 的两个口头组成部分中,最有能力的人是不成比例的犹太人。 犹太人和亚洲人都在某种程度上因肥尾的压缩而受到惩罚,但亚洲人受到的惩罚要严重得多,因为没有 PSAT 空间(尽管您的无证断言与此相反),并且因为相比之下,数学部分的权重不足到两个言语成分。 犹太人因 1995 年重新集中口语而受到伤害,但他们从缺乏空间测试和口语的双重加权中获得的好处仍然强烈地偏向于大学录取和 NMSQT 测试,与他们相比,不仅亚洲人,但对非犹太白人。

    一个有用的思维工具是通过将论文带到逻辑极限来测试论文。 如果我们让测试如此简单以至于每个人都有满分,那么没有一个小组有机会取得优异成绩。

    足够长的准备时间(以多种形式)可以做很多事情。

    有大量文献可以量化升力量,但并不多。 我认为预科课程应该被禁止,因为富人应该能够但他们的孩子在 SAT 上甚至 10-20 分是不公平的。 你对龙女等的一些评论似乎表明它们是某种亚洲阴谋。 我在 80 年代从未参加过预科课程,但我记得它们通常被称为 Kaplan 课程,而不是 Wang 课程。

    • 回复: @Ron Unz
    , @res
    , @keuril
  199. @Johnny Horton

    智商不能用作尺子。 在最好的情况下,它是一个高通滤波器

    这是我见过的最好的类比之一。

  200. @Ron Unz

    不管塔勒布相信什么或不相信什么,他不就是在说强烈符合他个人利益的事情吗?

    他不是也快出书了吗? 所以这是免费的预出版宣传。

  201. Curmudgeon 说:
    @Chrisnonymous

    在我 33 年的工作生涯中,我接触过数百名医学博士和博士。 虽然所有人都是各自领域的专家,但有些人在这些领域比其他人更好。 此外,虽然他们可能是各自领域的专家,但让我惊讶的是,他们中有很多人对他们专业领域之外的现实世界知之甚少。 博士尤其如此,他们更有可能夸大自己的重要性。 除此之外,他们几乎和其他人一样。 有些讨人喜欢,有些不讨喜。
    我一直认为智商是相关的而不是因果关系。 我认识许多非常聪明的人,他们的个性阻碍了他们的成功。 同样,我认识许多“普通”的人,他们几乎是在特定技能或知识领域方面的专家。 他们中的一些人拥有专利,这使他们赚到的钱比大多数博士一生中看到的要多得多。

    是的,智商是预测因素,但不是保证因素。

    • 回复: @ThreeCranes
  202. @Ron Unz

    塔勒布没有原创想法,也不是天才。 但他假装是一个相当不错的工作,所以可以理解,很多人被他的智慧外表所愚弄。

  203. JLK 说:
    @utu

    这就是您设想您的财富再分配计划的方式。 先从聪明人那里拿钱,因为聪明是不配的。 我们应该根据智商征收累进税吗? 你会从 Ron Unz 开始吗? 但是从那些愚蠢的人那里拿钱呢? 你会如何证明?

    我最初的观点不是提倡任何政策。 我只是指出,如果人们普遍承认智力在很大程度上是可遗传的,那么财富分配的论点将获得更大的力量,这可能就是在机构认可的情况下,“功绩”一词往往与“才能”混为一谈的原因。信息环境,当它们真的不是一个意思时。

    就政策而言,良好的开端是向认知精英传授谦逊、同情心和社会责任感他们可能是静脉曲张整天站在沃尔玛结账柜台的人,而不是其他人。

    他们中的一些人可能声称有同情心,但当银行以零利率向美联储借款时,纽约时报不会大肆吹嘘掠夺性信用卡利率,或者高盛从真正的投资者和其他交易员那里窃取资金的道德使用高频交易,因为他们有更好的电脑。 我没有看到低俗的电视情景喜剧传播关于购买州彩票或因在月底未全额还清信用卡而陷入债务陷阱是多么愚蠢的信息。

    在聪明地利用不那么聪明的人与实际上将后者引诱到可以被利用的情况之间,这是一个滑坡。 我们的社会伦理已经降到最低的公分母。 它让我想起了反乌托邦的另类未来 回到未来,比夫是市长,田园诗般的小镇到处都是赌场和脱衣舞。

    • 同意: Ron Unz
  204. Ron Unz 说:
    @JLK

    对论点的极好总结……

    多年来,我经常与另类右翼类型和犹太激进主义者争论各种定量的事实问题。 但是,尽管前者经常显得固执、无知或无能,但几乎只有后者才会让人觉得不诚实或完全不诚实。 这个“学术八卦”的角色一直给我留下了鲜明的印象。

    这有点像与傻瓜争论与与骗子争论的区别。 我有时确实对前者发脾气,但对后者更容易发脾气。

    • 回复: @jilles dykstra
  205. keuril 说:
    @academic gossip

    足够长的准备时间(以多种形式)可以做很多事情。

    这是白人大体上无法理解的事情,因为他们真的无法想象亚洲学生为备考付出的努力。 这种努力反映在亚洲家庭愿意为教育做出比白人家庭更大的经济牺牲,结果亚洲人也超越了他们的社会经济权重(例如,送孩子上私立中学的收入门槛)。

  206. dvorak 说:

    Taleb 解释说,他对 IQ 背后的数学表示怀疑的原因是,他可以根据特定假设通过计算机生成相关性,这些相关性看起来就像一些报告的关于智力和学业成绩的发现。

    这就好比说,如果1万只猴子敲打打字机1万年都能写出莎士比亚全集,那么实际上这些猴子确实创造了莎士比亚全集。 牛津人和斯特拉福人,遇见黑猩猩。

  207. @Ron Unz

    和傻子争论没有意义,和聪明的骗子争论是可能的,他们走投无路的时候就明白了。
    聪明的人就沉默了。
    早在 1870 年左右,Brit'n Brai 就以这种方式制定了讨论策略,拼写总是错误的。

  208. @JLK

    我们应该根据智商征收累进税吗?

    有人可能会争辩说,就智商决定劳动收入而言,西欧国家有这样的系统。

  209. @Johnny Horton

    我注意到您的重要问题没有得到答复。

    我在另一个相关线程中提出了相同的总体基本问题,同样没有得到真正的答案 [除非我错过了]。

    在这个讨论中真正隐含的是什么?

    是:谁来决定智商测试的标准和参数,是个人,还是政府“一刀切”的裁决?

    这就是讨论的最终目的吗?

    在我看来,未说明的、隐含的假设是,“我们”需要政府介入,做出他们通常半途而废的“一刀切”的裁决,然后每个人都应该心满意足并停止争论。 .

    还是我只是在想象这个未说明的假设?

    因为如果[政府参与智商测试标准等] 不能 它是关于什么的,为什么有人甚至会关心是否对个人进行了 IQ 测试,除了有关个人,他们可以同意或不同意此类测试,这取决于他们认为最符合自己的利益。

    在这种情况下,在我看来,塔勒布的“惊人”主张只不过是“茶杯中的风暴”。

    然而,正如其他人指出的那样,对任何即将出版的书进行良好的宣传。

    此致onebornfree

  210. @JLK

    如果利用低智商的人如此有利可图,为什么这些银行在 2008 年几乎倒闭。是次级贷款让他们陷入困境。正好相反。 低智商的人是从高智商的高智商人那里转移的数十亿美元财富的接收者。

    • 回复: @JLK
  211. JLK 说:
    @grey enlightenment2

    如果利用低智商的人如此有利可图,为什么这些银行在 2008 年几乎要倒闭。

    他们把它们卖给美国和国外小镇上不太聪明或不太了解的银行家。 他们只被那些仍在筹备中的人抓住了,他们知道过道两边的政治家都被收买和支付,无论发生什么他们都会得救。 最终的道德风险。

    低智商的人是从高智商的高智商人那里转移的数十亿美元财富的接收者。

    这在整个人类历史上都发生过。 伟大的夏延猎人比其他人得到了更多的尊重,成为了酋长并得到了最好的小伙子,但其他人并没有被饿死。 当然,当他们是堂兄弟和朋友而不是陌生人时,更容易接受这一点。

  212. res 说:
    @JLK

    Jensen 认为最有效的一般智力或 g 测试可以通过省略数学部分并严格测试语言和空间来构建。

    你有詹森说这句话的地方的参考吗?

    • 回复: @JLK
  213. @JLK

    所以这也与拉里·萨默斯 (Larry Summers) 在提出数学和物理相关时遇到麻烦的方差问题有关吗?

    • 回复: @JLK
  214. JLK 说:
    @res

    我去看看这是我几周前读到的一段回忆录。

    • 回复: @res
  215. JLK 说:
    @Wizard of Oz

    是的,这就是为什么女性在 1995 年重新定位时没有受到那么多惩罚的原因。 他们的方差(标准偏差)在 15 岁左右,而不是男性 ~XNUMX 岁。

  216. @utu

    是不是头脑简单——一个相对无辜的缺陷——解释了为什么你,在你之前对我的评论中,似乎把你的阅读限制在一半的标题和很少的文章中? 或者你为什么错误地说我抱怨它没有提到 Raven's Matrices? 或者这让您似乎没有注意到它在询问某些测试是否适合衡量“智能”,从而不可避免地增加了引入其他措施的可能性?

  217. Anon[436]• 免责声明 说:
    @Agent76

    年轻时,我参加了合同法等法律科目的补习班,我突然意识到,19 岁的孩子应该先完成其他学位,然后长大一些,因为他们并不太明白为什么人们签订了合同。 有趣的是,班上最聪明的两个年轻人通过提问和贡献展现了自己的品质,后来分别成为上诉法院法官和日薪 15,000 美元的皇后区律师。

    • 回复: @Hypnotoad666
  218. @Rich

    在询问您的消息来源之前,我已经进行了搜索,并且只能得出结论,您一般指的是原始人,而不是具体指智人。 否则,请提供链接。

  219. @anonymoys

    建立一个模型,其中包括接受教育的潜在知识精英,但以一种系统地引导他们在发达国家从事智力要求高的工作的方式会很有趣。 好吧,继续思考,你开始问所有非洲裔诺贝尔奖获得者都在美国还是欧洲。 尽管如此,展示我们通常不允许的新问题的比较现代化模型会很有趣。

  220. Wally 说:
    @jilles dykstra

    事实上,吉尔斯,根本没有德国人使用的杀人毒气室。 时期。
    据称使用的化学药剂 Zyklon-B(活性成分:氰化氢 – HCN)是一种杀虫剂。
    代替了德国人拥有的大量极强效,速效,高效且廉价的气体(如沙林毒气和塔邦气),声称的Zyklon-B的使用确实是荒谬的。
    更多:
    Zyklon B: https://forum.codoh.com/viewtopic.php?f=2&t=10798
    例如:
    更愚蠢的是,据称首先使用了杀虫剂 Zyklon-B。
    德国人拥有大量非常有效、所需数量很少的沙林和塔布神经毒气,如果他们真的想灭绝人,他们可以使用。
    =
    和:
    鲁道夫报告: http://vho.org/GB/Books/trr/

    如果德国人真正从事“消灭犹太人”活动,那么这里可能使用了哪些毒气:
    德国毒气(1914 – 1944): https://codoh.com/library/document/976/

    • 回复: @Wizard of Oz
  221. @res

    天花板在几个方面很重要。 你到底有什么想法? 这些问题包括:

    1. 数学考试贡献了多少分数差异。 如果上限太高或太低,方差就会很低,并且(数学)考试准备中可用的分数也不多。 符合亚洲人利益的是,天花板处于中间最佳位置,在那里推动数学准备变得最有效,同时……

    2. 构建 PSAT 以抑制白人准备(通过不像 SAT 那样将其用作录取选择器,并通过添加第三个测试部分来增加长度和复杂性),但也……

    3. 提供另一个可准备的部分(写作部分),在 NMS 选择边缘附近的亚洲人,无论如何都可以最大限度地提高他们的数学成绩,并且可能也在 SAT 的口头部分尽其所能,可以获得通过准备这个子测试比通过在数学或更多的g-loading口头上多考几分来获得更多的分数。

    所有这些都适用于 PSAT。 我会说它非常适合那些在数学上得分高得多并且比其他人做好准备的子群体的兴趣。

    PSAT 中视觉空间谜题或微积分部​​分的幻想(或让每个人都用韩语或中文参加)很有趣,但与考试的目的背道而驰:预测 SAT 分数、大学成绩/入学,以及大学毕业后的表现(所有使用的基本问题似乎都与典型的高中或大学课程有关)。 完成其中任何一项都需要对语言进行大量加权,而 SAT 和 PSAT 都可能相对于最佳预测指标而言低估了它的权重。 我知道在对包括 SAT M、V 和 W 在内的加利福尼亚大学成绩进行的几次回归分析中,由于某种原因,写作部分完全占据了预测能力,其系数大于 M 和 V 的总和。 对大学成绩最具预测性的国家优异选择指数可能是口头+数学+4xWriting或类似的东西,比现在所做的更倾向于口头测试。

    • 回复: @res
    , @JLK
    , @JLK
  222. @JLK

    1995 年重新定心截断了右侧尾部。 一下子,以前在 SAT-V 上得分 720-800 的每个人都是平等的。 这对犹太人和非犹太人的伤害比对女性和非犹太人的伤害更大,因为 720+ 的范围远远超出了犹太人的数量不成比例,并且没有太多女性受到影响。

    这看起来像是(对犹太人)的二阶或三阶效应,而不是抹去他们在常春藤盟校的代表权。 关心 730 或 770 之间差异的学校在他们的录取决定中还有很多其他信息,这些信息并没有突然消失,特别是与一般能力和语言类型能力有关。

    在这两个阈值下,犹太人的代表性并没有那么大的不同,如果是的话,模糊分数差异的整体效果是对常春藤盟校申请者的排名顺序进行一些有限的局部洗牌,以便一些哈佛犹太人随机前往普林斯顿(或相反),一些普林斯顿犹太人到康奈尔大学等等,在两个方向都有运动,但总体而言是反向人才外流,将犹太人转移到排名较低的学校。 十几所学校关心这些分数差异的犹太人的典型百分比与排名没有很好的相关性,所以有些学校会赢而另一些学校会输,而且看起来不像是精英大学的犹太人突然大规模消失.

    • 回复: @JLK
    , @res
  223. APilgrim 说:

    德克萨斯大学(根据法律)自动录取大多数 (75%) 新生,来自德克萨斯高中 GPA 得分前 6% 的学生。 UT Austin 是最受欢迎的州立大学,招生人数上限为约 50,000 名本科生。 孤星州的其他大学自动录取德克萨斯州高中毕业生 GPA 前 10% 的学生。 无论如何,德克萨斯公立大学的大多数自动录取都具有高中成绩单。

    在 1970 年代,自动录取取自 SAT/ACT 分数的前 10%, OR 德克萨斯州高中 GPA 得分者的前 10%。

    与基于 SAT/ACT 或 GPA 的前 4% 系统自动录取相比,本系统允许更多低智商的黑人、西班牙裔和美洲原住民。

  224. Antiwar7 说:
    @res

    现在大多数情况下,这是我们每 5 年左右通过电子邮件收到的定期调查,他们会跟进以争取全面参与。 他们也向我们发送了一些有关他们研究的信息。 其基本要点是,他们所研究的人往往在生活中都相当成功,并且不会坚持一个尖尖的知识分子被困在角落里,怨恨和被社会忽视的刻板印象。 但是,我确信我没有为他们伸张正义。

    当我还是个孩子的时候,SMPY 由约翰霍普金斯大学的朱利安斯坦利经营,当时我和我的家人收到了许多有助于指导我学习的文章和信息的副本。 (特别是,我加快了学习速度。)有一次,我亲自去了巴尔的摩,作为小组的一员参加了一系列考试(在此之前,我在他们的要求下参加了 SAT)。 我当时遇到了 Benbow 博士,后来有幸与 Stanley 博士有过几次私人会面:他是一位才华横溢、谦逊且真诚的好人,他关心他所研究的孩子。 他在这方面的研究始于试图帮助一个数学天才的孩子。 后来,设立了各种学术项目来帮助这些孩子,包括学术夏令营,我作为学生参加,后来作为讲师参加。

    我和我的父母也是罗伯特·普洛明(Robert Plomin)研究的对象。

    • 回复: @Antiwar7
    , @res
  225. Antiwar7 说:
    @Antiwar7

    我忘了提到斯坦利和本博发表的最著名和最具争议的论文之一,你可能已经知道了:

    http://science.sciencemag.org/content/210/4475/1262

    它基本上表明,在数学能力的顶端,男性往往占主导地位,这似乎是与生俱来的。 我相信 Benbow 博士,一位女性,尤其为这个结论感到悲痛。

    我也相信他们的研究倾向于支持加速而不是丰富,因为为天才学生提供更好的结果。 我自己很幸运,在我的小学和高中都有非常合作的管理机构,对此我永远心存感激。 当时全国许多其他类似的学生肯定不是这种情况。

    • 回复: @res
  226. Che Guava 说:
    @HallParvey

    不错,简短,犀利。

    我不相信“最近走出非洲”的教条。 也确实认为水生/沿海阶段的想法确实适用于许多东亚人、许多北欧人以及其他一些群体。 物证在那里。 皮下脂肪分布,甚至在纤细、流线型的头发(身体和头部)中,水分更多。

    一个有效的假设,并有很多生理支持,但有人会说“哦,我没有任何可能的水生/沿海特征,所以我讨厌这个想法”,其他人也没有接受这个想法,等等!

    一个有效的想法超出了理性讨论的范围……至少多年来是这样。

  227. @Polymath

    我认为有一个更简单的解释,那就是很多投资者不看历史记录,因此被适当的时间段抽样不足所愚弄。 例如,阿根廷自 8 年以来已违约 1816 次,但一些投资者表现得好像这从未发生过。

    至于将智力测试的原始分数推入钟形曲线,这在一定程度上是正确的,但与其他一些选项相比不一定是问题。 例如,人们可能会指定数学中的难度级别以构建一个绝对量表,但是一旦这些问题广为人知,就很难将其作为基准来维护。

    SMPY 非常接近那种绝对标准(在大多数 13 岁的孩子都难以应付的数学考试中,18 岁获得满分)并且没有钟形曲线强迫,并且效果很好。

    • 回复: @Wizard of Oz
  228. res 说:
    @academic gossip

    天花板在几个方面很重要。 你到底有什么想法?

    如果你是积极响应而不只是试图谈论这个(你很擅长那个,太糟糕了我对这种“技能”有负面的尊重)你会从你最初回应的摘录开始 我引用的:

    错误的思维。 如果天花板很低,亚洲人实际上在顶端被截断了,

    你的回答(我也引用过)是:

    不会。数学测试中可能获得的分数 (60) 是固定的。

    第二句话显然是正确的(而且很重要),但忽略了这 60 点的分布对于给定范围内(即这里的最高端)的分辨能力非常重要。 因此,决不能证明前面的否。

    与此最匹配的后续回复是:

    1. 数学考试贡献了多少分数差异。 如果上限太高或太低,方差就会很低,并且(数学)考试准备中可用的分数也不多。 符合亚洲人利益的是,天花板处于中间最佳位置,在那里推动数学准备变得最有效,同时……

    然后你继续断言 PSAT 在那个最佳位置 对于亚洲人.

    基于什么? 你有按种族划分的 PSAT 子测试结果的数据吗? 我所知道的最接近的是 Dorans 2004,我认为它是 1995 的权威参考 SAT 重心: https://www.ets.org/Media/Research/pdf/RR-02-04-Dorans.pdf

    这里重要的(识别 NMSF)不是捕获的总方差。 它是高端的解决能力。 NMSF 正在寻找排名前 1% 的学生,因此这个范围(假设是机会和子测试上限效应的两倍)很重要。 对于任何感兴趣的人,以下是 2016 年各州对 NMSF 资格的了解: https://www.compassprep.com/national-merit-semifinalists-by-state/

    查看 Dorans (18) 的图 21-2004,我们看到 SAT 子测试分布直方图按 50 个点划分为四组(黑人、西班牙裔、亚洲人和白人)。 略高于 4% 的亚洲人和 1% 的白人在 SAT 数学排名中名列前茅(750-800)。 假设正态分布,很明显亚洲人将被较低的数学上限截断更多。

    在任何人说“但是口头上……”之前,亚洲人在口头上的比例实际上也高于白人(但幅度较小)。 尽管我怀疑与犹太人的比较(有没有人看过相关数据?)看起来会有所不同。

    我认为这足以证明在 NMSF 选拔过程中,相对于其他种族,降低数学上限会如何对亚洲人产生不利影响。 你不同意那个具体的断言吗?

    PS 在这一点上,我认为 Ron 是对的。 你像骗子一样争论。 我仍在试图弄清楚您是否像您认为的那样聪明(我敢打赌,您的凭据表明了这一点)。 我目前的结论是你很聪明,但认为我们其他人都很愚蠢。 后者实际上是前者的负面证据。 我用来判断智力的主要标志之一是从他人身上看到它的能力。

    PPS 以下是最近关于 PSAT 上限(2017 年末)的一些讨论,该上限声称现在较低: http://giftedissues.davidsongifted.org/BB/ubbthreads.php/topics/240599/Re_Ceiling_Effects_of_new_PSAT.html
    天才论坛讨论的一个好处是父母也往往很聪明; )

    有没有人通过子测试和分布获得有关 PSAT 上限的良好数据? 特别是被种族打破了?

    • 同意: Ron Unz, JLK
  229. @Miriam Spence

    错误的。 尝试将 Wechsler 放在搜索栏中。

  230. JLK 说:
    @academic gossip

    这看起来像是(对犹太人)的二阶或三阶效应,而不是抹去他们在常春藤盟校的代表权。

    我从来没有说过这会抹去他们在常春藤盟校的代表权,只是说这会在一定程度上损害他们的分数。 我的主要观点是,如果没有两个语言子部分并添加一个空间部分来代替或补充 SAT-M,情况会发生很大变化。

    哈佛大学 1990 年公布的 SAT 平均成绩为 1390。可以合理地认为,至少前四分位数的 SAT-V 超过 720。

    在这两个阈值下,犹太人的代表性并没有那么大的不同,如果是的话,模糊分数差异的整体效果是对常春藤盟校申请者的排名顺序进行一些有限的局部洗牌,以便一些哈佛犹太人随机前往普林斯顿(或相反),一些普林斯顿犹太人到康奈尔大学等等,在两个方向都有运动,但总体而言是反向人才外流,将犹太人转移到排名较低的学校。

    这无疑是正确的,这就是为什么哈佛的情况引起了人们极大的兴趣。 它位于堆的顶部。

    • 回复: @res
  231. res 说:
    @academic gossip

    这看起来像是(对犹太人)的二阶或三阶效应,而不是抹去他们在常春藤盟校的代表权。

    也许。 我认为重要的一个方面是语言上限变化影响报告的平均 SAT 统计数据的方式。 我认为这可能会降低犹太人的高口头分数(作为一个群体)对整体平均水平的影响,从而伤害他们。

    这可能是你想到的“二阶或三阶效应”,但我不确定它是否容易被忽略。

    我确实认为较低的上限让大学更容易用具有更理想人口特征的低能力学生取代高能力学生,而不会对平均水平造成太大伤害。 尽管报告中位数和 25-75% 范围的变化表明这还不够; )

  232. @Wally

    你让我离开了沃利。 我在谷歌上搜索了 Zyklon B,从中我了解到它是一种非常有效的人的杀手,并且能够被解释为一种熏蒸剂或欺骗剂。 如果最后提到的不是选择它的充分理由,那么还有对用户的安全考虑。 SS 总部的谈话可能是这样的。

    “将军阁下! 它在这里说,虽然齐克隆如此便宜,但这种新材料沙林的制造商可以在 30 秒内杀死一头大象,其价格与齐克隆相同。”

    “啊,Walther,我的孩子,就像我一直认为的 Yid 一样,在那双太蓝的眼睛后面的某个地方! 省钱,无论如何。 你要受到表扬。 但我很遗憾地说,我们派去守卫营地的元首出色的雅利安人都是沃利人,如果我们在营地里没有医生,他们不会知道如何清洗他们的鸡巴,而健康和标准的安全培训只是简单的如果我们让那些沃利斯中的一个人接触到我们的超级气体,这还不够。”

    我应该补充一点,根据你的评论最终也发给我的链接,德国目击者和/或他们的日记说他们目睹了大部分未被选中工作(或医学实验)的人死于毒气死亡,包括佩里·布罗德博士Johannes Paul Kremer 和 Rudolf Hõs​​s 等。 医生关于特殊行动和人们尖叫的日记可能被认为是非常有力的证据。

    • 回复: @Wally
  233. @Chrisnonymous

    抱歉,无法理解为什么您没有看到更聪明的人更有可能完成博士学位。 50 倍的可能性。 这是否意味着,在没有所有其他知识的情况下,一个更聪明的人比一个不那么聪明的人更有可能获得博士学位。 是的。

  234. res 说:
    @Antiwar7

    很有意思。 谢谢! 我已经阅读了大量他们的研究,并认为它具有开创性和重要意义。 你看过 Steve Hsu 关于 SMPY 的博文吗? 正如你所说,用真实数据反驳一些通常对高智商人的刻板印象非常有用。

    如果你不介意我的问题(即如果你不想回答就不需要回复),你在哪个队列中? IIRC 他们从最高的 ~1% 到 ~0.01% 不等。 我怀疑他们会更加小心地跟进后者,因为它们非常罕见。

    您是 Robert Plomin 研究的遗传学方面的一员吗? IIUC 一些研究人员已将其以病例/对照的方式纳入其情报 GWAS。

    • 回复: @Antiwar7
    , @Antiwar7
  235. @Taleb Fanboi

    如果你翻阅我的博客,你会发现我经常批评社会科学,特别是当论文使用小而没有代表性的样本时,不考虑混杂变量。 此外,如果您再次阅读我的文章,您会发现我在各个方面都同意 Taleb 的观点。 我也批评那些试图表明风险是由特定金融产品设计的模型。 长数据集(至少 50 年)足以说明这一点。

    至于“热门”作品,请告诉我我错了哪一点,或者我对 Taleb 先生不公平的地方。 我的观点是,他对智力测试做出了没有根据的断言,在这篇和下一篇文章中,我提供了一些资料来支持这一说法。

  236. res 说:
    @Antiwar7

    谢谢! 我想我设法错过了那篇论文。 对于其他感兴趣的人,这里有一个免费的文本副本: http://beck2.med.harvard.edu/week6/Benbow%20and%20Stanley.pdf

    加速度是一个有趣的问题。 我在智力上早熟,但在身体和情感上更不成熟,这使这个决定变得更加困难。 他们是否考虑过类似的情况考虑加速? 我担心的一个问题是加速使参加高中运动变得更加困难。 虽然基于年龄的青年联赛可能是一个很好的替代品。

    听起来参与他们的研究对你很有帮助。 很高兴听你这样说。 听起来正是斯坦利博士想要完成的。

    • 回复: @Antiwar7
  237. @James Thompson

    您可能想通过参考吉姆·柯林斯 (Jim Collins) 的两本深入研究的著作“经久耐用”和“从优秀到卓越”来检查您对投资者失败的看法,这两本书在我多年前阅读时都给我留下了深刻的印象。 但我现在看到一篇文章,标题是“是否经久耐用?” 这让我想起了汤姆彼得斯和鲍勃沃特曼 1980 年代的书“追求卓越”,这本书让这两位前麦肯锡人参加了全球巡回演讲。 四年前与其中一位共进晚餐时,他和蔼地承认,当我试探性地提到达美航空公司已经破产时,他们精心挑选的很多经过充分研究的公司都做得不太好。

    • 回复: @James Thompson
    , @res
  238. @utu

    如果“他的攻击精神”(无论这意味着什么)是“健全的”——来自一个所谓的高智商、平衡、超脱的“专业人士”的无尽的阴沉、被宠坏、恶毒、背道而驰的广告,肯定是不是——为什么他觉得需要“眼花缭乱”?

    作为(少数民族)白痴,你是(少数民族)白痴的辩护者。 所以你是一个白痴。

  239. res 说:
    @JLK

    哈佛大学 1990 年公布的 SAT 平均成绩为 1390。可以合理地认为,至少前四分位数的 SAT-V 超过 720。

    也许(我假设你的意思是 720 或以上?),但鉴于当时 M/V 难度的不平衡,我不确定这一点。 我的猜测更像是 700,但我会对真实数据非常感兴趣。 鉴于我有 STEM 背景并且比哈佛的典型情况更习惯于更大的 M/V 倾斜,我可能会有偏见和/或错误。

    查看最近的哈佛数据(在我写完以上内容之后),我们看到前四分位数 V 为 790: http://www.prepscholar.com/sat/s/colleges/Harvard-SAT-scores-GPA
    根据这个等价表对应于预重心 720: http://www.greenes.com/html/convert.htm
    所以也许你是对的。 这假设 SAT 分数自 1996 年以来一直稳定,但我对此持怀疑态度。

    以下是来自 Swarthmore(1990 年平均值 1342)的一些历史信息,其中包括 1990 年分测试的四分位数、平均值和中位数: https://web.archive.org/web/20080325012348/http://www.swarthmore.edu/Documents/administration/ir/SAT.pdf

    他们的最高口头四分位数是 700,所以我会说你毕竟可能是对的。

  240. keuril 说:
    @JLK

    有大量文献可以量化升力量,但并不多。

    这些研究可能侧重于传统的 SAT 课程,这是一种浪费。 我怀疑他们是否考虑孩子从五年级开始并坚持五六年(包括暑假和寒假)的课程。 或者让学生记住以前管理的测试中数千个实际问题的答案的课程(这些问题在新测试中循环使用)。

    我认为应该禁止预科课程,因为富人应该能够但他们的孩子在 SAT 上甚至 10-20 分是不公平的。

    嗯,这些课程现在也可以从可汗学院免费获得,而且它们可能也一样好。 我认为这里没有太多的社会经济论据。 无论如何,这是一个转移注意力的问题,考虑到富人使用的更有效的测试技巧是支付医生或心理学家的费用来编写学习障碍诊断,然后建立 IEP 以获得特殊的便利。 双倍时间对ACT很有帮助。 了解排名前 1% 的得分者中有多少百分比具有“学习障碍”会很有趣,但此信息并未公开披露 AFAIK。 我读过一篇奇怪的文章,声称私立学校学生的比例很高。

    • 回复: @res
  241. @Anonymous

    钉在两行。

    我发现这个特定评论线程上的许多条目完全没有必要、不相关且毫无意义。 在我的生活中,我一直注意到,在生活或经历的任何领域,都有很多人似乎喜欢谈论他们实际上知之甚少或一无所知的高科技、复杂、专业的主题。

    而且我完全确定他远没有他想象的那么聪明——而且永远不会。

    恭喜。

    • 回复: @JLK
  242. res 说:
    @keuril

    了解排名前 1% 的得分者中有多少百分比具有“学习障碍”会很有趣,但此信息并未公开披露 AFAIK。

    这是一个有趣的问题。 学习障碍诊断似乎确实是一个可以利用系统进行游戏的领域。

    以下是 2005 年 ETS 关于延长时间对 SAT® 考试成绩的影响的报告: https://files.eric.ed.gov/fulltext/ED563027.pdf

    他们的三个主要发现是:

    • 能力较差的应试者从额外的时间中获得很少或没有任何好处。 如果学生没有知识或技能,再多的额外时间也不会提高成绩。
    • 分节符似乎可以帮助不同能力水平的考生,无论他们的残疾状况如何。
    • 额外的时间帮助有残疾和没有残疾的中高能力考生。 然而,额外的时间无济于事,实际上可能会阻碍低能力的残疾学生。

    图 15 和 16 清楚地显示了在测试中的后面项目的性能额外时间的好处。

    这是一篇讨论延长时间的文章: https://edsurgeindependent.com/a-case-against-extended-test-time-71aa4a82148d
    他们提倡不定时测试。

    它包括与您的最后一句话相匹配的引用:

    根据大学理事会的数据,在精英高中中,多达 46% 的学生获得了特殊的考试便利,包括额外的时间。 学习障碍的自然比例应该在 2% 左右。

    参考我上面链接的研究,他们还指出:

    在延长时间后,具有平均学术能力且没有学习障碍的学生在 SAT 上的得分大约高出 43 分。

    • 回复: @keuril
    , @Wizard of Oz
  243. keuril 说:
    @res

    有趣的信息,感谢发布。 当然,2018-19 SAT 与 2005 版有很大不同。 SAT 口语以“技巧问题”(即,g-loaded)而闻名,并且正在失去市场份额给更直接的 ACT。 所以几年前,ETS 放弃了 IQ 测试风格的问题,转而采用更类似于 ACT 的问题。 看起来 ACT(或许还有新的 SAT)更注重表现(回答很多不那么棘手的问题,但速度更快),对于那些有当 Junior 还年轻,可以通过 IEP 获得通过的 IEP(据报道你必须得到诊断) *早期的*,比如最好是小学)。

  244. JLK 说:
    @res

    这不是我记得几周前读过的 Jensen 引文,但它自始至终都将语言和空间作为主要的群体因素,有时但并不总是与记忆结合,但从未与数学结合。

    http://arthurjensen.net/wp-content/uploads/2014/06/Psychometric-g-and-Achievement-1993-by-Arthur-Robert-Jensen.pdf

    这篇特别的文章很有趣,因为它表明空间组因素增加了黑人和白人之间的分离,但与流行的看法相反,在纠正 I 后,口头语言不会增加。这可能是我们在 PSAT/SAT 上看到两个口头组件的另一个原因并且没有空间成分。

    顺便说一句,语言和空间是正交的,足以独立进行良好的智力测试的想法至少可以追溯到 Swineford (1948),从那时起在文献中出现了很多次。 Johnson 和 Bouchard 的“VPR”模型(00 年代中期)包括它,并且已经通过一些非常有趣的功能性 MRI 研究进行了测试,这些研究显示了大脑中进行语言和空间思维的位置。

    • 回复: @res
  245. JLK 说:
    @Dave Bowman

    我发现这个特定评论线程上的许多条目完全没有必要、不相关且毫无意义。 在我的生活中,我一直注意到,在生活或经历的任何领域,都有很多人似乎喜欢谈论他们实际上知之甚少或一无所知的高科技、复杂、专业的主题。

    我绝对同意。 一群业余爱好者在一个言论自由的网站上将大学入学考试系统置于显微镜下,使用谷歌和胶带将他们的研究和分析拼凑起来,这是一种荒谬的做法。 哈佛应该自己做研究和出版,而不是掩盖它的踪迹,纽约时报、华尔街日报、华尔街日报和经济学人应该用一些高质量的报告和分析来奖励他们的订阅者,这样这些学校的毕业生就可以参与辩论而不是被完全隐藏在黑暗中。

    • 回复: @Anonymous
  246. res 说:
    @JLK

    有趣的。 谢谢!

    对于其他感兴趣的人,这是该链接第 23/139 页的引述(您很好地解释了):

    空间可视化因子是唯一的非 g 因子,它在黑/白变量上的负载上与 g 相当一致(另见 Naglieri 和 Jensen,1987)。 因此,在 g 和空间因子上负载最高的那些测试中可以看到最大的黑/白平均差异。 最小的黑白平均差异出现在 g 上负载最少和短期记忆因子负载最高的测试中。 与流行的看法相反,口头因素(独立于 I)的平均黑白差异为零。
    对在 121 项研究中进行因素分析的 30 项心理测量测试的检查还表明,各种测试的 g 载荷分布为一个连续变量,其范围很广——从大约 90 到接近 XNUMX。 在同一组测试中,黑/白均值
    差异(以标准偏差单位表示)也作为连续变量分布,范围从接近零到大约 1.3 个标准偏差 (SD)。 根据测试的 g 载荷的平均黑/白差异的线性回归,假设纯 g 测量的估计平均差异约为 1.2 SD。

    下面是更多的是第 156-158 页的扩展引用,我认为它与塔勒布所写的一些内容有关。

    [更多]

    在人员选择测试有效性的审查中经常注意到,预测特定职业工作绩效的有效性系数相当低,大部分在 20 到 30 的范围内。 通常给出的原因是职业能力范围的限制。 但这只是一幅相当复杂的图景中的一个因素,而且可能是一个次要因素。 碰巧的是,g-loaded 测试的预测效度实际上在能力范围更有限的职业中比在能力范围非常广泛的职业中要高一些。 (这种看似矛盾的原因在下面的工作复杂性和 g 有效性中进行了解释。)当我们分析来自数量非常多且种类繁多的职业的人的 g 负载测试分数时,我们发现大约有一半的人
    分数的总方差存在于各种职业的平均值之间,而总方差的大约一半存在于职业中(即,任何给定职业中的人之间的个体差异)。 根据这一经验观察,从统计学上得出,如果
    我们对职业进行排序,以最大限度地提高他们在 g-loaded 测试中的排名与其平均分数之间的相关性,个人的测试分数与其职业排名之间的相关性将是二分之一的平方根,或大约为 +.70 . 换句话说,g 以+.70 的有效性系数预测职业状态。 这种相关程度正是在研究中实际发现的,在这些研究中,许多不同的职业被排名,不是根据任何心理测量标准,而是根据他们的声望(在进行排名的受试者眼中),他们的可取性,人们对他们认为在职业中取得成功所需的智力量的主观判断。 (这三个标准,当基于大量人的汇总排名时,彼此惊人地一致,并且在整个工业化世界和十年间都高度稳定。)似乎无法避免得出这样的结论:什么人通常所说的“职业状态”与心理测量 g 高度相关。
    另一方面,在给定职业中观察到的 g 与熟练程度度量之间的相关性通常远低于 +.70,即使总 g 方差的一半存在于职业中。 这意味着,一般来说,g 预测职业绩效的能力远低于职业水平。 主要原因(除了前面提到的有效性系数的衰减形式)是,一旦员工达到了在给定职业中执行的最低资格水平,许多与 g 无关的其他因素至少变得同样重要g 表示成功的工作表现(或主管和同事对有效性的看法)。 对于给定的职业,大多数名义职业类别在最低水平或阈值之上容纳了令人惊讶的广泛 g。 这个阈值水平可以根据给定职业中的人的平均 g 负载测试分数来估计,这些人的分数处于该职业分数分布的第一个百分位数。 各种职业的 g“阈值水平”的差异远大于各职业 g 的平均水平,而且各职业 g 的最高水平显示出令人惊讶的小变化——只有大约七分之一(以智商单位计) ) 正如我们在阈值级别看到的那样。 在一些非常低g的职业中发现了一些非常高g的人,但在高g职业中没有发现非常低g的人。 (证据由 Jensen, 1980, 343-45 审查。)这个被广泛认可的 g 阈值属性,在教育和职业方面,可能在很大程度上是人们对心理能力测试的焦虑和反感的原因。

    • 回复: @James Thompson
  247. Sparkon 说:

    Is 中文写作甚至 比中国算术难?

    我最喜欢的理论是,学习汉字会给年轻的亚洲学生带来几个显着的优势。 中国和日本的孩子在他们的童年中花费了大量时间学习汉字,这项任务既需要自律,也需要长期、艰苦的学习。 谁会怀疑这些是从小养成的优秀工作习惯,只能为任何学生支付福利?

    英文写作比中国算术更难吗?

    谷歌翻译是准确的,但双关语不起作用,因为用中文制作双关语肯定是 比中国算术难。

    • 回复: @keuril
  248. JLK 说:
    @academic gossip

    我知道在对包括 SAT M、V 和 W 在内的加利福尼亚大学成绩进行的几次回归分析中,由于某种原因,写作部分完全占据了预测能力,其系数大于 M 和 V 的总和。

    几天前,我遇到了一项研究,该研究将写作测试与第一年 GPA 的相关性在 0.6+ 的低范围内,而数学和语言的相关性在 0.5+ 的高范围内。 我不会花时间回去找到它,因为你没有提供你的引文,而且还欠我另一个证明 PSAT-M 有任何明显的空间权重。

    如果不深入研究,我无论如何都不会相信这些相关性。 这些强大的作家中有许多不是必须应对新生微积分的 STEM 学生。

    我对 SAT 写作部分不太了解,但我在 Kaplan 网站上简要查看了有关它的准备建议,并注意到他们强调了正确的标点符号,例如将句号和评论放在正确的位置。 如果他们真的为此评分,那么可能会压缩左侧尾巴的低垂果实,几乎可以肯定是为了提升西班牙裔和黑人的目的,就像压缩语言和数学的上端抑制犹太人和亚洲人的分数一样, 分别。

    对大学成绩最具预测性的国家优异选择指数可能是口头+数学+4xWriting或类似的东西,比现在所做的更倾向于口头测试。

    你的无耻真是令人叹为观止。

  249. Anonymous[171]• 免责声明 说:
    @JLK

    一个言论自由的网站,居然被一群小白把大学入学考试制度放在显微镜下看,真是可笑

    为什么? 为什么是嘲讽? 言论自由是混乱的,但涉足者正在交换信息甚至学习。 在等待纽约时报温暖、熟悉、困倦的手以“适当”的形式为它服务时,请尽量不要装满垃圾。

    • 回复: @APilgrim
  250. Alfred1860 说:
    @Polynices

    对我来说,NNT 在这个话题上的无知仅仅是基于他坚持认为高智商应该或必须与成功相关。 我不认为断言(尤其是在高智商类型中)收入\净资产不是衡量成功的绝对标准是不合理的。 我已经在 99.5 个百分位数中进行了测试,并且我个人对赚取数百万或在我的“职业生涯”中的发展毫不在意。 通过不把我所有的时间和精力都集中在赚钱、攀登公司阶梯和给人们留下深刻印象上,我能够追求我的许多不同的兴趣和爱好。 我怀疑有比智商更重要的解释财务和职业成功的因素。

    • 回复: @Anonymous
    , @Curle
  251. Anonymous[171]• 免责声明 说:
    @Alfred1860

    根据我的经验,赚钱和智商只与中产阶级相关,在这些中产阶级中,学术成就可以带来良好的职业生涯。 如今,大多数 真实 金钱被继承和/或保留给关系密切的个人。 一些聪明人会靠自己创造成功的企业,但在许多情况下,“真”钱的人只会买下他们并吸收创新/成功。

    我还在某处读到过,最聪明的人因不关心财务而臭名昭著。 不确定这是否属实,但他们有“更好的事情要考虑”是有道理的。 特斯拉就是一个很好的例子。

    • 回复: @mikesmith
  252. mikesmith 说:
    @APilgrim

    纳西姆 *尼古拉斯* (提示,提示!)塔勒布是基督徒,不是穆斯林。 即使我在阅读他的两本书时没有学到这一点,这个名字也是一个绝妙的赠品。 穆斯林不会以圣尼古拉斯的名字命名他们的孩子。 然而,我注意到,如果他们的特定群体往往得分不那么好,即使他们自己的得分很高,许多人,无论他们的信仰如何,都会对智商得分变得非常敏感。 参见,例如,Thomas Sowell,一个智商为 160 的非洲裔美国人,他不能接受由于遗传原因黑人得分低于白人。

  253. mikesmith 说:
    @Anonymous

    我认为比我们意识到的更多的人并没有强烈的金钱动机,但是当这些人也非常聪明时,我们往往只会注意到这一事实。 因此,永恒的问题是,“如果你这么聪明,为什么你不富有?” 我们的狩猎采集祖先是游牧民族,无法积累大量财产。 强烈的贪欲一定是后来进化出来的,但它在物种中仍然不普遍。

  254. @Wizard of Oz

    谢谢。 是的,追求卓越是做空股票的好信号。

  255. @res

    有趣的点。 我想对那些希望了解智能的人最好的建议是:阅读 Spearman,然后是 Jensen,然后是 Deary。

    • 回复: @res
  256. res 说:
    @Wizard of Oz

    关于“追求卓越”,这是 2002 年福布斯的一篇赞誉文章,其中包括完整的股票列表及其迄今为止的表现: https://web.archive.org/web/20040217221744/https://www.forbes.com/2002/10/04/1004excellent_print.html

    以及 Tom Peters 2006 年的博客文章: https://tompeters.com/2006/11/in-search-of-excellence-at-almost-25-and-standing-tall/

    但是这个 2017 年的后续行动最直接地说明了你的观点: https://www.marketwatch.com/story/great-companies-are-more-likely-to-do-really-badly-over-time-than-really-well-2017-07-12

    他们的结论:

    伟大的公司更可能做得很差,而不是做得很好。
    只需要几家伟大的公司,投资组合就可以跑赢大盘。
    伟大并不能保证生存。

    链接到您提到的 2004 年文章: https://www.fastcompany.com/50992/was-built-last-built-last

  257. APilgrim 说:
    @Anonymous

    纳税人基金,或 DEFUND 教育,因为我们最近对 MSM 和当地报纸进行了 DEFUNDED。

    EFFING 处理它。

    还是不要。

  258. @res

    谢谢。 引人入胜的阅读。 伯克希尔哈撒韦似乎是一个更好的选择。

    • 回复: @res
  259. keuril 说:
    @Sparkon

    我最喜欢的理论是,学习汉字会给年轻的亚洲学生带来几个显着的优势。 中国和日本的孩子在他们的童年里花了很多时间学习汉字

    但是韩国孩子并没有花太多时间在这上面(现在的韩国语 99%+ 都是用韩语拼音写的,汉字的重要性与英语中的罗马数字一样重要),而且他们做得还可以。 韩国人甚至可能会争辩说他们比日本人做得更好,因为他们有更多的时间学习 STEM 科目而不是记住汉字。 中国和韩国移民在美国出生的孩子做得很好。

    • 同意: JLK
    • 回复: @res
    , @Sparkon
  260. res 说:
    @James Thompson

    有趣的。 根据您对此的后续帖子,我猜想一个好的方法是阅读 Ritchie 的所有重要的短书,然后您在 Deary 的评论论文上的帖子以及该论文。 然后在某个时候(也许有细节层次?)Jensen 的 g 因子。

    斯皮尔曼显然做了革命性的工作奠定了大部分基础,但你会推荐他作为起点吗? 具体是哪些作品? (我尝试快速搜索您的博客,但没有看到明显的答案,也许我错过了?)

    • 回复: @James Thompson
  261. res 说:
    @James Thompson

    我的投资遗憾之一是没有购买伯克希尔哈撒韦公司在互联网热潮期间失宠的股票。 但是这张图表让我想知道它未来的赌注有多好: https://www.theatlas.com/charts/S1mAAANEz

    让我们尝试嵌入它以保存点击。

    该图表的一个值得注意的地方是它在修正附近的行为(大约在 1987、1991、2000、2008 年)。 我想知道在市场动荡时期购买它是否有值得学习的教训。

    我想知道沃伦巴菲特和查理芒格还能坚持多久。 有趣的是,巴菲特的年龄现在似乎比 2001 年少

    PS 我认为用 log y 轴重新绘制该图会提供信息。

    • 回复: @James Thompson
  262. res 说:
    @keuril

    也许对于智商进化过程中的选择来说,比文化效应更重要?

    • 回复: @Wizard of Oz
    , @keuril
  263. klokman 说:
    @Anonymous

    实际上,不,他们没有。 作为 80 年代的一名军事承包商雇员,我参加了空军生存训练计划。 我们的工作要求我们访问 Dugway 试验场上的远程雷达站点,在那里,由于天气或军事紧急情况,总是存在被抛在 7-9K 英尺的高风险。 它是专门发起的,因为被击落的飞行员在隔离中无法幸存下来的后果率很高。 他们担心的是失去对飞行员的大量投资,以及缺乏足够聪明的人来做到这一点。 这些死亡或即将死亡的飞行员所做的第一件事就是恐慌,做出非常愚蠢和不合理的决定。 他们身上有救生包。 他们接受了如何使用装备的培训。 但是,在需要社会和技术情报的支持环境之外的突然孤立并没有帮助他们。

    知道如何在驾驶舱内拯救自己的屁股并不一定会延续到知道如何在驾驶舱之外单独进行。 成就一名出色的战斗机飞行员的品质并不一定会延续到野外的独创性、创造力和敏感性中。

    • 回复: @Wizard of Oz
  264. @res

    谢谢您的链接。 我无法像我的一位同学那样成为天才投资者,他的父亲于 1931 年在一座 500 万人口的城市郊区买了一座农场,当这座城市人口达到 5 万时,他最近以 2 亿美元的价格出售了该农场的四分之三。 。 因此,我试图将我的注意到转化为“明智的锯子和现代实例”,即现代有一些几乎自然的特征,可以实现 3% 或 1694% 的实际投资回报率,为此我可以声称其基础上有一点原创性注意到,从 1914 年英格兰银行成立到 4 年,Consols 都返还了这种利息,其支持率为 1%,这是大土地所有者可以期望其租户获得的体面回报率,也是相当标准的住宅第一抵押贷款速度。 之后,与市场择时或同等时间和精力的选股相比,您对税法和税率的了解和调整可能更有可能为您带来适度的 XNUMX% 的表现。

  265. @klokman

    非常有趣,但我认为你应该称赞他说“其他条件相同”。 诚然,这与坚持概率和平均值的关键重要性并不完全相同,因为,当然,cet。 标准杆条件可能只是无关紧要,从不适用。

  266. @res

    “智商”?? 也许其他生产特征的演变会更重要?

    • 回复: @res
  267. keuril 说:
    @res

    我的评论特别直接针对 OP 的“宠物理论”(“学习汉字赋予年轻的亚洲学生几个显着的优势”)。 我不确定在更长的(进化)时间段内可能会产生什么影响。 也许中国科举制度有一些优生效应,但我不知道你会如何梳理出学习表意文字的影响。

    • 回复: @res
  268. @res

    我想那里一定有关于测试虎妈身份的事情🙂

  269. res 说:
    @Wizard of Oz

    也许。 你想到了哪些? 我提到 IQ 是因为它是这里的一个常见话题并且相当可衡量。 也因为识字困难与智商选择压力之间的联系对我来说似乎是合理的。 对其他事情不太确定(例如尽责?)。

    我的主要观点是 keuril 的评论似乎证明汉字对当前良好表现的贡献更多是进化的(遗传,对于任何特征)而不是文化。 我认为这是他评论的隐含要点。

  270. res 说:
    @keuril

    谢谢澄清。 在看到这个之前我写了评论285。 同意中国科举制度。 而且我也不确定如何分离效果。 虽然我的猜测是两者都在起作用。 您知道与文盲率/识字率相关的考试成功/失败率吗? 我的猜测是考试会产生更强但分布不那么广泛的选择效应。

    • 回复: @keuril
  271. Sparkon 说:
    @keuril

    F空中点,但根据维基,韩国人直到 1971 年才停止教授汉字,即使是现在,北方和南方都在某种程度上教授了汉字。 即使机器翻译相当不错,难道一些韩国学者和历史学家不希望了解汉字,以便能够阅读较旧的文件,尤其是那些手工编写的,机器会出现故障的地方吗? 我不知道,但我疯狂的猜测是,如果没有被淘汰,官方汉字已经被弃用,但实际上,可能不会。

    事实上,我的怀疑得到了证实,根据我在 Quora 上找到的这条评论,韩语在韩国仍然用于科学和法律写作:

    然而,韩国社会有两个方面抵制了降低书面语言的趋势,并继续大量使用汉字,直到今天仍然有增无减。 这两个领域是科学研究和司法部门,两者都严重依赖汉字来传达复杂的思想,无论是在韩语科学期刊上发表的研究,还是在起草国家法律法规的司法研究中几乎完全使用大量汉字。

    https://www.quora.com/How-often-is-Hanja-still-used-today-in-South-Korea

    无论如何,首先学习汉字或罗马字符的幼儿都不是在学习 STEM 科目。 相反,他们正在为他们的思考和学习方式奠定基础并开发框架和工具,这些框架和工具将适用于他们长大后所研究的特定领域。

    • 回复: @keuril
  272. @res

    你说得对,我说的太含糊了。 詹姆斯弗林说 20 世纪后期的所有心理测量出版物都是詹森的脚注。 我赞同地报告了这个评论,Deary 对我说所有的心理测量出版物都是 Spearman 的脚注。 Jensen会同意的。

    所以,明确地说,如果现在有人问我在了解心理测量时从哪里开始,我会建议 Stuart Ritchie 的介绍。

  273. @res

    同意它在这张图表上看起来不太好。 我的评论基于他给股东的信的首页,但这是基于账面价值。

    • 回复: @Wizard of Oz
  274. JLK 说:
    @Ron Unz

    所以就像我说的,PSAT 是两部分语言和一部分数学,缺乏直接的空间部分,因此对犹太人来说是理想的,对亚洲人来说是不利的。

    这对美洲原住民也是不利的,因为他们在很大程度上共享东亚情报资料。

    https://velesova-sloboda.info/antrop/lynn-race-differences-in-intelligence.html#_12

    21 项研究的中位数是 86 的智商。在对这两种能力进行测试的所有六项研究中,美洲原住民获得了比语言能力更高的可视化能力。 这些研究中可视化智商的中位数为 89.5,这些研究中语言智商的中位数为 81。 东亚人(见第 10 章第 1 节)也存在同样强烈的可视化能力——弱语言能力,美洲原住民是他们的基因密切相关。

    Lombardi (1970) 的一项研究将 80 名美洲原住民与 80 名 6-8 岁白人进行了伊利诺伊心理语言能力测试,发现美洲原住民的语言智商为 73,可视化智商为 93。智商被构建为六个可视化能力子测试的总和,其中一个是视觉记忆,美洲原住民的智商为 104。这是美洲原住民得分高于白人的唯一子测试。

    更贴近你的心的是,PSAT/SAT 的偏斜也对墨西哥裔美国人不利,他们主要是欧洲和美洲原住民的混合物:

    第 9 行给出了斯坦福 Binet 4 标准化的结果,显示西班牙裔的非语言推理智商为 99; 该样本在言语推理方面的智商为 93。

  275. JLK 说:
    @academic gossip

    PSAT 中视觉空间谜题或微积分部​​分的幻想(或让每个人都用韩语或中文参加)很有趣,但与考试的目的背道而驰:预测 SAT 分数、大学成绩/入学,以及大学毕业后的表现(所有使用的基本问题似乎都与典型的高中或大学课程有关)。 完成其中任何一项都需要对语言进行大量加权,而 SAT 和 PSAT 都可能相对于最佳预测指标而言低估了它的权重。

    几乎一致认为一般智力因素的文献,最常被称为 g,是学术和职业成功几乎所有方面的最佳单一预测指标。 因此,PSAT/SAT 应该测试与以下方面最密切相关的技能。 g. 在互联网上很难找到这些相关性,这令人惊讶,因为心理测量学家和任何对大学招生公共政策感兴趣的人都对它们非常感兴趣。 我终于从 Richard Haier 博士的书中找到了一些重点 智力神经科学:

    当您认为 PSAT/SAT 口头部分是词汇的混合时(与 g)和推理(类比之类的项目;与 g),空间相关性 g 0.92,你认为已经有语言偏差的 PSAT/SAT 低估了语言作为最佳预测指标的建议显然是荒谬的。

    这张图表明,PSAT/SAT等智力测试中省略空间测试是不合理的。 它本质上也对亚洲人、美洲原住民、墨西哥裔美国人以及可能大量的非犹太白人男性具有歧视性,因为很明显,这些群体在空间导向测试中的表现优于口头测试,甚至是口头推理测试。

    • 回复: @utu
    , @Hypnotoad666
  276. Wally 说:
    @Wizard of Oz

    说过:
    “你让我离开了沃利。 我在谷歌上搜索了 Zyklon B,从中我了解到它是一种非常有效的人的杀手,并且能够被解释为一种熏蒸剂或欺骗剂。”

    [更多]

    哈哈。 尝试阅读真正的科学来改变,而不是犹太复国主义的宣传。
    – 顺便说一句,Tabun & Sarin 已经有很大的商店,就没有必要使用 Zyklon-B,它不便宜,也不安全。 但这是“大屠杀”骗局,事实并不重要。
    – Zyklon-B 需要数小时才能释放其氰化物负载,这并不完全有效或安全。 更不用说它在虚假和不可能的故事中声称的大量使用时具有高度爆炸性的特性。 请注意,据称位于“毒气室”上方的火葬场会散发出强烈的热量,会发现挥发性氰化物非常有吸引力。

    – 现在对所谓的 Zyklon-B 放气过程一笑置之。
    所谓的奥斯威辛杀人毒气过程在这里审查和拆除。
    https://forum.codoh.com/viewtopic.php?f=2&t=11143&p=83723&hilit=model+asmarques#p83723

    和: 关于奥斯威辛集中营的化学和技术方面的鲁道夫报告/专家报告
    http://vho.org/GB/Books/trr/index.html
    – 您引用的“目击者”提到了从未发生过的事件,在科学上是不可能的,当然,使用盟军酷刑是有说服力的。 我注意到您害怕提及他们实际声称的内容。 但这里对你有帮助:
    https://codoh.com/search/?sorting=relevance&q=pery+broad
    https://codoh.com/search/?sorting=relevance&q=Johannes+Kremer
    https://codoh.com/search/?sorting=relevance&q=Rudolf+hoess

    并且完全缺乏犹太人声称仍然存在于已知地点但已不在那里的数百万具人类遗骸。
    在我的评论中查看更多信息:Pravda: Holocaust Denial,作者:Ron Unz: https://www.unz.com/runz/american-pravda-holocaust-denial/

    你是在你的头上,拼命地如此。
    http://www.codoh.com

  277. keuril 说:
    @Sparkon

    韩语有一半的词汇来自汉语,尤其是抽象事物的词汇。 问题是,中文是声调语言,而韩文不是,所以当所有这些词被导入韩文时,声调丢失了,导致大量的同音异义词。 在汉语中,你可能会根据声调来区分“妈”四五种方式,根据声调就可以很明显地看出你是妈妈、麻、骂、马还是问题。 但在韩语中,你只有一堆“ma”。 当然,其中一些你可以改成“ba”或“mu”之类的,但是这些会与原本是“ba”或“mu”的其他词混在一起,所以并不能真正解决歧义问题. 这在对话中不是什么大问题,因为上下文往往更清晰(主题,也许更简单),但在写作中,存在歧义。 所以即使在今天,在一些专门的文本中,他们可能会在一个词​​的第一次使用时包含汉字作为一种消歧。 例如,我刚刚查了一下,“法院”和“法律渊源”的韩文维基条目的韩文发音完全相同법원,根据汉字(法院和法源)的差异在介绍中消除了歧义. 但是汉字不再像在汉语(当然)和日语中那样在文本中经常使用。我认为,即使是最狂热的韩国民族主义者也会承认,与韩国人相比,韩国人对汉字的了解相形见绌。中国人和日本人。

    书面韩文过去不是这样的——他们过去使用汉字和母语(韩文)拼音的混合体,日语仍然如此(感谢上帝!)。 但从 60 年代到 80 年代,汉字在书报等韩国文字中被淘汰。 阅读韩语有点像如果一切都在拼音中阅读英语会是什么样子。 如果他们在一夜之间这样做,就会发生叛乱。 但是如果你多年来逐渐引入变化,慢慢简化文本,慢慢降低传统拼写/字符在教育中的重要性,人们就会习惯它。 这并不是说韩国人在阅读他们自己的语言时有任何问题,这向我表明,在老一辈人死后,语音学在英语中也能很好地工作。

    当然,他们仍然会在学校或放学后在补习班(hagwons)教汉字,但与日本人和中国人成长和成人相比,典型的韩国人接触汉字的程度微不足道。 所以回到你的宠物理论,我不认为接触汉字是韩国人学习成绩优异的原因。

    • 回复: @Sparkon
  278. utu 说:
    @JLK

    理查德·海尔 (Richard Haier) 书中的图表是如何生成的? 他们是否进行了 15 种不同的测试,并将它们应用于许多(大 N 个)受试者,然后通过因子分析获得协方差矩阵,从中获得 g 因子? 他们是否将测试应用于白人、亚洲人、犹太人、黑人和美洲原住民? 图中标注的载荷是否适用于所有种族? 还是 g 对所有种族都有效? 如果你有两个协方差矩阵,一个用于白人,一个用于犹太人,你会从两个矩阵中得到相同的 g 吗?

    • 回复: @JLK
  279. JLK 说:
    @utu

    理查德·海尔 (Richard Haier) 书中的图表是如何生成的?

    我不知道。 我没有整本书。

    图中标注的载荷是否适用于所有种族? 还是 g 对所有种族都有效? 如果你有两个协方差矩阵,一个用于白人,一个用于犹太人,你会从两个矩阵中得到相同的 g 吗?

    我只是想知道和我固定晚餐一样的事情。 也许汤普森先生或其他专家可以分享他对此的看法。

    • 回复: @res
  280. keuril 说:
    @res

    抱歉,我对这些事情一无所知。

  281. Antiwar7 说:
    @res

    谢谢你的关注! 我相信我在 1980 年被包括在 SMPY 中,这会将我排在第 3 组。对不起,我不知道我的能力落在了哪个百分位。我似乎记得我的 SAT 分数是 700-M,650-我 12 岁时是 V,750 岁时(750 年代初)是 14-M、80-V。 我 11 岁时在公立学校接受了某种智商测试。我相信结果是:心理年龄为 20.5,智商 > 160。(这是否有意义?)我相信他们正在努力达到尽可能多地参与其中; 否则,人们可能会认为那些容易做出反应的人会取得不成比例的成功结果,从而扭曲结果。

    是的,我参与了 Plomin 研究的基因方面的工作。 首先,一名护士被派到我家为我抽血。 几年后(当技术进步时),我的父母被要求提供面颊拭子。

    • 回复: @res
  282. res 说:
    @Antiwar7

    感谢您的回复! 队列 3 是 0.01% 的组,您的 SAT 分数与此一致(令人印象深刻!您同时设置了 M 和 V 阈值)。 本文的表 1 给出了队列的简明描述: https://my.vanderbilt.edu/smpy/files/2013/01/DoingPsychScience2006.pdf
    这不会很好地格式化,但可能是可读的。

    队列 | 否 | 确定的年份 | 确定时的学生年龄| 识别标准
    1 2,188 1972–1974 12–13 SAT-M 390 或 SAT-V 370(前 1 名)
    2 778 1976–1979 12 SAT-M 500 或 SAT-V 430(1 名中的前 200 名)
    3 501 1980–1983 12 SAT-M 700 或 SAT-V 630(1 名中的前 10,000 名)
    4 1,130 1992–1997 12–14 在年级水平成绩测试的任何子测试中排名前 3%
    5 714 1992 23–25 注册为研究生课程的一年级或二年级学生
    在一流的工程、数学或科学系
    美国

    我知道有人参加了该队列的 SAT 但没有资格(可能会参加前两个)。 有效样本量(~5e6 = 501 人,每 1 人中有 10,000 人)令人印象深刻。

    IQ > 160 对您的数字有意义。 我不确定心理年龄是如何翻译的。

    知道 Plomin 的研究是如何使用你父母的遗传信息的吗? 我不知道它的那个方面。 他们也考你父母的智商吗?

    关于 SMPY 的一件事,在查看他们的图时请记住,将所有群组分组在一起并按四分位数查看它们会给出非常不具有代表性的前四分位数。 因为队列 3 的阈值比其他队列高得多,所以前四分位数比纯粹的代表性样本要聪明得多。 更具体地说,看看前三个队列,第一个队列是迄今为止最大的,并且将用大约 1%-0.5% 的组填充前两个四分位数。 第三个四分位数将包含大约 1%-2% 的群组 0.5 和 0.25,而最高四分位数将包含来自前两个群组的 0.25% 及以上的人以及 0.01% 的人(501,第一个两个队列在那个级别可能只有~40)。

    • 回复: @Antiwar7
  283. res 说:
    @JLK

    图中标注的载荷是否适用于所有种族? 还是 g 对所有种族都有效? 如果你有两个协方差矩阵,一个用于白人,一个用于犹太人,你会从两个矩阵中得到相同的 g 吗?

    我只是想知道和我固定晚餐一样的事情。 也许汤普森先生或其他专家可以分享他对此的看法。

    我对这个也很感兴趣。 对我来说,分数分布显着不同的组应该具有不同的主成分,这似乎很明显,但数据胜过“显而易见性”。

    最有可能获得好的数据的地方是欧洲/亚洲的比较。 有没有人这样做过?

  284. Antiwar7 说:
    @res

    忘了说:不,我不知道 Steve Hsu 关于这个主题的博客文章。 感谢您向我指出它们!

  285. Antiwar7 说:
    @res

    他们基本上向我们提供了大量关于各种教育选择的信息,他们迄今为止的研究,以及关于个人及其经历的文章。 由我的家人和当地学校官员决定如何处理。 (其中一个选择是一次上一两节高级课程,也许在一个人目前的学校或在另一所学校参观一天的一部分,这可能不会太具有破坏性。)我当地的学区给了我点菜布兰奇,我被分配了一个很棒的(我不确定他的职位是什么,但基本上是一个有天赋的)顾问,他可以帮助解决很多问题。 碰巧我比我的年龄大,看起来更老,所以早点上大学也不错。 (然而,在最初的几年里,当地报纸上的几篇关于我的文章让我感到非常暴露和害羞。)我不太喜欢有组织的运动,尽管在大学里我参加了宿舍足球队。 我的尺寸在那里有所帮助。

    是的,总的来说,他们帮了我很多。 此外,参加并随后在他们介绍给我的一些相关暑期课程中任教(我参加了 TIP 并在 CTY 和 MTS 任教)让我接触到了类似的人,这让我非常愉快并结交了许多良好的友谊。

    • 回复: @res
  286. Anonymous [又名“Arne_K”] 说:
    @Johnny Horton

    Afrikaners 或早期的澳大利亚先驱者不符合资格吗?

  287. @James Thompson

    这个帖子似乎是一个提出问题的好地方,这个问题来自观看我游泳回家时播放的一些电视节目(这里是仲夏)。 它涉及人类从很小的时候就开始成熟,大约在它提到的 6 岁阶段,并提供了一些突触修剪的细节。 这被描述为身体在非常耗能的身体部位节省能量的有效方式的一部分。 这似乎表明,在 6 岁之前不断演奏乐器或其他任何东西会建立一类突触(和神经元???),同时让其他人离开。

    这让我有两个问题
    1. 他们有没有研究不同年龄(包括完全成熟)的差异突触修剪的影响?
    2. 突触修剪可以在多大程度上得到补偿和逆转,在什么年龄? 正如我们至少 30 年来知道大脑中存在干细胞一样,似乎有理由说即使“使用它或失去它”也不是硬道理。

    • 回复: @James Thompson
  288. Sparkon 说:
    @keuril

    但是,与日本人和中国人长大后和成年后的情况相比,一个典型的韩国人对汉字的接触是微不足道的。

    T谢谢你的评论。 我知道 Hanja 已被弃用,但我不确定 Hanja 在韩国的常规使用程度如何,所以我故意在我对我的宠物理论的原始评论中没有引用韩国。 然而,我之前从 Quora 链接的 Hyun Kim 的评论,当完整阅读时,给出了与你所画的完全不同的画面,并证明了 Hanja 目前正在科学和法律界使用的事实,或者你是完全有争议吗?

    无论如何,韩国人对汉字的平均接触和熟悉程度比几乎任何西方人在形成时期接触和了解汉字、汉字或汉字的程度要大得多,也许要大几个数量级,这导致了这样的情况。欢笑是“比中国算术还难”的表达方式。

    我的宠物理论还有另一个方面,它与大脑半球有关,其中左侧被认为更多地参与语言,而右侧更多地参与处理图像。 我知道这可能完全过于简单化,但我将其作为起点或参考点扔在这里。

    虽然严格来说,汉字并非纯粹的图形或象形文字,但我从我自己的日本汉字经验中知道,“读”汉字与读英文是不同的过程,甚至可能是不认识的单词在语音上“发音”,知识渊博的读者可能能够从词干或词根中了解其含义,特别是如果该读者会多种语言并且知道例如拉丁语、法语和/或西班牙语。

    相比之下,对于汉字,要么你知道这个字符,要么你不知道,尽管汉字是由各种通常熟悉的部首(它们本身基于更简单的字符)组成的,而且知识渊博的人可能能够试探一下意思,事实仍然是汉字比英语单词更难解构,并且需要一些相当重要的记忆措施来掌握汉字。

    你提到的大量同音异义词似乎是在韩国保留或恢复汉字的有力论据。 我将在 Quora 引用 Hyun Kim 的另一句话作为结束:

    这是一种极其无知、短视和荒谬的尝试,通过完全删除汉字来提高韩国的识字率,据称它更平等,这是几个世纪以来韩国受过教育和未受过教育的真正区别。 通过试图以这种方式提高韩国的识字率,此类政策无意中增加了韩国的实用文盲率。 全斗焕和他的手下通过贬低语言并使公共教育系统几乎毫无意义,对韩国未来的智力和社会发展潜力造成了无法弥补的伤害。

    • 回复: @keuril
  289. Antiwar7 说:
    @res

    好点:四分位数。 感谢您提供有关队列的信息。 我原以为他们一直使用相同的标准,并且接受新成员的时间更长。 我很幸运在正确的时间出生以适应这个群体。

  290. @Wizard of Oz

    我看到我只提到了一个修剪的相关内容,如下所示:

    https://www.unz.com/jthompson/the-well-tempered-clavichord/?highlight=pruning

    我还没有涉及围绕这个的开发问题,只是看起来像上图所示的成品。

    早期建立并在儿童时期练习的技能似乎有优势。 语言和演奏乐器浮现在脑海中。

    有趣的是,这类问题在 1960 年代就已为人所知,但在神经心理学文献中通常被描述为“保留”。 过度训练迷宫跑等特定技能的老鼠在大脑相关区域消融后失去了所有技能,但重新学习任务的速度比刚刚训练到标准的老鼠快得多,然后消融。 在我的博士论文中,我推测这些技能已经存储在大脑的各个部分,现在被称为分布式网络。

    所有这些大脑重塑的迷人之处,尤其是在儿童早期,孩子们不会在发生时失去记忆,但一旦过程完成,他们往往会失去前四年的记忆。 聪明的。

    • 回复: @Wizard of Oz
  291. Anon[306]• 免责声明 说:
    @RaceRealist88

    智力测试项目不是个人意志的主体,即测试项目是多人参与、协商、相关性测试等的结果,这意味着它们不是任意的。

    智力测试项目不是任意的,因为它们不是个人意志、反复无常或其他方面的决定。

    还有心理物理定律。 一些精神状态显然可以还原为物理可量化参数。 来我的实验室,我给你看。

  292. res 说:
    @Antiwar7

    谢谢! 我对加速问题很感兴趣(尤其是因为它似乎变得不那么受欢迎了?)并且您的帐户很有帮助。 不久前,我与一个考虑让他儿子跳级的人进行了交谈,我的想法归结为(假设智力显然存在):
    – 他在其他方面有多成熟?
    – 他现在的年级有一群好朋友吗? (如果友谊的持续时间很短,那么与未来一年的情况进行比较才是最重要的)
    – 他想参加竞技运动吗? 尤其是在高水平上(研究表明,即使在这一年的出生日期也对以后的结果很重要)。
    如果有人对这个主题有更多实用的建议和/或经验,我会感兴趣。

  293. Pat Boyle 说:
    @dearieme

    亲爱的“亲爱的”——这次你错了。 我参加了很多标准化考试,并且都做得很好。 我和你一样,观察到许多拒绝接受智商和类智商测试有用证据的人在这类测试中表现不佳。 事实上,自从我参与计算机技能测试多年以来。 我已经看到很多来自不赞成 Microsoft 或 Novell 认证测试价值的学生的类似批评。 你几乎总是会发现他们是那些一遍又一遍地参加考试却永远无法通过分数的可怜虫之一。 他们经常以某种身份受雇于数据处理或网络,他们鼓吹“真实”需要“动手”经验。 他们似乎在用手指思考。

    但塔勒布的情况并非如此。 塔勒布非常聪明。 他在智商测试方面大错特错,但不是因为他愚蠢。

    你必须了解他的职业。 与许多人的信念相反,塔勒布并不是一名市场交易员。 他确实在这方面工作了一段时间,但据马尔科姆格拉德威尔说,这是一个悲惨的失败。 Thais 是说——他很悲惨。 于是他转行做古鲁生意。 他在这个角色中茁壮成长。

    上师必须自信。 塔勒布掌握了这一点——他是世上最傲慢的人之一。 他们还需要定期发表声明以宣传他们的特殊性。

    所以塔勒布所做的愚蠢声明只是营销。

    • 同意: Dieter Kief
    • 回复: @keuril
  294. keuril 说:
    @Sparkon

    然而,我之前从 Quora 链接的 Hyun Kim 的评论,当完整阅读时,给出了与你所画的完全不同的画面,并证明了 Hanja 目前正在科学和法律界使用的事实,或者你是完全有争议吗?

    我的意思是,汉字或 Hanja(在韩语中被称为)将在这样的上下文中用于 *消歧*——有点像植物学家或动物学家为了尽可能准确而使用一个物种的拉丁名称而不是它的白话名称的方式。 但这并不意味着动物学家和植物学家实际上会说或读拉丁语。

    从我自己的日语汉字经验中知道,“读”汉字与读英文是不同的过程

    是的,对日本失语症(失去某些语言功能的人,通常是由于中风损伤大脑部分)的研究表明,根据大脑的哪些部分受损,受试者可能保留以下能力,例如,识别汉字(对于那些试图关注此讨论的人,汉字是汉字的日语单词,即韩国人称之为汉字的同一事物)但不发音(例如通过查看树、太阳或水的字符并指向相应的正确照片)并且也无法发音音节(称为平假名和片假名); 或者可能能够发音音节但无法识别任何汉字。 这有力地表明,像汉字这样的表意文字的处理方式与语音书写系统不同。

    相比之下,对于汉字,要么你知道这个字,要么你不知道,虽然汉字是由各种通常熟悉的部首(它们本身基于更简单的字)构成的,而且知识渊博的人可能能够试探一下意思

    许多字符在其中都有一个意义部首和一个声音部首。 通常中文比日文更容易猜,因为激进的发音更直接。 例如,蚂蚁的中文单词, 我可以, 由两个字符蚂蚁组成。 两个字的左边都有虫部虫,右边有声部(马马,义一)。 在日语中,蚂蚁这个词是 白羊座,这两个汉字中的第二个(蚁)用来表示它。 在这种情况下,昆虫部首对日本读者来说同样明显,但是部首义(日语发音为 gi)在弄清楚如何用日语发音时没有用处。 所以用日语阅读汉字实际上比用中文更难——考虑到这些汉字是为中文设计的,并且受日本书写系统的约束(但已经成为日本书写系统的一部分太久了,以至于书写系统的一部分),这也许并不奇怪没有汉字会完全不同,就像如果我们使用拼音,英语写作会非常不同)。

    你提到的大量同音异义词似乎是在韩国保留或恢复汉字的有力论据。

    在这一点上你不会得到我的任何反对,但我认为这不太可能发生。 到 1980 年代,汉字的广泛使用几乎被淘汰了,因此整整一代人都没有使用汉字作为书写系统的重要组成部分。 考虑到韩国经济的表现,他们似乎在没有它的情况下过得很好,即使一些学者和纯粹主义者(或懂日语或中文的人)可能希望事情不是这样。 摆脱汉字对于韩语来说可能比日语更可行,因为韩语的声音系统比日语更丰富,允许汉语借词的语音消歧(韩语和日语都没有声调,所以他们都面临同样的基本问题,但在日语中更糟)。

    这是一种极其无知、短视和荒谬的尝试,通过完全去除汉字,使书面语言据称更加平等,从而提高韩国的识字率。

    这听起来像是一个有斧头的人(我什至说这是一个真正喜欢汉字的人,如果韩国人使用它们会很高兴)。 我认识很多韩国人,我不能说我曾经听过有人以这种方式抱怨他们的书写系统。 虽然改用无汉字书写系统在当时可能被宣传为提高识字率的一种方式,但韩国人实际上为他们的母语文字(韩文)感到非常自豪,事实上他们有一个国定假日(韩文)日)。 我必须说,从语言学的角度来看,这是一个设计精良的剧本——它非常合乎逻辑地结合在一起,令人惊讶的是他们在 15 世纪就想出了它。 韩国在日本和中国的控制下花了很多时间,所以拥有自己独特的剧本(恰好设计得非常非常好)也许是一种民族自豪感的表达。

  295. @JLK

    非常有启发性的图形。 但是底部编号为 1-15 且与 g 的子类别具有单独相关性的框代表什么?

    这些是问题类型类别的相关性吗?

    • 回复: @JLK
  296. keuril 说:
    @Pat Boyle

    你必须了解他的职业。 与许多人的信念相反,塔勒布不是市场交易员

    作为喜剧演员,他被低估了。 在Monty Python中“我向你的大方向放屁”的法国人的这一边,他有最好的粗鲁的schtick。

  297. @Anon

    年轻时我参加过辅导班,其中包括其他法律科目,合同法,让我感到震惊的是,19 岁的年轻人应该先完成其他学位,然后长大一点,因为他们并不太了解为什么人们签订了合同。

    我认为您已经指出了我们整个教育范式的核心问题之一。 要真正学习一些东西,你需要一些上下文来在你的大脑中交叉引用它。 在年轻人拥有基本的核心知识和经验之前,他们根本无法理解所谓的“教育”。

    有生活经验的成年人会从教育中获益更多——但机会成本要高得多,因为他们(通常)是有报酬的工作,并且必须放弃有偿工作才能投入时间上学。

    尽管如此,我们将所有资源用于教育 年轻 人们。 不一定是因为他们需要它或可以有利地吸收它,而是因为他们在其他方面对社会毫无价值,无论如何也无事可做。 (学校的核心使命是让孩子们在白天离开街头,如果他们在此期间学到了一些东西,那就是一种奖励。)

    俗话说“年轻人浪费了年轻人”,但教育可能也是如此。

    • 同意: keuril
    • 回复: @Anon
  298. JLK 说:
    @Hypnotoad666

    非常有启发性的图形。

    这是一支冒烟的枪。 我记得在 80 年代初,当我参加 PSAT/NMSQT 时,我很困惑,为什么他们会加倍加权口头。 那个时候没有互联网。 我能找到的最好的结果是测试手册小字中的一句话,说明语言“更重要”。 我记得在 80 年代读过几篇印刷文章,这些文章明确指出词汇是智力的最佳单一指标。 此图显示这是一个严重的失实陈述。 即使在这个网站上,Wordsum 测试也有很多内容,例如来自 GSS 调查的数据包含非常可疑的信息,至少在某些情况下是这样。 Wordsum 的 0.71 相关性 g 海尔的词汇相关系数为 0.74 是一个很好的迹象 g 不远了。 如果它有十多道题,它可能会接近 0.74。

    崔伯诺? 当然是犹太考生,但我认为这可以追溯到有钱的 WASP 精英的偏见,当时绅士不做 STEM,公共政策由白鞋律师和其他受过经典和教育培训的学者控制。避开数学和硬科学。

    我记得大学里的法律预科生对工程专业的学生嗤之以鼻,他们的态度是,他们都注定要被关在密室里,远离公众视线。 我不认为哈佛有工程学院,但大多数其他常春藤大学和类似的精英私立学校都有。 有一个鲜为人知的事实,至少对于其中一些我看到的数字,工程师不仅综合 SAT 分数高于文科学生,而且 SAT-V 分数甚至更高。

    如果我们要对谁从口头倾斜的 PSAT/SAT 中受益谁受到伤害进行评分,它看起来像这样:

    受益于 PSAT/SAT 语言双权重/无空间的群体

    1. 法律预科/政治类学生
    2.犹太人
    3. 女性
    4. 黑人

    因 PSAT/SAT 上的语言双权重/无空间而受到伤害的群体

    1. STEM类学生
    2.亚洲人
    3.美洲原住民
    3. 墨西哥裔美国人
    4。 男人

    前一段时间有一个帖子说各种“人权组织”监视这个网站,但我不会屏住呼吸等待 ADL 或 SPLC 为他们声称的美洲原住民或墨西哥人拿起火炬这么关心这个问题。

    当影响如此多的人的影响深远的政策决定是在烟雾缭绕的房间里做出的,而没有媒体进行审查时,就会出现问题。 有些团体比其他团体更有可能让他们的人在房间里。 这是一篇博客文章,其中包含一些非常有趣的延时图形,展示了自 1980 年代以来各种人口群体的 SAT 分数如何变化:

    http://theunsilencedscience.blogspot.com/2012/04/racial-amplitudes-of-scholastic.html

    很明显,早在 1995 年重新调整中心和后来增加写作考试之前,他们就在修补 SAT。 将测试政治化是一个坏主意,因为当你试图让一组人生活时,结果其他人会被拖着走。 篡改的测试结果也被用作某些社会政策比实际效果更好的证据。

    也有国家安全方面的影响。 测试分数用于直接或间接招聘和评估有才华的分析师。 可能是直接的,因为他们有一份方便的清单,列出了 NMSQT 每年提供的所有 140 多名高中生的 IQ 学生。 空间智能与能够在数据中形成模式和关联有关,这对于此类工作至关重要。

    但是底部编号为 1-15 且与 g 的子类别具有单独相关性的框代表什么? 这些是问题类型类别的相关性吗?

    我们必须阅读这本书才能找到答案,但它们可能是对同一类别智力的不同测试。 例如,要测试空间智能,您可能需要 3D 旋转、2D 旋转和模式识别。

    海尔的图表显示了最密切相关的主要能力 g 推理、空间能力、记忆力、处理速度和词汇量。 如果您使用此列表来计划最公平的 PSAT/SAT,您将消除记忆(长期记忆过于依赖环境因素;短期记忆无法在纸上测试)和处理速度(计时,而不是心理测量)。 在剩下的三个中,与推理和空间相比,词汇的权重应该大大不足,而不是过高,因为它与推理和空间的相关性较低。 g. 当您考虑到许多推理问题(例如单词类比)本身是词汇密集型的时,可能应该完全放弃单独的词汇测试。

    • 回复: @res
    , @utu
  299. res 说:
    @JLK

    优秀的评论。 我更深入地研究了该图形,并认为总结我在此处找到的内容可能会有所帮助。

    先上图网上的一些讨论: https://psychology.stackexchange.com/questions/19481/how-is-the-g-factor-of-intelligence-calculated

    该图出现在海尔著作的第 25 页上,其中注释:改编自 Deary 等人。 (2010)
    我没有看到书中子测试的更多细节(但看起来并不那么难)。

    按照参考
    Deary, IJ, Penke, L. & Johnson, W. (2010)。 人类智能的神经科学
    差异。 Nature Reviews Neuroscience, 11, 201–211。
    其中有全文可在 http://larspenke.eu/pdfs/Deary_Penke_Johnson_2010_-_Neuroscience_of_intelligence_review.pdf
    您也可以在 https://www.researchgate.net/publication/41420018_The_neuroscience_of_human_intelligence_differences

    我们在图 1 中看到一个类似的数字,但数字略有不同。 我看到的最大差异是非常不同的 g-Memory 相关性和额外的词汇子测试。 该图的 b 组给出了每个子类别的年龄效应(这很有趣)。

    该图的标题表明它基于来自
    Salthouse, TA 在层次结构中定位与年龄相关的个体差异。
    情报 32, 541–561 (2004)
    其中有全文可在 http://faculty.virginia.edu/cogage/publications2/Localizing%20age%20related%20individual%20differences.pdf

    我相信表 1 是 16 个子测试的列表。 我没有看到 Deary 和 Haier 使用的数字的映射,但假设它们在类别中按顺序排列似乎是合理的第一次削减(可能可以通过检查相关性来改进)。 剪切和粘贴格式很糟糕,但在这里作为尝试,它使用类别和子测试名称进行复制。

    表1
    分析中包含的认知变量的描述

    词汇
    WAIS 词汇 – 提供词的定义 Wechsler (1997a)
    图片词汇 - 命名图片对象伍德科克和约翰逊 (1989)
    反义词词汇——选择目标词 Salthouse 的最佳反义词 (1993a, 1993b, 1993c)
    同义词词汇——选择目标词 Salthouse 的最佳同义词 (1993a, 1993b, 1993c)

    推理
    Ravens – 确定哪种模式最能完成矩阵 Raven 中缺失的单元格 (1962)
    Shipley 抽象 – 确定作为序列的最佳延续的单词或数字 Zachary (1986)
    字母集 - 确定五组字母中的哪一组与 Ekstrom 等人的其他字母不同。 (1976)

    空间可视化
    空间关系 - 确定 3-D 图形和替代 2-D 图形之间的对应关系 Bennett、Seashore 和 Wesman (1997)
    纸张折叠 - 确定由一系列折叠和打孔导致的孔图案 Ekstrom 等人。 (1976)
    模板——确定需要哪些形状组合来填充更大的形状 Ekstrom 等人。 (1976)

    情景记忆
    逻辑记忆——三个故事中回忆的想法单元的数量 Wechsler (1997b)
    自由回忆——在单词表 Wechsler (1b) 的第 4 到 1997 次试验中回忆的单词数
    成对的同事——当出现刺激词 Salthouse、Fristoe 和 Rhee (1996a)、Salthouse、Hambrick、Lukas 和 Dell (1996b)、Salthouse、Hancock、Meinz 和 Hambrick (1996c) 时回忆的响应词数

    迅速的
    数字符号 – 使用代码表在每个数字下方写下正确的符号 Wechsler (1997a)
    字母比较 – Salthouse 和 Babcock (1991) 对字母串的相同/不同比较
    模式比较——Salthouse 和 Babcock 线模式对的相同/不同比较 (1991)

    表 3 包含每个子测试的均值、标准差和因子相关性。

    表 5 具有因子-因子相关性。 推理空间是迄今为止最高的,为 0.9

    • 回复: @JLK
  300. Anon[413]• 免责声明 说:
    @Hypnotoad666

    谢谢你的有趣,我认为对这一点的有效推断是轶事。

  301. Curle 说:
    @Alfred1860

    对。 如此多的在经济上取得成功的聪明人最终为了追求爱好而放弃自己的事业,这一事实证明了这些事情的吸引力。 我不禁想到了这样的人的数量,他们让女孩(约翰·迈克菲)、体育和音乐(保罗·艾伦)或艺术和女孩(盖蒂)成为他们的主要兴趣后钱堆。 那么为什么不开始追求这样的事情呢?

    来自 Chris Langan 的 wiki,报告的智商最高的人:

    “兰甘在一段时间内从事了一系列劳动密集型工作,到 40 多岁的时候,他曾是一名建筑工人、牛仔、林务护林员、农场工人,二十多年来,他还在长岛做过保镖。”

    “Langan 也与迪士尼研究公司[11] 接触并签约,他之前曾在一家技术公司 Virtual Logistix 工作。[3]”

    “Langan 说他制定了一种‘双重生活策略’:一方面是一个普通人,做他的工作并互相寒暄,另一方面回家在他的脑海中执行方程式,孤立地研究他的认知理论模型宇宙 (CTMU).[4]”

    换句话说,他安排了他的生活,以便他可以做他想做的事。

    最有趣的是迈克菲,他着手在 Balize 创建一个相当于后宫的“社会工程系统”。 http://www.whoismcafee.com/the-girls/

    • 回复: @DFH
  302. @James Thompson

    你的最后一个标准让理论家们可以为所有那些解释不幸成年人的压抑记忆进行辩论,或者让有人争辩说我们已经失去了0-4岁大脑可能获得的误导性想法和价值观因此,人们应该怀疑声称糟糕的父母或照顾者注定了他们的性格……。

  303. utu 说:
    @JLK

    您已经发布了两次的图表作为评论者“res”的显示是来自几篇独立论文的数据汇编的结果。 没有论文将所有 15 个测试一起比较并提取基于它们的 XNUMX g。 图中的相关性与 g 不同。 g是不同的。 因子 g 取决于电池测试。

    这不是 IQism 领域的作者第一次夸大他们的案例和误导。 来自 Deary 的图表用于灌输 g 的概念是有效的。 作为这种灌输的受害者,你是一个完美的例子。 虽然你还有另一把斧头要磨,我什至可能会支持你的事业,但我怀疑你是否会通过采纳来自超科学领域的论点而有所收获。

    • 回复: @JLK
  304. JLK 说:
    @res

    感谢您提供此信息。 它应该对其他研究人员甚至是记者有所帮助,以防万一有人敢写这样的故事,或者有一个会批准它的编辑。

  305. JLK 说:
    @utu

    您已经发布了两次的图表作为评论者“res”显示的图表是来自几篇独立论文的数据汇编的结果

    正如 res 所指出的和 Haier 在他的书中承认的那样,图表和相关性来自 2010 年的 Dearing 论文,而不是多篇论文。 Dearing 分析了来自同一位研究员 Salthouse 的 33 项研究,测试了 7000 多名 18-95 岁的受试者。 海尔的相关性与迪林的略有不同,但相差不大。 目前还不清楚为什么。 也许他在写书时咨询了迪林,并获得了修改后的相关性。 无论如何,与推理和视觉空间测试相比,词汇测试显然不能很好地代表智力。

    图中的相关性与 g 不同。 g是不同的。 因子 g 取决于电池测试。

    文献几乎一致认为只有一个 g. 如果对一项小型研究进行因子分析,则计算出的 g 可能与实际不同 g,但通常不会相差太多,并且结果往往会收敛于实际 g 随着样本量的增加。 另一条评论提出了是否 gs 可能因种族而异。 这是个好问题,我不知道。 我怀疑不是,因为对于那些认为智商测试本质上是种族歧视的人来说,这是一个显而易见的问题,我还没有看到它提出来。 不同的种族表现可能是在控制后的各种测试的残差中的某个地方 g. 这是另一个原因,最公平、最客观的测试是最 g-加载。 这 g- 在过去的几十年里,SAT 的负担显着下降。

    这不是 IQism 领域的作者第一次夸大他们的案例和误导。

    IQism 是 1940 年代中期 SAT 考试的基础。 你指责迪里和海尔误导,经过几十年的媒体洗脑,词汇是智力的最佳代表,八十年的大学入学考试过分强调它?

    虽然你还有另一把斧头要磨,我什至可能会支持你的事业

    你相信那是哪把斧头?

    我怀疑通过采用来自超科学领域的论点,你会得到什么。

    八十多年来一直用于筛选大学申请者和军事人员的领域并没有什么超科学的东西,但似乎确实存在研究人员不愿跨越的盲点和禁忌。 它甚至可能超出此范围。 如果我在起诉哈佛的亚洲法律团队中,我会传唤谷歌,看看他们是否在禁止此类主题的搜索结果。 在深入研究了这个话题几周后,我产生了深深的怀疑。

    • 回复: @res
  306. Antiwar7 说:

    Taleb-IQ 争议摘要:

    1) 智商与在学校和工作中表现的不足和成功方面密切相关。

    2)塔勒布似乎只尊重企业家和绅士科学家(又名独立科学家)。

    显然 2) 比单独的智力更罕见,而且可能取决于更广泛的情况。 也许这就是塔勒布如此轻视智商概念的原因,因为他对 1) 中的成就不屑一顾。

  307. DFH 说:
    @Curle

    最有趣的是迈克菲,他着手在 Balize 创建一个相当于后宫的“社会工程系统”

    后宫里有迷人的女人

  308. res 说:
    @JLK

    另一项评论提出了 gs 是否可能因种族而异的问题。 这是个好问题,我不知道。 我怀疑不是,因为对于那些认为智商测试本质上是种族歧视的人来说,这是一个显而易见的问题,我还没有看到它被提出。 在控制 g 之后,不同的种族表现可能在各种测试的残差中。

    这是一个有趣的段落。 我表面上不同意其中的一部分,但想知道经过更详细的考虑后我们是否会达成一致。

    我认为以下命题是正确的。

    – g 是一个有意义的概念,可能有生理基础。 也许基于整体大脑功能,其中所有区域可能取决于一些共同的物理特征(可能受遗传和环境影响),例如髓鞘形成、突触分支和修剪、代谢因素,例如细胞内/细胞外离子浓度。

    – 在特定人群中给出的智力测试的第一个主要成分表示 g 和该人群共有的(平均)特定特征的组合。 一个推论是不同的人群似乎有不同的 gs,因为第一批 PC 不同。 这里有趣的问题是 g 的大小和不同 PC 的人口特定部分如何比较。

    这让我们想到了。

    – IMO 任何基于测试的 g 估计都将依赖于测试的总体。 我认为评估这一点(并更好地估计 g 本身)的最佳方法是分析大量且异质的人口。 理想情况下,这将导致g 的PC、特定人群概况的PC 和特定技能子组的PC(即海尔图形中的那些)。 一个可能的问题是,在这些 PC 上施加正交性会产生不明显的结果(例如,捕捉 M/V/S 技能配置文件中的亚洲/白人差异如何影响单个因素组件?)。 也许只对 g 和总体差异使用 PCA 而不对因子施加正交性是有意义的。

    另一个想法。

    – 是否有可能从纯粹的物理测量中推导出个人 g 的近似值? 理论上,我可以设想一套全面的物理测量(例如大脑大小、神经连接、髓鞘厚度、细胞离子浓度,还有其他想法吗?),为个人提供一个合理的 g 近似值。

    在座的每个人对这些提议有什么看法?

    PS是否有必要单独考虑智商测试是否因为测试问题或因为规范而具有种族歧视? 您关于 PSAT M/V/S 加权的观点在很大程度上是一个规范问题。 据我所知,歧视的焦点更多地集中在问题上。

    PPS 由于各种原因,我对细胞离子浓度有点着迷。 但我确实认为这很重要,因为它们会影响动作电位的行为。 任何对这个想法感兴趣的人都可能想查看这个在线课程: https://www.coursera.org/learn/bioelectricity

    • 回复: @JLK
  309. JLK 说:
    @res

    我表面上不同意其中的一部分,但想知道经过更详细的考虑后我们是否会达成一致。

    您没有指出您不同意哪一部分,但首先让我澄清一下我的说法:

    在控制 g 之后,不同的种族表现可能在各种测试的残差中。

    我指的是任何 不公平的歧视性差异 在种族表现中,我仍然怀疑它包含在控制 g 后的各种测试的残差中。 当然,大多数差异源于能力和环境因素的先天差异。

    您引用的 2010 Deary 文章是这方面的一个很好的来源。 亲爱的状态:

    只要测试电池相当多样化,来自不同测试电池的 g 因子几乎完全相关

    在包含视觉空间电池以及推理和/或语言电池的意义上,“合理多样化”的影响可以在 #290 后关于美洲原住民测试的引用中看到:

    在对这两种能力进行测试的所有六项研究中,美洲原住民的形象化能力都高于语言能力。 在这些研究中,可视化智商中位数为 89.5,语言智商中位数为 81。在与美洲原住民有遗传基因的东亚人(见第 10 章第 1 节)中也存在同样强烈的可视化能力——弱语言能力概况密切相关。

    换句话说,在 g 根据对美洲原住民的各种测试计算得出的结果将比基于 Wordsum 之类的测试更准确。 像 Stanford-Binet(如果我没记错的话是本研究中使用的)这样的测试具有空间电池与 g (0.9 或以上)远高于 Wordsum。 这与我的论点一致,即任何 不公平的歧视性差异 种族表现可以在控制后的各种测试的残差内占到 g. Wordsum 的残差更大,更具种族歧视性。

    在特定人群中给出的智力测试的第一个主要成分表示 g 和该人群共有的(平均)特定特征的组合。 一个推论是不同的人群似乎有不同的 gs,因为第一批 PC 不同。

    我不确定你所说的“主要成分”是什么意思,但迪里说不会,只要测试的电池相当多样化。 分数可能因组而异,但这是一个不同的概念。

    • 回复: @res
    , @utu
  310. res 说:
    @JLK

    您没有确定您不同意的部分

    我特别指的是“我怀疑不是”关于 gs 是否可能因种族而异。

    我认为我们在歧视性问题上基本上是一致的。 值得注意的是,这些可能以各种形式出现,并且符号可能会因所选指标而异。 例如,正如您所说,Wordsum 可能会错误地估计跨种族的 g(例如,与包含空间成分的测试相比,黑人相对于亚洲人的表现可能更好),但它仍然可能会显示出一些人认为是歧视性的差异。 甚至可能在现实中具有歧视性,具体取决于所选的特定单词。

    我不确定你所说的“主要成分”是什么意思,但迪里说不会,只要测试的电池相当多样化。

    啊,这很关键,因为它支持了我试图提出的观点。 我在主成分分析(PCA)中使用主成分: https://en.wikipedia.org/wiki/Principal_component_analysis
    我指的是 第一 一组不同智力测试的 PC 作为 g 的一个定义: https://en.wikipedia.org/wiki/G_factor_(psychometrics)#Factor_structure_of_cognitive_abilities

    你能详细说明迪里在哪里和什么地方说了什么与此相反的吗? 对于一个具体的(尽管是极端和人为的)示例,假设一个组总是得分 M = 2V,而另一组总是得分 V = 2M,那么 MV 空间中的相应第一台 PC 将是(为简单起见,未标准化为长度 1)(1, 2)和 (2, 1)。 两个组的相同样本量组合(假设范围相同)将 (1, 1) 作为第一台 PC。 其中哪一个应该被认为是g?

    分数可能因组而异,但这是一个不同的概念。

    同意(我认为)。 我指的是每个组中分数相互关联的差异。 不是分值本身。

    PS Jensen 提出了一个论点和数据,即 g 在人群中是稳定的,所以我可能错了。

    从第46页 http://arthurjensen.net/wp-content/uploads/2014/06/Psychometric-g-Definition-and-Substantiation-2002-by-Arthur-Robert-Jensen.pdf

    跨人群

    假设一个测试组合中的所有子测试在心理测量上都适合从两个或多个不同人群中选择的受试者,无论定义如何,所获得的组合的 g 因子在不同人群中都是高度相似的。 心理测量上的合适是指测试具有大致相同的心理测量特性,例如相似的可靠性系数、没有地板和天花板效应,以及每个项目与总分之间非常相似的相关性(即项目-总相关性)。 当满足这些充分测量的标准时,从各种测试电池中美国黑人和白人群体的代表性样本获得的 g 负载之间的平均同余系数为 + .99,或虚拟身份(Jensen,1998,pp. 99-100;374-375)。 在日文版韦氏智力量表分测验(日本)中日本人的 g 载荷与美国标准化样本中的 g 载荷之间存在相同的同余系数。 在欧洲样本中发现了类似的一致性 (Jensen, 1998, pp. 85-86)。

    Jensen,1998 年是 G 因素。

    我会对查看数据感兴趣,这样我就可以了解不同的 M/V/S 倾斜如何在数学上产生这种情况。 FWIW 我认为亚洲和黑人的比较最有可能显示出这种效果,因为我相信他们在 M/V/S 倾斜方面的差异最大。

    • 回复: @JLK
  311. utu 说:
    @JLK

    这个说法:

    只要测试电池相当多样化,来自不同测试电池的 g 因子几乎完全相关

    你采取了形式亲爱的p。 204

    http://larspenke.eu/pdfs/Deary_Penke_Johnson_2010_-_Neuroscience_of_intelligence_review.pdf

    它基于 Johnson(Deary 论文中的参考文献 110):

    http://emilkirkegaard.dk/en/wp-content/uploads/Still-just-1-g-Consistent-results-from-five-test-batteries.pdf

    谁确实采用了 46 x 46 的荷兰协方差矩阵,该矩阵由 5 组用于 500 名受试者的测试组成。 他没有个人分数,只有协方差矩阵。 他为每个电池提取 g 并获得不同 g 之间的相关性。 在表 2 的上半部分,他列出了相关性。 您可以看到测试 3 和 2 的相关性为 0.79,3 和 4 为 0.77,3 和 1 为 0.88。 很明显,测试 3 即 Cattell 文化公平测试并没有“几乎完美”相关。

    但实际上,测试 1,2、4、5 和 XNUMX 的 g 之间的相关性很高。 但那又怎样? 毕竟,它们都是在同一个超科学范式下,受共同“群体思维”影响的人设计的“智力测试”。 该测试中的每一个都是针对行业内的其他现有测试而编制(设计)的,并进行了调整,直到它们最大化它们之间的相关性。

    令人惊讶的是,像约翰逊或迪里这样的成年人可以像孩子一样顽固地坚持单一的 g 结构,并在他们的确认偏见范围内继续撰写论文,而不会走出去发现该结构本身在本体论和认识论上存在缺陷。

    • 回复: @JLK
    , @CanSpeccy
  312. JLK 说:
    @res

    你能详细说明迪里在哪里和什么地方说了什么与此相反的吗? 对于一个具体的(尽管是极端和人为的)示例,假设一个组总是得分 M = 2V,而另一组总是得分 V = 2M,那么 MV 空间中的相应第一台 PC 将是(为简单起见,未标准化为长度 1)(1, 2)和 (2, 1)。 两个组的相同样本量组合(假设范围相同)将 (1, 1) 作为第一台 PC。 其中哪一个应该被认为是g?

    我会看看迪里的盒子 2。在你的假设中,一组同样权重 M 和 V 的测试将是“合理多样化”的,足以不歪曲 g.

    您提供的 Jensen 引用本质上是在说同样的事情,尽管他没有详细说明“心理测量上的合适”是什么意思与现实世界的例子。 对于该领域的专业人士来说,似乎有一些第三条轨道可以阻止他们像我在 #315 帖子中那样过于直率和具体。

    我会对查看数据感兴趣,这样我就可以了解不同的 M/V/S 倾斜如何在数学上产生这种情况。

    我也会。如果你找到一些,请给我一个提示。

    FWIW 我认为亚洲和黑人的比较最有可能显示出这种效果,因为我相信他们在 M/V/S 倾斜方面的差异最大。

    我建议看一个亚裔和犹太人的比较,因为两组之间的智商存在明显的 VS 偏斜,在智商上没有很大的长期差异,但祝你好运找到数据。 互联网上不乏吹捧犹太人智力的文章,但自从贝克曼 1972 年的研究发现空间智商为 91.5 以来,几乎没有关于他们空间能力的文章。 你会认为很容易找到一项研究列出了斯坦福-比奈空间分量上的犹太人分数,就像我引用的美洲原住民研究一样,但事实并非如此。

    • 回复: @res
  313. JLK 说:
    @utu

    令人惊讶的是,像约翰逊或迪里这样的成年人可以像孩子一样顽固地坚持单一的 g 结构,并在他们的确认偏见范围内继续撰写论文,而不会走出去发现该结构本身在本体论和认识论上存在缺陷。

    您有权发表您的意见,但您的帖子中没有任何内容表明单数理论存在缺陷 g.

    我没有像许多专家那样在该领域度过我的一生,并且在这个问题上持开放态度。 但请记住,证明存在黑色将是古尔德人群的圣杯 g 和一个白色 g,而且心理测量学家花了几十年的时间让黑人达到任意的白人标准,就像门格勒博士用一套卡尺一样。

    证明这一点的研究资金不会短缺,但它还没有发生。 g 看起来它会留下来。

    • 回复: @utu
  314. res 说:
    @JLK

    互联网上不乏吹捧犹太人智力的文章,但自从贝克曼 1972 年的研究发现空间智商为 91.5 以来,几乎没有关于他们空间能力的文章。

    你看过这些报纸吗?
    http://psycnet.apa.org/record/1975-21109-001
    https://journals.sagepub.com/doi/abs/10.2466/pms.1977.45.1.279

    • 回复: @JLK
  315. may mori 说:

    在这个论点中,我没有强烈的感觉,但这篇文章更像是打败了一个稻草人。 这是我对塔勒布观点的最佳总结:

    数学提供的结论是通过满足某些假设来确定的。 如果这些假设不正确,那么您获得的任何结果都是虚假的。

    除其他外,统计回归要求任何无法解释的变化都是正态分布的,并且所有点的变化均等。 更简单地说,它要求可以将结果和预测变量之间的关系分解为趋势和完全随机的错误。

    智商与财务成功之间的关系显示出不均等的方差,而且肯定不服从正态分布。 这种观察违反了数学证明正常工作所需的假设,因此得出了统计结论的有效性。 特别是,关于统计显着性和解释变异的结果可能会被夸大。

    塔勒布并不是说智商是一个随机值。 他提出了可以从中产生结果的合理替代分布,并且提出统计声明的很大一部分是排除分布。 可以排除的是智商与成功没有关系。 不能排除的是,智商只是捕捉到了智力中最简单的部分,只有智障者没有这些部分,并且遗漏了一些非常重要的东西。

    实际上,这意味着如果你试图预测成功,智商可能没那么有用。 举个例子,假设智商是平均​​分布的,一半的人智商低,一半的人智商高。 低智商的人可能有 0.1% 的成功机会(取决于你如何定义),而高智商的人可能有 0.7%。 如果是这样的话,绝大多数成功人士的智商都会很高。 您甚至可能会看到 Grinblatt 论文中的“相关性”。 但是,以这种方式选择工作人员绝对是垃圾。 你的失败率将是难以置信的。

    关键是有更好的方法来评估能力,而这个方法被不懂数学的人赋予了数学的外表。 如果你想就此与 Taleb 争论,直接谈论相关性并且从不理解使它们起作用的基本假设会让你看起来像个傻瓜。

  316. utu 说:
    @JLK

    你真的认为培养门格勒有用吗? 你是犹太修辞学院的毕业生吗?

    “古尔德群体”对有关 g 因子的技术争论不感兴趣,就像他们对讨论独角兽的属性不感兴趣一样。 假设独角兽存在并且所有独角兽都是一样的来自 Jensen 人群。

    此外,如果一些测试电池显示黑人、白人、亚洲人和犹太人的 PCA 结构不同,“古尔德人群”宁愿远离它,因为他们首先不想处理种族差异。 这应该是不费吹灰之力的。

    您似乎非常热衷于指出种族差异,并不断暗示犹太人的 PCA 结构与亚洲人不同。 如果你相信这一点,你就不能同时相信存在一个唯一的单数 g。 你至少需要两个因素来解释犹太人和亚洲人的测试结果。 这意味着 Jensen 的假设已经过时了,还是 Jensen 的超科学只对白人有效? 显然你没有想到。 我以前告诉过你 “我什至可能会支持你的事业,我怀疑你是否会通过采用来自超科学领域的论点来获得任何好处。”

    如果你不假思索地跟随迪里和约翰逊,他们展示了一个图表(基于 500 名荷兰学员的样本),其中一些测试与他们的 g 具有非常高的相关性,而一些测试与相同的 g 具有较低的相关性,你没有意识到这本身指向第二个不容忽视的因素的存在。 如果您有数学想象力,您会发现可以构建一系列测试,其中其主要因素与视觉或语言子测试(您的选择)的相关性甚至更低,并且该主要因素与 g 的差异越来越大原电池测试。 您可以通过重新设计测试或什至仅在测试之间调整个别问题(即通过增强电池的视觉或语言部分)来操纵 g 并制作几乎任何您想要的东西。 你用“g看起来像它要留下来”这句话表达的坚持只是表明你还没有考虑到这个问题。 问自己一个问题,为什么我们没有 g 检验? 为什么一些国际智商研究机构没有将智力换成克单位? 有没有可用的ag测试? 为什么不进行一项与 g 100% 相关的测试? 为什么我们需要用马来定义独角兽?

    • 回复: @JLK
  317. JLK 说:
    @res

    你看过这些报纸吗?

    不,我注意到他们都来自 1970 年代,就像 Backman。 我很想看看他们的数字,但我不会为副本付费。

    理查德林恩在 2002 年发表了一篇关于犹太情报的文章(不涉及这两项研究),得出的结论是,需要对德系犹太人情报概况进行更多研究。

    http://www-personal.umich.edu/~negisama/asdf2.pdf

    • 回复: @res
  318. JLK 说:
    @utu

    对因子分析和主成分分析的优缺点的简短讨论似乎表达了您关注的核心:

    https://www.thoughtco.com/principal-factor-analysis-3026699

    主成分分析和因子分析的问题

    PCA 和 FA 的一个问题是没有标准变量来测试解决方案。 在判别函数分析、逻辑回归、轮廓分析和多变量方差分析等其他统计技术中,解决方案是根据它对组成员的预测程度来判断的。 在 PCA 和 FA 中,没有用于测试解决方案的外部标准,例如组成员身份。

    PCA 和 FA 的第二个问题是,在提取之后,有无数次可用的旋转,所有这些都说明了原始数据中相同的方差量,但与定义的因子略有不同。 最后的选择留给研究人员根据他或她对其可解释性和科学效用的评估。 研究人员经常对哪种选择是最好的意见不一。 第三个问题是 FA 经常被用来“拯救”构思不佳的研究。 如果没有其他适合或适用的统计程序,则至少可以对数据进行因子分析。 这让许多人相信,各种形式的 FA 都与草率的研究有关。

    我已将我认为对本次讨论最重要的部分加粗。 如果您将智力测试视为登山比赛,犹太人在一侧攀登,亚洲人在另一侧攀登,那么过多地关注无限可能的旋转次数之一当然是歧视性的(从顶部向下看)比亚洲/美洲原住民/墨西哥一侧更接近山的犹太人一侧。 迪里通过建议因子具有合理数量的多样性来解决问题。

    PSAT/SAT 上的两门语言、一门数学和没有空间的内容并不合理。

    我对单数持开放态度 g 理论,但请记住,因子分析并非不可避免地解决单个解决方案,并且测试与最密切相关 g 是最长和最多样化的。 似乎有一个收敛。

    • 回复: @res
    , @utu
  319. res 说:
    @JLK

    感谢林恩链接。 我正在尝试决定是否购买 Storfer 的这本书,因为它一直显示为参考: https://www.amazon.com/Intelligence-Giftedness-Contributions-Environment-BEHAVIORAL/dp/1555421857

    这两篇论文在 libgen.io 上可用 DOI 搜索效果最好,我链接到的页面有 DOI。

  320. res 说:
    @JLK

    如果您有兴趣搜索历史,那么在此博客上与 utu 讨论因子分析和 PCA 的历史由来已久。

    请记住,因子分析并非不可避免地解决单一解决方案

    这适用于因子分析,但不是 PCA。 这就是我专注于讨论 PCA 的原因。 尤其是在查看第一个组件时。 但是,当试图映射到常见概念(例如 g、数学、语言、空间)时,因子分析可能更有意义。

    表达我在上面试图提出的观点的一种方式是,我怀疑第一台 PC 主要由 g 以及该人群的特定配置文件组成。 问题是这两个数量相对较大。 我怀疑 g 大了一点,但发现 0.99 的相关性对于广泛不同的人群来说是不可信的。

  321. anonymous[417]• 免责声明 说:
    @Nobody

    这个世界上有很多数学。

    我可以花几个小时研究质数,我可以得到任何人想要的技术,但归根结底,我对柏拉图的数学一无所知,他的作品包括数量非常有限的方程——有我想,我对数学的了解没有什么是柏拉图不会很快理解的,只要我足够聪明,能够很好地解释它:而且我想你,归根结底,认为塔勒布会认为柏拉图只是“冗长的”如果柏拉图不同意塔勒布的话。 我相信你会错的。

    请更加小心地称呼整个群体——其中至少有一个,如果至少有一个,那么可能不止一个——可能比你想象的聪明得多——江湖骗子。 当然,塔勒布的评估在一定程度上是正确的,但对他的主张的挑战并不全是江湖骗子,相反,这一事实并不使他成为江湖骗子。

    讲礼貌! 表示尊重! 人们通常不是很聪明,从真实的角度来看,可能不是塔勒布,可能不是你,可能不是我,相互尊重有助于我们相互寻求诚实和真理。

    不要像孩子一样等待有人讨论塔勒布的数学并向你展示数学中的错误! 长大了,自己评估数学。 很可能其中没有错误! 但地图不是领土,如果波普尔先想到它,他会说的。

    • 回复: @JLK
  322. utu 说:
    @JLK

    对因子分析和主成分分析的优缺点的简短讨论似乎表达了您关注的核心:

    My 应该是你的顾虑。 我试图带来重要的一点来拯救你自己,但你假装你不明白:

    您似乎非常热衷于指出种族差异,并不断暗示犹太人的 PCA 结构与亚洲人不同。 如果你相信这一点,你就不能同时相信存在一个唯一的单数 g。 你至少需要两个因素来解释犹太人和亚洲人的测试结果。 这意味着 Jensen 的假设已经过时了,还是 Jensen 的超科学只对白人有效?

    就我而言,它也与 FA 和 PCA 以及心理测量师如何使用 FA 有关。 它不是一个客观的过程,而是基于所使用的不同标准的主观。

    https://pdfs.semanticscholar.org/1691/2e7c392550ab7f18f9b0096f04a0d239bd0e.pdf
    分解的主成分解决方案经常旋转十次(下一步),以实现因子载荷的简单结构。 然而,旋转方法并不能提高因子结构对数据的拟合程度:所解释的协变只是在因子之间重新分配。 旋转旨在生成易于解释的解决方案。 旋转方法的选择同样是任意的,什么构成了“更简单”的结构 是一个判断的问题,即使解决方案完全是数学的。 一个重要但随意的决定是旋转方法是否应该导致正交(不相关)或倾斜(相关)因素。

    FA 允许您进行大量任意数据按摩,直到获得所需的结果。 这些方法就像博士学徒教授的炼金术配方,他们从大师那里学习并在其他环境中复制,这就是不同炼金术士群体之间明显的一致性和一致性的实现方式。 他们的活动最好用中世纪工艺和行会的模型来描述,而不是用科学来描述,这就是为什么我坚持称它为超科学的原因。 当一些炼金术士决定像瑟斯顿或卡特尔一样走自己的路时,也有例外。 同一个 Cattell,其 g 与 Johnson 论文中的其他 g 没有很好的相关性。

    我的批评不仅仅是 FA 的古怪和任意性:

    https://www.unz.com/jthompson/fear-and-loathing-in-psychology/#comment-1948489
    我试图解释,但显然失败了,通常使用的测试电池是以这样一种方式构建的,因此它们会产生一个主导因素,正如 Spearman 和 Jensen 所希望的那样,这一事实被用作修辞手段(例如 Jordan Peterson)认为 IQ 或 g 是唯一重要的智力,因为所有其他因素都很弱,因此可以忽略不计。 但我认为,一个人可以以不同的方式构建一组测试,因此这两个因素将获得大致相等的强度,然后就不能再说只有一个单一的智力,而 g 奇点论证的修辞力量是丢失。 我不知道因素 g 和 s 可能被称为什么以及它们的解释可能是什么。 这超出了我的论点和兴趣。 与斯皮尔曼平行的还有其他研究人员,比如瑟斯通(如果我没记错的话),他们主张智力不是斯皮尔曼所设想的单轴,因此不能忽视其他因素。 我的论点比瑟斯顿的论点更普遍、更强有力。 我声称这都是任意的,因为我可以以不同的方式构建测试和子测试以及协方差矩阵,从而产生让瑟斯顿高兴或斯皮尔曼高兴的结果。 因此,Jordan Peterson 的论点“g 吞噬了所有的变化”只是局部的,对于所使用的特定矩阵,而不是一般论点。 同样,瑟斯顿的论点说 g 比 s 强 x 倍也是局部论点,而不是一般论点,因为可以构建一组测试,其中 g 将 y 时间比 s 强,其中 y≠x。 我想在这一点上引用 GB Shaw 的话可能是合适的:“他们、Spearman 和 Jensen 是野蛮人,他们认为他们的部落和岛屿的习俗是自然法则。”

    • 回复: @JLK
    , @JLK
  323. JLK 说:
    @utu

    在你的 PCA 与 FA 的斗争中,我不一定有一只狗,因为我在这个评论线程上的大部分评论都与我认为大学入学考试中的歧视性偏差有关。 我认为我的观点不管怎样。 你的评论引起了我的兴趣,我可能会稍后介入并出于纯粹的求知欲来解决它们,因为这些天我大部分时间都很无聊并且有太多空闲时间在我手上。 但在我这样做之前,我希望你澄清你在一个问题上的立场:

    您是否认为词汇因素与空间和推理因素一样重要?

    • 回复: @utu
  324. utu 说:
    @JLK

    “我可能会稍后介入并解决他们” - 如果您这样做,请分享您的发现。 我自己是在 2-3 年前开始阅读 UR 时才对这个问题产生兴趣的,我第一次遇到了 IQism 的真正信徒,并开始倾听他们使用的语言和论点。 我从来不知道像 Sailer 或 Derbyshire 这样的人。 他们来自哪里? 是什么让他们抽动? 他们的信仰体系、他们的心理、他们的认识论? 在此之前,我对此一无所知。 为了了解发生了什么,我必须在某种程度上解构他们的信仰体系。 现在我知道 g 概念只是 IQists 使用的一种修辞手段。 使用一个短语,即某事已加载或不应该让你的对手闭嘴——这是一个致命的论点。 这种机制与苏联政府官员没有太大区别,他们会使用马克思主义的科学性质的论点,即共产主义在科学上是不可避免的,或者自由经济学家关于市场力量隐藏之手的论点。

    对于测试的哪些部分(空间、词汇或推理)更重要,我没有意见。 重要的是什么? 然而,我确实相信,如果我们想量化和测量智力,它不是像 g 或 IQ 这样的数字,而是一个向量。 我们需要多项措施而不是一项。 这些措施的相对优势之间是否存在种族或种族差异? 它们在多大程度上是可遗传的? 如果我们决定在社会工程中使用这些措施,我们应该如何使用它们来建立一个公正的社会? 这些措施是否有目的地偏向某一民族? 如果是这样,它是怎么发生的? 这些是您似乎关心的重要问题。 你应该追求它。 但是你需要绕过智商主义和心理测量研究的许多谬误。 你会发现没有那么多数据,比如说犹太人的智商,而且对其平均值的估计相差很大。

    • 回复: @CanSpeccy
  325. @utu

    为什么你认为这篇文章中的汤普森博士没有引用一个数字来表示相关性、成功的可预测性、遗传力以及其他任何 IQ 分数应该表现得如此出色的数字? 他没有这样做是因为他不记得这些数字中的任何一个,因为它们在非常宽的括号中给出,如果他引用其中任何一个,它们可能会受到质疑。

    对于那些想要参考显示智商与生活成功之间相关性的工作的人,这里有一篇——诺贝尔奖获得者詹姆斯赫克曼和其他人的论文,揭示智商只提供生活成功的最模糊暗示,在在学校成绩和成绩测试结果方面:

    什么成绩和成绩测试衡量

    Lex Borghans、Bart HH Golsteyn、James J. Heckman 和 John Eric Humphries
    PNAS 22 年 2016 月 113 日 47 (13354) 13359-8; 提前出版 2016 年 XNUMX 月 XNUMX 日 https://doi.org/10.1073/pnas.1601135113
    供稿人:James J. Heckman,19 年 2016 月 22 日(2016 年 XNUMX 月 XNUMX 日送审;由 Armin Falk 和 Patrick Kyllonen 审阅)

    特曼的研究也显示了同样的情况,许多高智商的人从事卑微的工作,收入几乎是最低的,经常受到酗酒、离婚、抑郁等的折磨。

    • 回复: @utu
  326. @utu

    然而,我确实相信,如果我们想量化和测量智力,它不是像 g 或 IQ 这样的数字,而是一个向量。

    为什么你认为智力是可以量化的? 它甚至是一回事吗? 在几秒钟内解决魔方的能力是否取决于允许一个人创作一首伟大的交响乐或另一个人制定有效的战斗策略的相同思维质量?

    大脑是模块化的,具有多种类型的神经元和神经递质(最近统计超过 80 种),这意味着在心理能力之间必须存在独立的遗传变异,这些变异以大多数极端形式被专家揭示:例如,格洛丽亚·伦霍夫 (Gloria Lenhoff),智商60 岁左右,知道 15 种语言的歌剧乐谱——她能够交流的语言(她还知道手语。); 但谁,谁,数字不知道,可能是因为她所患的威廉姆斯综合症剥夺了她处理数字的大脑模块。

    一般智力因素只能存在于非常基本的水平。 例如,心血管健康将影响所有大脑功能,同样许多其他身体和代谢因素也会影响整个大脑。 但仅仅因为那些在所有方面都健康的人,在所有其他条件相同的情况下,具有比患有循环系统或代谢疾病的人更高的平均心理测量 g 因子,这并不意味着他们都是天才。 大多数将只是正常的健康人。

  327. utu 说:
    @CanSpeccy

    Borghans 等人的结论:

    “认知技能预测生活结果。 本文通过分析广泛使用的认知技能代理——成绩和成就测试的组成部分,重新​​解释了认知技能与各种重要生活结果之间关系的证据。 人格测量可以预测成就测试分数和高于和超出智商分数的成绩。 使用成绩测试分数和成绩作为 IQ 代理的分析将 IQ 的影响与人格的影响混为一谈。 这两项指标都比单独的智商和人格具有更大的预测能力,因为它们体现了我们的指标没有捕捉到的额外人格维度。”

    “尽管数据集之间存在差异,但出现了一致的模式。 人格是所有数据集中大多数生活结果的强大预测指标。 成绩和成绩测试分数比智商更能预测成人的结果。”

    “请注意,这两种措施的大部分差异仍然无法解释。”

    • 回复: @CanSpeccy
  328. JLK 说:
    @anonymous

    当然,塔勒布的评估在一定程度上是正确的,但对他的主张的挑战并不全是江湖骗子,相反,这一事实并不使他成为江湖骗子。

    如今,对主流心理测量科学的许多攻击都与美国大学入学考试有关,主要是 PSAT/SAT,因为它们是大多数美国人曾经参加过的最接近 IQ 测试的东西,而且它们起到了至关重要的作用。学业和生活成功的守门人。

    毫无疑问,包括高级决策者在内的许多政党都知道我在第 315 号帖子中指出的问题,在某些情况下几十年来,除了在各种可悲的人中,没有任何公开辩论的迹象,本文将很快就会离开头版并永远被遗忘。

    迟早,这个问题将在集体诉讼中暴露出来,决策者将面临困境。 他们可以按照我建议的方式修复测试,但随后他们将不得不面对某些人口群体的表现可能下降的后果,然后他们必须解释为什么他们的常春藤盟校毕业生和国家过去 50 年的优秀半决赛选手不应该在他们的简历上加上 Roger Maris 式的星号。

    或者,他们可以完全取消测试,这对美国的长期竞争力来说将是一场灾难,并且可能会冻结现有的和不合理的特权水平。

    我会非常注意不诚实的建议,即智商测试突然变成了坏科学,应该和洗澡水一起扔掉。

    • 回复: @Wizard of Oz
  329. @utu

    “请注意,这两种措施的大部分差异仍然无法解释。”

    没错。

  330. @utu

    令人惊讶的是,像约翰逊或迪里这样的成年人可以像孩子一样顽固地坚持单一的 g 结构,并在他们的确认偏见范围内继续撰写论文,而不会走出去发现该结构本身在本体论和认识论上存在缺陷。

    但必须有一个潜在的因素, g, 我, 或者随便你怎么称呼它,否则整个智力的概念,至少就像心理学家所认为的那样,都是假的。 如果基础设施构成 视唱唱唱法 与制定电磁方程或蒙娜丽莎的绘画所依据的设施不同,那么智能不能是一个单一的东西,可以用线性尺度上的一个点来衡量,而必须是一整套花样。

    100 多年来,心理学家一直将他们的可信度押在这样的说法上:智力不仅是一件事,而且是一件可以在 20 分钟的多项选择题测试中降低到分数的单一事物。 而且,现在,作为支持这一主张的证据(心理学中最合理的说法, 根据乔丹·B·彼得森的说法) 未能积累,心理学家必须要么承认另一个巨大的错误,以配合弗洛伊德主义和行为主义,要么继续 BSing。 目前,他们似乎致力于继续使用 BS。

    • 回复: @Wizard of Oz
  331. @CanSpeccy

    我不理解心理学家将他们的可信度押在智力是一件事上,更不用说通过 20 分钟的测试来衡量的了,即使你说的不是多元智能人群,而是儿童心理学家或那些筛选求职者的人。 你从哪里得到相反的观点?

    我的理解是,通常会有基于概率范围的指导,以便父母可以放心,他们的孩子(几乎可以肯定)完全准备好接受算术挑战,或者雇主告诉 2 名申请人中只有 10 名可能能够在 4 周的培训期内掌握所需的技能。

    • 回复: @CanSpeccy
  332. obwandiyag 说:
    @Intelligent Dasein

    总的来说,廷巴克图和古代马里是复杂的社会,比当时的一些欧洲社会更先进。 这个博客上的所有这些讨论都只是一般性的概括,通常可以用明显且易于发现的例外来进行。

    • 回复: @Colin Wright
  333. @JLK

    有点令人沮丧的是,我会远远超出您和其他评论者对关注此线程的兴趣,以获得足够的关于 FA、PCA、正交变换、旋转等的数学知识,以自信的专业知识讨论技术问题。 但请允许我尝试一些简单的常识。

    假设大脑的生理特征和功能有许多相似的组合,除了一些异常值(如学者)之外,它们对速度和正确性(在现代术语:生存可能是古标准)认知任务的结果(不排除颜料的最佳选择或球员的选择)? 即使处理速度(?s)、工作记忆等的精确组合在非专家中可能在一个很小的范围内变化,g 是否可以被合理地视为大脑这些元素的质量和效率的衡量标准?

    略有不同的主题...... 鉴于 SCOTUS 规定对求职者使用 IQ 测试的限制,如果雇主说“我们每年选择 10 名研究和 IT 人员、15 名销售人员和 20 名一般管理人员”,那他会怎么做? 我们分别使用心理咨询顾问的数学、语言和视觉空间测试来缩小这些领域,我们对结果感到满意(顺便说一句,在一般管理晋升方面,所有表现最好的人往往表现良好) ”?

    回到技术细节。 给定语言和数学测试与 g 的 0.7 相关性,以及视觉空间测试的 0.9 相关性(尽管 0.7 和 0.9 之间的差异不是我的问题的关键)可以在给定的概率下对 g(智商)做出什么样的预测V、M 或 VS 测试的分数? 尽管 g 相等,但个人在不同测试中的分数差异很大的可能性有多大?

    • 回复: @JLK
    , @anonymous
  334. 雇主被告知必须如何选择员工,这真是太奇怪了。

  335. @Wizard of Oz

    我不理解心理学家已经把他们的可信度押在智力是一个单一的东西上

    维基百科是这样定义智商的:

    智商 (IQ) 是从多项旨在评估人类智力的标准化测试中得出的总分。

    这肯定意味着智商,一个单一的数字,衡量智力,暗示必须是一个单一的东西,因为你不能用一个数字来衡量自变量。

    如果维基百科错了,为什么一些心理学家不纠正它们?

    但无论你怎么理解,心理学家都提倡智商衡量智力的观点。 这是乔丹·彼得森,多伦多大学教授,也许是世界上最著名的在世心理学家,当然也是最富有的人之一*:

    自 1920 年代以来,人们一直在研究智力,IQ 智力,它是心理学的一个非常成熟的分支。 关于智商研究,我要告诉你的一件事是,如果你不买智商研究,你还不如扔掉所有其他的心理学。 ……智商人士对智力的定义比我们几乎能够定义任何其他心理构造的方式都更加严格和准确。

    此后,彼得森继续努力区分智力形式、实用与分析等。

    此外,彼得森认为智商是我们生活成功的最佳预测指标。 但是作为 诺贝尔奖获得者詹姆斯赫克曼等人的论文显示, 智商实际上是生活成功的不良预测指标,例如以收入衡量,不如成就测试、学校成绩和性格测试。

    ---
    顺便说一句,彼得森声称自己的智商为 150,如果有人接受他关于 IQ 测试的预测能力的说法,这与他作为千万富翁的学术企业家和公共知识分子的地位是一致的。 然而,根据彼得森揭穿书的作者西奥多·比尔收集的证据“乔丹学” Peterson 未能通过加拿大 LSAT 考试,IQ 不能超过 124,这恰好在教授范围的中间,并且似乎与 Peterson 有点狡猾的推理过程一致。 但是,如果我们要考虑挥手一种独特的智力形式,(见上面的视频链接)我会把彼得森列为天才。

    • 回复: @CanSpeccy
  336. JLK 说:
    @utu

    就我而言,它也与 FA 和 PCA 以及心理测量师如何使用 FA 有关。 它不是一个客观的过程,而是基于所使用的不同标准的主观。

    我们可以使用类似十项全能的评分系统设计所谓的智力测试,其中十个(或任何数量)不同的子测试加权相同,按百分位单独评分,然后计算分数。 但是,如果没有某种类型的统计分析,我们将无法评估每个子测试对整体智力和专业效能的重要性,或者所选测试是否重叠并因此超重某些因素质量与其重要性不成比例。

    这是一个真正的十项全能的因素分析: https://eric.ed.gov/?id=EJ172671

    它解决了四个解决方案,而不是一个平衡的智力测试总是解决一个 g,表明十项全能运动员在确定最佳田径运动员方面比使用 FA 分析更好。

    有明确的相关性 g 诸如获得高级学位、收入、犯罪和长寿之类的东西。 我们可以修改 Dear/Haier 图形以将它们插入到小圆圈上方 g 在里面。 但是我们可以很容易地使用路径分析来消除 g 例如,计算空间智能与获得高级学位、收入、犯罪率和寿命之间的直接相关性。 我们甚至可以更进一步,计算其中一项空间测试与此类生命成就的相关性。

    所以,至少为了我构建最好的大学入学考试的目的(并证明词汇的相对重要性),我们可以不 g 并且基本上达到相同的权重。 只会更难,这就是为什么 g 有实用性。 与十项全能不同,智力测试收敛了一个单一的 FA 解决方案,因此使用它有更好的理由。

    • 回复: @res
    , @utu
  337. JLK 说:
    @Wizard of Oz

    假设大脑的生理特征和功能有许多相似的组合,除了一些异常值(如学者)之外,它们对速度和正确性(在现代术语:生存可能是古标准)认知任务的结果(不排除颜料的最佳选择或球员的选择)? 即使处理速度(?s)、工作记忆等的精确组合在非专家中可能在一个很小的范围内变化,g 是否可以被合理地视为大脑这些元素的质量和效率的衡量标准?

    g 只是一个数学结构,你提到的所有物理因素加上神经元密度和葡萄糖摄取等更多因素可能会产生影响。 就像心脏容量、腿长、体脂百分比和肌肉纤维比率对马拉松时间有影响一样。

    略有不同的主题...... 鉴于 SCOTUS 规定对求职者使用 IQ 测试的限制,如果雇主说“我们每年选择 10 名研究和 IT 人员、15 名销售人员和 20 名一般管理人员”,那他会怎么做? 我们分别使用心理咨询顾问的数学、语言和视觉空间测试来缩小这些领域,我们对结果感到满意(顺便说一句,在一般管理晋升方面,所有表现最好的人往往表现良好) ”?

    Griggs诉Duke Power 禁止将 IQ 测试用于招聘目的,这导致公司使用更具歧视性的 SAT 分数(出于我在帖子 #315 中指出的原因)! 对于男性、亚洲人、美洲原住民和墨西哥人来说,这里有巨大的诉讼潜力! 我希望他们能坚持下去,看看《纽约时报》如何围绕推论问题跳踢踏舞会很有趣。

    回到技术细节。 给定语言和数学测试与 g 的 0.7 相关性,以及视觉空间测试的 0.9 相关性(尽管 0.7 和 0.9 之间的差异不是我的问题的关键)可以在给定的概率下对 g(智商)做出什么样的预测V、M 或 VS 测试的分数? 尽管 g 相等,但个人在不同测试中的分数差异很大的可能性有多大?

    这将取决于测试的细节。 除了通过调整 V/M/S 的权重来堆叠甲板之外,还有很多方法可以向它们暗示偏见。

    • 回复: @Wizard of Oz
  338. res 说:
    @JLK

    感谢您提供十项全能 FA 链接! DOI 是 10.1080/10671315.1977.10615462,对于其他想要全文的人。 那篇论文着眼于四种不同的因子分解,鉴于上面的 FA 与 PCA 对话,这很有趣。

    但真正令人震惊的是(以及对“g 只是一个数学工件”阵营的谴责)是如何将性能分解为 3-4 个大致同等重要的因素(如您所述)。 部分摘录:

    因此,应用了以下四个程序。
    (A) 根据直接 quartimin 标准进行倾斜旋转的主(截断)分量分析(Jennrich & Sampson,1966)
    (B) 根据 varimax 准则 (Kaiser, 1958) 进行正交旋转的主(截断)分量分析。
    (C) 根据 Harris-Kaiser 标准 (Harris & Kaiser, 1964) 进行倾斜旋转的最大似然(唯一性重新缩放)分析。
    (D) 根据 varimax 准则 (Kaiser, 1958) 进行正交旋转的最大似然(唯一性重新缩放)分析。

    除了可解释性之外,更喜欢哪个因子解决方案是根据特征值的大小(Guttman,1954;Kaiser,1960)、提取的总方差比例(Gorsuch,1974)以及显着和超平面载荷方面的模式简单性(Cattell,1966 )。 显着载荷和超平面载荷分别定义在 +/- 0.40 区间外和 +/- 0.20 区间内 (Cattell, 1966; Gorsuch, 1974)。

    根据主成分分析,前三个特征值——3.78、1.51、1.10——大于0.92; 第四个是 0.72,第五个是 64.0。 三、四、五因素提取的总方差比例为73.2%。 分别为 80.4% 和 53.1%。 最大似然分析的相应方差比例分别为 58.5%、62.5% 和 XNUMX%。 关于可解释性,各自的四因素解决方案 - 倾斜或
    正交旋转被认为是最有意义的。

    从表 2 中可以明显看出,备选程序 A、B、C、D 相互支持通常指示的因子模式的有效性。 因此,标记变量的识别和模式的简单性非常接近
    ...
    所呈现的因素被连续解释为跑步速度(因素 I)、爆发性手臂力量或物体投射(因素 II)、跑步耐力(因素 III)和爆发性腿部力量或身体投射(因素 IV)。
    ...
    根据程序A和B,跑步速度、爆发力臂力和爆发力腿力因子分别提取总方差的约20%,根据程序C和D提取约16%。这表明这三个因素应该大致相等对十项全能锦标赛很重要。 跑步耐力因素不太重要,但分别占总方差的 12% 和 9%。

    女子七项全能类似(但不如 IMO 好): https://www.mdpi.com/2075-4663/4/1/12/pdf

  339. 他们很快就摆脱了华尔街的多元化招聘。

    证明合法移民的不利之处。

  340. @JLK

    谢谢你的耐心。 我也一直试图清楚地了解不同种族(杂交大家庭)有不同的 g 意味着什么。 与种族 B 的 g 因素相比,它是否会通过种族 A 的 g 因素与 V、M 和 VS 子测试(或其他一些子测试)的相关性显着不同来显示?

    • 回复: @JLK
  341. anonymous[417]• 免责声明 说:
    @Wizard of Oz

    在衡量经济成功时,塔勒布教授有点过于兴奋了。

    以创造我们技术文明的人的身份在经济上取得成功的人中的一大部分——由于各种原因(包括但不限于我对地图的理解和领土遗传适应复杂的社会和经济挑战——如果我们能够正确衡量它,我们会毫不犹豫地称之为“g”)——这些人中的很大一部分是那些在使当前世界成为一个谁提醒他们自己在物质上取得了成功(在未来,从他们的角度来看,高克的结果需要几年时间才能发挥作用)。

    例如,收入最高的蓝领工作——例如,在核潜艇上服役或操纵税法——对“钟形曲线”沿线不是那么高的人最有利——塔勒布正确地指出,当衡量标准是现实世界的经济成功。
    但其原因是处于钟形曲线极端(而不是肥尾)的人们以这种方式设计了我们当前的现实世界。

    塔勒布教授提出的这些问题将更容易理解,当一个不同的文明,由不同世代的人在钟形曲线的窄端设计时,一个世纪(或更短时间),已经呈现了所有可能的具体结果或者可能不会在不同的文明下发生.

    数字、基因、当前的成功机会(从低悬的果实到天才的创新),以及非常聪明的人在玩这个系统——这就是这场辩论的原材料。 塔勒布教授在指出我们许多人的某些假设的错误方面一直令人信服,但在替换更好的假设时却完全没有说服力。

    就像我之前在这个评论线程中所说的那样,这个世界上有很多数学,但也有很多复杂性远远超出了我们发现的大量准确数学。

    想象一下阅读 Eco,然后说,这太棒了,然后阅读 Borges,然后说,嗯,这更令人惊奇,然后阅读一些比 Borges 更有灵感和智慧的作者。 (我不会说出我认为你不会相信我的那些好东西,除非你有一个小时来听我的见解......)。 现在,请记住,我们正在讨论现实世界,有限个体人类及其理性与现实世界的关系。 我们需要谦虚,即使是我们中间的科尔莫哥洛夫。 从这个角度来看,塔勒布的论点有时符合常识标准,有时则不然。

    • 回复: @anonymous
    , @Wizard of Oz
  342. anonymous[417]• 免责声明 说:
    @anonymous

    wwebd 说——郑重声明,在我最熟悉的领域,在钟形曲线的最右边没有肥尾。

    百万分之一的埃科对 40 多岁和 50 多岁的博尔赫斯印象深刻,千万分之一,而与 40 多岁和 50 岁的乔伊斯相比,博尔赫斯显然在这条曲线的左边,亿分之一40 多岁的人,而他显然至少比 50 多岁和 XNUMX 多岁的普鲁斯特偏左一点,而后者又明显比早逝的 Peguy 的左侧(例如,在他的史诗《夏娃》中最好的两三千行)。

    Chandrasekhar,很久以前在一个更简单的前硅谷世界中,他以一种非常令人印象深刻的方式弄清楚了大明星是如何过着他们的大生活(想象一下!),他在最后几年致力于尽可能地理解原理。 他并不幻想牛顿并不比他聪明,反之亦然。

    钟形曲线最右侧的这些明显差异暗示着靠近中间也存在真正的差异。 只是一个提示,看在上帝的份上,我知道这只是一个提示,但它就是这样。

    当然,除非你认为神圣的灵感比基因能力更重要。
    这就是我所相信的,在幸运地观察了几十年的聪明人、聪明的动物和聪明的昆虫之后,从卑微的蟑螂到高贵的龙虾(我知道严格来说不是昆虫),但我没有找到很多同意我的人。

  343. @obwandiyag

    “总的来说,廷巴克图和古代马里是复杂的社会,比当时的一些欧洲社会更先进……”

    …和 绝非巧合 受到阿拉伯的影响。

    任由他们自己动手,黑人真的不会写字,不会冶炼金属,不会建造任何比未铺砌的石墙更精细的结构,甚至连最粗略的原理都不会知道。医疗保健,不会设计出比长矛更精致的武器,不会在部落之外没有稳定的政治组织形式,而且总的来说,似乎被合理地描述为异常先进的灵长类动物而不是人类。

    ……但是在那里。 我夸大了。 仍然…

    我怀疑你自己是不是全黑的。 这是可能的,但在统计上不太可能。 你可以拼写。

    • 回复: @obwandiyag
  344. JLK 说:
    @Wizard of Oz

    谢谢你的耐心。 我也一直试图清楚地了解不同种族(杂交大家庭)有不同的 g 意味着什么。 与种族 B 的 g 因素相比,它是否会通过种族 A 的 g 因素与 V、M 和 VS 子测试(或其他一些子测试)的相关性显着不同来显示?

    您可能最好考虑在各种 R/S/V 子测试中具有不同平均分数的组,而不是具有不同的 gs.

    当然,这些都是统计概括。 有很多女性具有很高的 STEM 能力,还有很多犹太人是或曾经是高空间(想想爱因斯坦)。

    以下是对空间思维特征的有趣总结:

    http://www.gifteddevelopment.com/resources/visual-spatial-resource/spatial-strengths

    创造性、好奇、开箱即用的思想家,VSL 通过直观的飞跃学习。 他们记得他们所看到的并忘记他们所听到的。 他们可能会忘记细节,但永远记住大局。

    换句话说,他们通过在课堂环境中阅读来学习得更好。 我们需要对他们进行测试,因为这种特质不一定有利于成为一名告别演说家。

    有时被视为组织能力差,图片思考者有他们的秩序。 它以意义为中心,一种情绪反应。 空间不是像循序渐进的学习者那样勾勒出主要思想,而是像平原上的树木一样突出,而是对重要性的感觉做出反应。 如果某件事让他们觉得有价值,它就会成为他们的基本要素网络的一部分,一张值得关注的事物的心理地图。 而不是轮廓——这对逐步来说很舒服——一个图片思考者的现实方案更像是一张 3D 星图。 各种恒星和星座以不同程度的亮度脱颖而出,都在它们周围的黑暗空间中闪耀,并以某种方式相互关联。 这些联系是基于感觉和感知到的重要性。 有时,图像思考者不仅通过概念看到,而且通过概念感受他们的方式。 它们具有动觉输入,就像那些能够唤起对玩家“体验”的肌肉反应的网络空间现实游戏一样。 这些空间在空间中摸索,好像它们可以触及想法和可能性以找到那里的东西。 能够形象化思维实验的爱因斯坦谈到在他的思维过程中使用“一种想象的、动觉的速记”。 (当他试图解释自己时,他正在摸索,典型的空间。)

    开箱即用的思维

    理解图片思考者的一个重要方面是他们需要以自己的方式思考。 他们对跟随别人的思路感到不舒服,部分原因是这种线性思维不是他们的思维方式。 有时他们真的无法一步一步地跟随。 他们可以采取每一步,但没有那张大图,这些步骤就会消失。 他们记不住细节,除非这些细节具有重要意义,带有他们自己的感受,或者是突然完形的一部分。 图片思想家开辟了自己的思想之路。 大多数人需要处理时间来整理他们自己的大图。 他们的想法似乎没有真正的步骤。 他们经常突然意识到“事情就是这样在一起的!” 慢慢地或在一瞬间,一个完整的概念出现了,它可能是辉煌的,也可能是有缺陷的。

    可能是我们凡人最接近神圣灵感的地方。

    我不是一个纯粹的智商主义者,因为在我接近六年的航行生活中,我很清楚,情商等因素在谁成功和保持成功方面发挥了重要作用。 此外,物质上的成功并不总是衡量个人对社会贡献的最佳标准。 我们需要逆势者、反传统者和喜欢搅局的人,即使社会并不总是宽容他们。

    也许这些天我会在相当多的空闲时间为特殊需要的孩子写一本书,那个喊皇帝没有衣服的孩子属于自闭症谱系的某个地方,错过了他应该不相信他说谎的眼睛的社会线索像其他人一样。

    • 回复: @Wizard of Oz
  345. @anonymous

    在第一眼看到你雄辩的贡献后,我只是一个快速的回应,因为我怀疑的眼睛会发现一些狡辩的机会。

    首先是您对下一代“设计”的第 4 条参考。 哪个“一代”曾经“设计”过这个世界? 为了尝试理解你在说什么,让我问一下可以说上一代设计了世界或世界的任何划时代的方面?

    接下来,您似乎否认了高斯智商曲线上的胖右尾。 但 Cyril Burt 的英国男学生数据表明了这一点。 一旦你在 Ivies 的婚姻市场之前很久就注意到各种交配,这并不奇怪。 考虑格雷格克拉克的“告别施舍”中的证据。 另一种看待它的方式是注意,对于相关估计,美国人口由几个相当不同的人口组成,每个人口都有自己的平均值和标准偏差。 你会期望一条包括整个美国人口的单一正态曲线有一条肥大的右尾,因为犹太人口的平均智商(仍然)更高,而由于非裔美国人的平均智商低,左尾更肥。

    • 回复: @anonymous
  346. @JLK

    谢谢。 我期待跟进视觉空间。
    您的第一个标准杆无疑是正确的,但我试图接受某人的评论,即左撇子反种族主义者(或某些此类)会在很大程度上证明黑人有不同的 g。 我希望了解这在实践中可能意味着什么,以及什么可以算作证据。

  347. obwandiyag 说:
    @Colin Wright

    全是骗人的。 愚蠢的偏执狂只是头脑简单的谎言。 阅读一些真实的历史古生物学考古学。 哦,我忘了。 你不能。 太文盲了。

  348. utu 说:
    @JLK

    您是自己提出以下声明还是从某个地方提出来的?

    它解决了四个解决方案,而不是 一如均衡的智力测试总是为 g

    您意识到这是在乞求问题谬误或真正的苏格兰人谬误的某种变体。 这是没有意义的。 巧妙放置的副词“总是”使它听起来像是智能测试电池的真正专家所说的相关且深刻的事实信息。 你是自己想出来的,还是从爱因斯坦的 IQism 那里提出来的?

    除了找到一个主导因素外,没有“均衡智力测试”的定义或测试。 因此,当一组测试产生第二个强因素时,通过添加额外的测试或删除一些测试来调整电池,直到只找到一个主导因素,或者它被拒绝并标记为不平衡。 这就是单个 g 因素背后的全部谜团。

    我不知道你为什么要把十项全能带到这里来。 迪卡侬确实存在。 没有人在探索十项全能采用因子分析的定义。 它就是这样,它有三四个因素可以解释 70% 的方差,而这些因素可以解释为速度、力量等。 但是在智能的情况下,我们不知道它到底是什么。 我们正在摸索中。 心理学家正试图通过一系列测试来定义它。 问题是测试什么。 有测试电池组产生一个主导因素,但也有测试电池组产生两个或更多强因素。

    让我们回到十项全能,假设我们想找到最适合十项全能的运动员,假设我们测量候选人中因素分析发现的三个因素。 这会奏效,但如果由于某种原因在下一届奥运会上改变了得分,例如,在几代人之后强调速度而不是力量的比赛,不同的运动员将被十项全能吸引,并且基于他们的结果的因素分析将产生与以前不同的因素,即, 因素之间的比例会有所不同。

    最后'res'提出了一个稻草人:

    但真正令人震惊的(以及对“g 只是数学产物”阵营的谴责)是性能如何分解为 3-4 个大致同等重要的因素(如您所述)。

    我认为我们在这里争论的水平理所当然地认为协方差矩阵总是半定正的具有正特征值,其中一个必须是最大的。 然后从瑞利商的性质可以看出,在这个空间中所有可能的向量中,一个特征向量比任何其他的特征向量解释了更多的数据方差。 这个向量是 g 的最佳候选。 它不是人工制品,而是协方差矩阵的属性所必需的数学必然性。 所以 g 在这个意义上存在,因此它不是什么大问题,因为它必须存在,在这个意义上,它可以被认为是不屑一顾的人工制品。 为什么以不屑一顾的方式? 因为许多智商主义的爱因斯坦喜欢用 g 的存在作为证明他们自己的存在和活动的论据。

    这个最强的特征向量是 Spearman 和 Jensen 假设的方式,必须显着支配其他特征向量,因此可以忽略其他向量。 事实上,心理测量学家和心理学家设计的一系列测试通常就是这种情况。 正如我们已经说过的那样,不是因为测试电池“平衡良好”,这是一个毫无意义的概念,而是因为它们是这样设计的。 如果您通过偏向视觉想象的额外测试来加强测试电池,您将增加第二个特征值,然后提出关于两个因素而不仅仅是一个因素的问题。

    • 回复: @JLK
    , @res
    , @JLK
    , @JLK
  349. JLK 说:
    @utu

    除了找到一个主导因素外,没有“均衡智力测试”的定义或测试。

    我会假设你是一个真诚的真理寻求者,但要小心那些攻击“一个主导因素”的人的动机(g),尤其是那些在主流媒体中获得关注的人,因为他们的真正目标似乎是冻结现有的不合理的特权级别(如常春藤盟校代表),并隐藏过去在大学录取中偏向精英的证据测试系统。

    要了解偏见的根源,您必须回到 1940 年代中期,当时正在构建 SAT 以在全国范围内推广。 当时的精英们明白,我们正在与苏联进行长期的意识形态和经济竞争,苏联对工人阶级做出了平等主义承诺,即他们最聪明的孩子将与迄今为止的党内精英处于平等的竞争环境中至于大学录取问题。 这对苏联来说是一个很好的选择,它在经济上帮助了他们,因为他们的系统在将有能力的人安置在关键岗位方面做得更好。

    另一方面,主要是美国东北部的精英不希望他们的机构充斥着来自中美洲的资产阶级,他们是当时的“黄祸”。 Ivies 习惯于从大约 10-15 万东北黄蜂和大约 3.5 万东北犹太人中抽取大部分学生。 仅中西部就有四倍多具有大致相当的本土能力的人,加利福尼亚和南部还有更多。

    我们都不是真正的心理测试专家,但这里有一点常识。 如果一个人试图识别具有内在才能的人,他或她将努力确保结果尽可能少地受到环境因素的污染,例如测试对象是富裕还是贫穷。

    这让我们想到了词汇。 测试对象在词汇测试中无法识别单词的可能原因有两个:(1)他或她的语言记忆力比正确回答问题的人差; 或 (2) 他或她一开始就没有遇到过这个词。 原因(2)与环境有关,与能力无关。 尽管如此,词汇还是从一开始就成为 SAT 的支柱,让可以使用飞利浦安多弗学校图书馆和 “纽约时报” 在大学入学考试中比爱荷华小镇的人高出一大截,如果他们的父母收到了 读者文摘 每月一次。 当然,大多数有色人种的信息环境比爱荷华州的孩子还要糟糕。

    爱荷华州的一个农场男孩与菲利普斯·安多弗的孩子在环境方面的地位要高得多,因为他们俩都被给予了现代长篇斯坦福-比奈或韦克斯勒智商测试中发现的令人惊讶的视觉空间类型问题,但有在 PSAT/NMSQT 或 SAT 上从来没有这样的问题。

    主流媒体如 “纽约时报” 在 80 年代,当他们同时向我们保证词汇是智力的最佳单一代表时,他们口中的双方都在说,同时对大学入学考试在文化上对黑人有偏见感到遗憾。 这是鳄鱼的眼泪,而精英们则确保他们的面包涂上黄油。

    他们也不能令人信服地声称自己无知,因为在科学文献和其他充满活力的智力测试中,有很多迹象表明,Deary/Haier 图表中显示的词汇和智商之间相对较弱的关联已经为人所知了几十年,相当就像回到 SAT 刚开始的时候。 事实上,我看过 Jensen 和其他人的几篇出版物,其中空间和 g 之间的高度相关性以及词汇与 g 的较低相关性是在推理中表达的,但他们实际上是在踮着脚尖说。

    科学依赖于寻找资源,它并不存在于政治真空中。 即使是政治不正确的科学也不能免受政治和精英影响。

    这会告诉你所有你需要知道的事情 “纽约时报” 加入塔勒布的攻击 g. g 可能并不完美,但它是一个非常有效的废话检测器 事后 PSAT/SAT 的亲精英偏见的理由,本质上是试图说服计算的人在看迪里/海尔图表时不要相信他们撒谎的眼睛。

    正如另一位海报所建议的那样,如果您不喜欢 g,自己算算,然后像 Spearman 和 Jensen 那样提交给同行评审。 面对相当大的政治逆风,他们的作品幸免于难,因此它们是我们目前拥有的最佳模型。

    • 回复: @DFH
  350. DFH 说:
    @JLK

    原因(2)与环境有关,与能力无关。 尽管如此,词汇从一开始就成为了 SAT 的支柱,让可以使用飞利浦安多弗学校图书馆和在房子周围的纽约时报的孩子们在大学入学考试中比爱荷华州小镇的孩子们有很大的优势,如果他们的父母每月收到一次《读者文摘》,他们就很幸运了。 当然,大多数有色人种的信息环境比爱荷华州的孩子还要糟糕。

    但是词汇量的黑白分数差距比其他分测验小

    • 回复: @JLK
    , @JLK
  351. res 说:
    @utu

    最后'res'提出了一个稻草人:

    但真正令人震惊的(以及对“g 只是数学产物”阵营的谴责)是性能如何分解为 3-4 个大致同等重要的因素(如您所述)。

    不是稻草人。 投影的另一个例子?

    我们 g,统计神话

    一个样品:

    因子分析对于汇总数据很方便,但不能告诉我们相关性来自何处; 它总是说,只要只有正相关,就会有一个普遍因素。 g 的出现是该相关结构的微不足道的反映。

    顺便说一句,我似乎记得你刚才争论过这个“稻草人”。 至少你有学习的能力。

    需要明确的是,重要的一点是由 g 解释的方差量与随机数据或真实数据(如十项全能)的第一个主成分中出现的方差量相比。

    • 回复: @utu
  352. anonymous[417]• 免责声明 说:
    @Wizard of Oz

    我使用“设计”的意思是“试图为更有天赋的人安排经济机会”——这会因代而异,我想我们都同意。 我并不是想暗示一个比这更雄心勃勃的设计。

    至于“肥尾”——我的意思是指在图表中观察到的那些“肥尾”,其中“g”是由可用的 IQ 测试沿一个轴衡量,“经济成功”是由工资和累积等粗略指标衡量的另一条轴线上的财富。

  353. JLK 说:
    @DFH

    但是词汇量的黑白分数差距比其他分测验小

    词汇测试往往会歧视穷人,无论他们是在南布朗克斯、巴里奥、保留地、唐人街还是在阿巴拉契亚农村。 您不太可能看到《哈珀斯》杂志的所有家庭。

    另一方面,正如您所指出的,黑人智力概况往往像犹太人概况一样,以言语为主。 只是它们之间的距离超过了一个标准差。 过度使用词汇可能会略微提高黑人分数,但它会大大提高犹太人的分数,并且会伤害相对于富人和中产阶级的所有其他穷人群体。

    • 回复: @James Thompson
  354. JLK 说:
    @utu

    除了找到一个主导因素外,没有“均衡智力测试”的定义或测试。 因此,当一组测试产生第二个强因素时,通过添加额外的测试或删除一些测试来调整电池,直到只找到一个主导因素,或者它被拒绝并标记为不平衡。 这就是单个 g 因素背后的全部谜团。

    您之前曾写道,您对哪些因素更重要或为什么更重要没有意见。 那么,什么是您认为被压制以支持单数的“第二强因素” g?

    命名它,以便我们评估您的论点。

  355. Joe Mack 说:

    没有君王,只有耶稣。
    塔利卜在很多方面可能是对的,有很多见解,但他不是耶稣来到地球。

  356. @CanSpeccy

    根据维基百科,这里是如何 g 或一般智力估计:

    g 因素……是一个总结不同认知任务之间正相关的变量……基于许多测试的综合分数(“智商分数”)通常被认为是对个人在 g 因子。

    对于任何具有生物学意识的人来说,个人测试的分数必然在某种程度上必然相关,这一点是不言而喻的。 所有形式的心理功能都依赖于神经元的活动。 因此,任何损害神经元功能的情况,无论是在细胞还是有机体水平上,都会对所有智力功能产生负面影响,即 g 因子。

    在细胞水平上,数十种基因决定的代谢异常会损害神经元的性能,环境毒素和营养缺乏也会如此。 同样,机体水平的多种条件、心血管健康以及大脑的氧气供应等都会影响整体心理功能。 这些因素可能在很大程度上解释了撒哈拉以南非洲人和欧洲人之间平均人口智商的差异:非洲人更严重地受到脑损伤疾病(例如疟疾)和营养缺乏(例如铁)和饮食摄入有毒重金属。

    但是大脑不像 PC 那样工作,所有的操作都在这里,因此 g 因素,取决于一个中央处理单元。 大脑 是模块化的:具有专门处理不同类型的感觉输入和提供不同类型的行为输出的区域。 这意味着不同心理功能之间的完美关联是最不可能的,每个大脑模块都存在独立的遗传变异,并且在不同程度上易受环境毒素、疾病或饮食不足的影响。

    此外,估计值 g 因素似乎完全是任意的,因为它们取决于哪些测试为分析提供了基础。 如果一个人超出了通常首选的测试,则会看到个人能力的巨大差异。 例如,在音乐能力和智商测试之间 相关性是微不足道的. 对相关文献的粗略研究表明,智商和艺术能力之间的相关性基本上为零。 因此,通过对人类思维范围采取更现实的看法, g 因素肯定会减少到完全无足轻重的地步。

  357. JLK 说:
    @James Thompson

    不确定哈珀斯杂志是否有你想象的效果。

    正如你可能想象的那样,我有点轻率。 我一直专注于美国大学入学考试,这些考试在 16-17 岁左右进行。 重要的是整个童年的信息和营养环境。 本刊物 将 WISC-R 分量表的词汇部分的遗传力设为 0.72,这允许基于环境影响的高达 0.28 的变化。 这不是微不足道的。

    顺便问一下,您对迪里/海尔相关性的可靠性有何评价? 我注意到您在之前的帖子中引用了迪里。

  358. JLK 说:
    @utu

    因此,当一组测试产生第二个强因素时,通过添加额外的测试或删除一些测试来调整电池,直到只找到一个主导因素,或者它被拒绝并标记为不平衡。 这就是单个 g 因素背后的全部谜团。

    实际上,我为您找到了一项研究,该研究得出的结论是双因素模型(空间与 g 分离)比奇异 g 理论更适合。 右上角,第 2 页。

    https://www.researchgate.net/publication/313885447_Phenotypic_and_genetic_evidence_for_a_unifactorial_structure_of_spatial_abilities

    CFA 还用于测试 g 的空间能力的独特性,由语言和非语言能力测量(方法)索引。 正如预期的那样,空间能力与 g 有相当大的重叠(可能是由与 Raven'sMatrices 的大量重叠驱动)(24),但双因素模型(空间能力和 g;SIAppendix,图 S2B)比一个更好地拟合数据-因子模型(g;SI 附录,图 S2A),表明空间能力与其他认知能力不同。

    有趣。

    • 回复: @res
    , @utu
  359. res 说:
    @JLK

    谢谢你的链接。 我在解释它们的空间因子与 g 的关系时遇到了一些麻烦。 如果我理解正确,图 S2 与该问题最相关。

    面板 S2b 表明 g 与其空间因子具有 0.79 的相关性。 他们的拟合测量(例如 AIC、BIC)表明双因素模型更好,但对我来说,对单因素模型的改进看起来很小。 他们似乎刻意避免量化第二个因素造成的差异。 考虑到我会用来查看的两种技术(ANOVA 和碎石图)在本文中用于其他目的,这对我来说似乎很奇怪。

    在我看来,它们的空间因素与 g 太纠缠不清,无法作为一个单独的概念使用。 我认为在这两个因素上强制正交并查看第二个因素的相对解释力会更好。 有什么替代的观点吗?

    这篇 2018 年的评论论文对我来说很有用。 有什么想法吗?
    空间能力的启发式框架:空间因素文献的回顾与综合,以支持其转化为 STEM 教育
    https://link.springer.com/article/10.1007/s10648-018-9432-z

    • 回复: @JLK
  360. JLK 说:
    @res

    谢谢你的链接。 我在解释它们的空间因子与 g 的关系时遇到了一些麻烦。 如果我理解正确,图 S2 与该问题最相关。 面板 S2b 表明 g 与其空间因子具有 0.79 的相关性。 他们的拟合测量(例如 AIC、BIC)表明双因素模型更好,但对我来说,对单因素模型的改进看起来很小。 他们似乎刻意避免量化第二个因素造成的差异。 考虑到我会用来查看的两种技术(ANOVA 和碎石图)在本文中用于其他目的,这对我来说似乎很奇怪。

    在我看来,它们的空间因素与 g 太纠缠不清,无法作为一个单独的概念使用。 我认为在这两个因素上强制正交并查看第二个因素的相对解释力会更好。 有什么替代的观点吗?

    图 S2 似乎是关于单数问题的非常有趣的证据 g,这可能有助于 utu 的观点。 什么时候 g 使用八个非语言的、主要是空间的子测试和两个语言的子测试来计算,Verbal 与 g 从迪里的 0.7+ 下降到仅 0.36。 他们看起来完全不同 gs.

    也许是迪里的 g 是典型的荷兰人 g,随之而来的是有特色的黑人、犹太人、亚洲人、美洲原住民和墨西哥人 gs。 这可能会引发关于哪个 g 在设计像下一版斯坦福-比奈或 WAIS 这样的长篇 IQ 测试时,应该加载一个,以及推定的相关性是否 g 长寿和各种形式的生活成功会因种族而异。

    詹森发表 本刊 几十年前,在检验“斯皮尔曼假说”时。 斯皮尔曼早些时候假设黑白智力差异在具有最高 g 负荷的子测试中最为明显。 Jensen 声称已经证实了这一点,但魔鬼在细节中,而且黑人比空间人更善于表达的一般结论似乎在测试中并没有得到一致的支持。 Jensen 的各种子测试的 g 相关性也表明,仅基于我的快速观察,他可能一直在使用与 Deary 不同的 g 范数。

    有一位已发表的专家扮演逆向投资者而不是像古尔德、塔勒布和霍华德加德纳这样的人会很好,无论他们是否有意为之,都可以作为稻草人被推到一边。

    • 回复: @JLK
    , @res
    , @utu
  361. JLK 说:
    @JLK

    也许Deary的g是典型的荷兰g,

    记录更正:Deary's g 是基于 Salthouse 研究,该研究测试了 7000 名受试者,可能是美国人。 我在重复 utu 写的关于荷兰学员的事情,这似乎是一个错误。

    • 回复: @utu
  362. res 说:
    @JLK

    当使用八个非语言的、主要是空间的子测试和两个语言的子测试计算 g 时,Verbal 与 g 的相关性从迪里的 0.7+ 下降到仅 0.36。 它们看起来像完全不同的gs。

    好点子。 我没有意识到这一点。 它还反对我关于它们的空间因子与 g 过度相关的观点(如果以更传统的方式计算)。

    Jensen 链接的表 2(第 11/203 页)很有趣。 它与空间推理的 g 相关性为 0.31,B/W 差异仅为 0.19 SD。 但真正引起我注意的是注释:“该分析中使用了来自较大样本(黑色或白色)的 g 载荷。” 如果计算出的 g 在人群之间没有变化,为什么还要为此烦恼呢?

    对我来说,计算出的第一个 PC 应该对总体和子测试的组成敏感,但我在思考其中的含义时遇到了麻烦。 面对这种情况,我们如何将 g 提取为子测试/总体不变量?

    也许考虑一般智力功能因素(giff)的一种方式是作为特定人群特征的乘数? 所以我们会有:
    – 具有长度和方向的群体特定剖面 (psp) 矢量。 长度反映了群体能力平均差异的大小。 M/S/V/等的方向。 平衡差异。 方向可能变化不大,但我认为足够重要。
    – 作为标量的 giff(比如归一化为 Z 分数),它乘以 psp。
    – 具有长度和方向的个人特定轮廓 (isp) 矢量,它捕获个人完整轮廓与其投影到适当 psp 线上的差异。

    因此,个人的能力概况将分解为:
    psp * gif + isp
    giff 大概是我们所知道的 g 的类比。

    这并没有考虑到变异的群体差异(即 giff SD 可能在群体之间有所不同)。 我认为空间的每个轴都应该进行 Z 评分,但不确定使用哪个组作为参考。

    对于实际使用此构造可能最好同时使用完整的子测试空间并减少到 2 或 3 台 PC 以进行可视化。 使用一组作为现有流行测试组合的超集的子测试,可以有趣地观察测试偏差(例如,您的 PSAT 将口头评论加倍)以及它们如何与组特征保持一致。

    有什么想法吗? 我在杂草丛中吗?

    • 回复: @JLK
  363. utu 说:
    @JLK

    亲爱的仅引用:

    (110) Johnson, W., te Nijenhuis, J. & Bouchard, TJ 仍然只有一克:五个测试电池的一致结果。 情报 32, 81–95 (2008)。

    (看) http://emilkirkegaard.dk/en/wp-content/uploads/Just-one-g-Consistent-results-from-three-test-batteries.pdf

    对于索赔:

    “只要测试电池相当多样化,来自不同测试电池的 g 因子几乎完全相关 (110). 也就是说,只要一个人进行了足够多的测试,一组测试的一般智力因素将与任何其他测试组的一般智力因素的排名非常接近。”

    你注意到他做了同样错误的(同义反复的)错误谓词“合理多样化”,就像你用“始终如一”强化“平衡良好”所做的那样。 请记住,从 Cattell 电池派生的 g 相关性不好,因此它必须(总是)意味着它不是“合理多样化”且不是“平衡良好”。

    无论如何,约翰逊使用的数据:

    “我们利用了 de Wolff 和 Buiten (46) 发表的 1963 项心理能力测试的数据矩阵。 该矩阵在此处作为附录 A 重印。我们无权访问任何类型的个人参与者数据。 矩阵所基于的样本包括 500 荷兰皇家海军的专业海员。 ”

  364. JLK 说:
    @res

    有什么想法吗? 我在杂草丛中吗?

    我对数学不可知论,因为(1)它并不是真正在我目前的工具集中,离大学这么远,没有付出更多的努力; (2) 就设计最有效的能力倾向测试而言,通往罗马的道路可能不止一条。 是否使用统计单轴标准 g 这是通过 FA/PCA、双因素解决方案或十项全能类型(SAT 所做的那种,语言过载)评分系统计算的,首先需要识别显着组件(例如 Deary 的顶部推理,空间和词汇三个)并决定如何分别对它们进行加权。

    单数 g 与十项全能的辩论与加权过程有关。 某些 g 支持者似乎认为在加权时只需要合理的多样性来保证准确性。 就个人而言,我会尝试获取更多关于 R、S 和 V 与广泛专业领域能力之间关系的数据,并根据重要性设计更精确地加权它们的测试。

    真正的问题是政治对科学的有害影响。 就未来国家竞争力而言,在所有领域都面临着太多的风险,继续允许这些关键的把关机制被秘密地分配和操纵,并通过涓滴心理测量赞助计划进行操纵。 这个过程需要透明,为了国家的利益而计算,并掌握在严格公正的人手中。 平权行动应该是一个完全独立的政策辩论。 不要搞乱测试。

    我不是中国体制的完全拥护者,但几周前我读到了最有前途和最雄心勃勃的基层党员是如何被送到农村作为培训的一部分的。 除非他们以切实的方式让村民的生活变得更好,否则他们不会回来或升职。 我们可以使用这样一个高层次、有原则的僧侣阶层,宣誓为公共服务和正直。 当他们完成心理测量系统后,我们可以派他们去监督华尔街。

    • 回复: @utu
  365. utu 说:
    @res

    是的,我正在学习。 你可以以我为榜样。 三年前左右,我对整个主题一无所知,对此没有任何意见或偏见。 直到在这里进入 UR 后,我才遇到说 IQism 语言的人,他们不加思索地大喊基于物化的重言式,这些物化清楚地表明基于一些认识论错误和灌输的邪教行为。 我想深入了解并回答这个问题,否则聪明和受过教育的人(比如你自己)是如何陷入其中的。 我对它产生了兴趣,从那以后我学到了很多东西。 请记住,我从不反对对智力的研究,无论它是什么,或者它具有很强的遗传成分。 我对邪教之类的团体怀有敌意,不幸的是,许多白痴和雅虎都接受并继续传播。

    “该相关结构的微不足道的反映”或“人工制品”是有效的修辞上的反对意见,因为 IQ 主义者已经具体化了 g,而它只是数学上不可避免的必然性,而且它在其他特征向量上占主导地位,这只是矩阵的偶然事件他们使用的测试,或者可能通过调整测试来抑制第二个特征值。 当他们进行的一系列测试产生的第二个特征值太强以至于他们不喜欢时,他们会认为该电池不够“平衡良好”。 因此,他们将永远在 Spearman 和 Jensen 施放的魔法咒语的恶性循环中跳舞。 邪教行为的临床案例。

    早在 1920 年代。 Godfrey Thomson 提出了类似的反对意见,称称为 g 的支配向量并不是一个大概念,因为具有 g 属性的实体出现在随机生成的数据中:

    http://www.aceintelligence.com/detailed_history_of_iq.php
    英国统计学家 Godfrey Thomson 在 1920 年代证明,即使是用掷骰子随机产生的分数也可以产生类似于 Spearman 使用的相关系数的分层模式(Fancher,1985),从而引发了一场持续至今的争论。

    • 回复: @Wizard of Oz
  366. utu 说:
    @JLK

    接得好。 感谢您的链接。 我发现有两篇论文挑战单 g:

    http://www.iapsych.com/articles/castejon2010.pdf
    项目范围活动的验证性因素分析。 二阶 g 因子还是多元智能?

    总体而言,结果表明,谱活动不像多元智能理论的捍卫者所提出的那样与 g 分离,也不像 g 因子模型的捍卫者所主张的那样单一。

    非 g 因素预测教育和职业标准:超过 g
    https://www.mdpi.com/2079-3200/6/3/43

    如下所述,我对非 g 因素的研究对 g 假设的首要性提出了质疑,该假设假设 g 解释了认知测试的预测能力,并且非 g 因素的预测能力可以忽略不计。

    关键结果是 SAT 和 ACT 的 non-g 残差对大学 GPA 的预测几乎与 g 对大学 GPA 的预测一样(βs ≈ 0.30)。2 结果与 g 假设的首要性不一致,该假设假设 non-g因素的预测能力可以忽略不计

  367. utu 说:
    @JLK

    “就个人而言,我会尝试获取更多关于 R、S 和 V 与广泛专业领域能力之间关系的数据,并根据重要性设计更精确地加权它们的测试。”

    我相信三个分数比一个加权分数要好,因为权重取决于效用。 你想如何利用人类的材料? 哪些牛会产奶,哪些牛更适合生产小牛肉? 一所需要律师的学校将使用与需要机械工程师的学校不同的权重。

    我同意应该做更多的研究来研究心理能力的许多方面。 但是 IQists 已经陷入了 Spearman 和 Jensen 的教条。 教条是进步的障碍。 停滞也可能来自这样一个事实,即种族主义者很高兴看到黑人失望而犹太人看到自己站起来,而情况可能更复杂,黑人在口头智力方面要好得多,而犹太人在空间智力方面比想象的要差得多.

    • 回复: @JLK
  368. utu 说:
    @JLK

    有一位已发表的专家扮演逆向投资者而不是像古尔德、塔勒布和霍华德加德纳这样的人会很好,无论他们是否有意为之,都可以作为稻草人被推到一边。

    很好的一点。 他们三个都是很好说话的人。 他们提出的某些观点是好的,但他们没有深入细节,他们错过了一些重要的观点。 为了显着削弱 IQists 构建的结构,我们需要具有高 SQ(Sitzfleisch 商)和显着数学技能的人。 这将需要时间、金钱和勇气,因为无论谁这样做,最终都会树敌众多。

    (1) 应攻击因子分析方面。 这是一种炼金术,类似于基于配方的练习,隐藏在数学程序背后,每个程序本身在数学上都是有效的,但作为一个整体,它可能(我相信在某些情况下是)骗子和骗子。

    (2) 应从怀疑者或有相反偏见的人的角度重新分析特定的研究和数据。

  369. JLK 说:
    @utu

    我相信三个分数比一个加权分数要好,因为权重取决于效用。 你想如何利用人类的材料? 哪些牛会产奶,哪些牛更适合生产小牛肉? 一所需要律师的学校将使用与需要机械工程师的学校不同的权重。

    我倾向于同意你的观点,尤其是对于本科入学考试。 为相关专业设置 LSAT、MCAT 等的样式是有意义的。

    我同意应该做更多的研究来研究心理能力的许多方面。 但是 IQists 已经陷入了 Spearman 和 Jensen 的教条。 教条是进步的障碍。 停滞也可能来自这样一个事实,即种族主义者很高兴看到黑人失望而犹太人看到自己站起来,而情况可能更复杂,黑人在口头智力方面要好得多,而犹太人在空间智力方面比想象的要差得多.

    这是关于个人,而不是群体。 我在这个评论线程中提到种族群体的唯一原因是指出,如果测试是为了提高其他群体的表现,一些群体(如美洲原住民和墨西哥人)会有意或无意地受到伤害。

    这个系统对我来说非常好,我现在已经退休了,所以我唯一能参与战斗的就是渴望一个有竞争力和公平的美国。

    • 回复: @utu
  370. @utu

    “物化”对推进对话并没有真正的帮助。 这可能只是一种侮辱,因为它暗示有人将地图误认为是领土,但即使在维基百科关于其不同应用的许多解释性文件中将物化视为一种谬误,也留下了模棱两可的空间。 你能告诉我们你认为史蒂夫·塞勒、彼得·弗罗斯特、格雷格·科克伦或约翰·德比希尔使用“g”或“IQ”的方式是错误的、完全错误的或荒谬的吗?

    这个常识怎么样? 至少一些大学课程需要通常被认为是智力的东西。 以法律为例。 因此,您想找到一种快速简单的方法来确定某些潜在学生是否有足够的智力资源来圆满完成学业。 凭借大量的直觉和大量的反复试验,您将一系列看似可能的测试组合在一起,并通过参考应届毕业生的表现来验证它们。 您发现您从数学上提取的 ag 用于对毕业生进行排名,与任何一项测试一样或更好。 此后,您使用 g 测试至少提供申请者必须跨越的门槛。 然后你会找到一个简化的、更短的测试组,它会给你几乎相同的 g 分数。 从这一点上,您会发现 g 与认知能力很重要的各种其他理想结果的相关性如何。 所以你承认 g 和 IQ 在一定程度上是有用的,而不必为头脑简单的具体化而道歉。

    • 回复: @utu
  371. utu 说:
    @Wizard of Oz

    “物化”对推进对话并没有真正的帮助。 这可能只是一种侮辱,如暗示 有人把地图误认为是领土 但即使在维基百科关于其不同应用的许多解释性文件中将物化视为一种谬误,也留下了模棱两可的空间。 你能带我们过去吗 你认为是错误的、完全错误的或荒谬的 例如,史蒂夫·塞勒、彼得·弗罗斯特、格雷格·科克伦或约翰·德比希尔使用“g”或“IQ”的方式?

    既然你提到了 Korzybski,我们就和他在一起吧。 任何时候你的英雄使用构装体 智商是…… 他们犯了物化罪

    ”(来自 Wiki on Korzybski)请注意,这个前提使用了短语“is not”,这是“to be”的一种形式; 这个和许多其他例子表明,他 [Korzybski] 并不打算放弃“存在”本身。 事实上,他 [Korzybski] 明确表示,动词“to be”在用作助动词或用于表示存在或位置时没有结构问题。 有时甚至可以使用动词“to be”的错误形式,只要一个是 意识到它们的结构限制设立的区域办事处外,我们在美国也开设了办事处,以便我们为当地客户提供更多的支持。“

    因为我怀疑他们是否“意识到结构限制”的形式 “智商是” 他们使用。 他们不使用“智商是……”的意思 “他的 GPA 是……” or “他的打击率是……” 开始他们知道,如果一个学生更加努力,他的 GPA 可能会更好,或者如果棒球运动员的对手不那么具有挑战性,他的击球率就会更高。 但是在 IQ 或 g 的情况下,他们给了他们不同的本体论属性,这是没有根据的。

    可以询问您上次参加考试时的 IQ 分数是多少以及您参加的考试是什么。 还要记住,测试和重新测试的相关性为 0.9 水平,这意味着两次测试之间的 ±1 sigma 差异为 ±6.7 IQ 点。 所以在简单的经验水平上 他的智商... 至少是非常不准确的。

    显然你被误导了:

    您发现您从数学上提取的 ag 用于对毕业生进行排名,与任何一项测试一样或更好。

    如果你给学生 T_1,..,T_n 测试,心理测量学家将根据 Jensen 的秘方从中推导出 g,那么这个 g 将不是 GPA 等的最佳预测指标。 T_i 与 g 不同的另一个线性组合将预测 GPA 优于 g。 更复杂的是,对于加州理工学院和乔治城的学生来说,最大化与他们的 GPA 拟合的系数将是不同的。

    在这里,您正在恳求实用价值:

    所以你承认 g 和 IQ 在一定程度上是有用的,而不必为头脑简单的具体化而道歉。

    关键短语是“达到一定程度”,因为智商的预测能力并不像你的英雄让你相信的那么高。 此外,我不认为 g 被报告在可能从中提取它的测试中。 您无法从 Raven 矩阵测试中提取 g。 顺便说一句,你知道你孩子的分数吗?

    g的全部目的是意识形态的。 部分原因是还原论和物理学嫉妒,然后是自我实现预言作为测试的效果,正如你所说,它是用“大量直觉和大量反复试验”设计的,其中在试验中出现错误部分是测试在第二个特征向量上太强的被淘汰了。

    • 回复: @Wizard of Oz
  372. utu 说:
    @JLK

    这是关于个人,而不是群体。

    我从你那里捡来的。 您在论点中使用了组。

    • 回复: @JLK
  373. JLK 说:
    @utu

    您在论点中使用了组。

    为了评估是否存在对群体的歧视,没有办法绕过参考群体的需要。

    如果你给学生 T_1,..,T_n 测试,心理测量学家将根据 Jensen 的秘方从中推导出 g,那么这个 g 将不是 GPA 等的最佳预测指标。 T_i 与 g 不同的另一个线性组合将预测 GPA 优于 g。 更复杂的是,对于加州理工学院和乔治城的学生来说,最大化与他们的 GPA 拟合的系数将是不同的。

    我与学术八卦有关。 她(我假设)指出,SAT 写作测试比 SAT-V 或 SAT-M 更能预测伯克利大学第一年的 GPA。 我质疑他们是否控制了课程难度。 我随后读到了另一项研究,他们正在对此进行纠正,但我怀疑还有其他重要的变量仍未得到控制。 你如何控制许多在 SAT 考试中获得高分的高中神童在被精英学校中具有类似能力的学生中学习之前从未需要发展学习技能的可能性? 一直自律学习的苦行者无疑会在一段时间内占据相对优势。

    • 回复: @utu
  374. @utu

    当您以“自从您提出 Korzybski 以来”开始您的回应时,很难将其视为对讨论的认真贡献,而且我没有提到他,无论他是谁。

    我注意到您批评 g 不是 GPA 的最佳“预测器”。 那么,如果它是有效有用的,除非您可以提供可比效率的更好替代品——即使目的实际上是预测 GPA,这不太可能?

    “g的全部目的是意识形态的”。 纯粹的咆哮。 无意义的。 我有一个关于如何以及为什么开发和使用 g 的简化版本,它与意识形态无关,你甚至没有试图质疑它并证明我所描述的确实是意识形态的。

    • 回复: @utu
  375. @Curmudgeon

    “我对他们中的许多人对他们专业领域之外的现实世界知之甚少感到惊讶。”

    当苏格拉底开始询问周围那些吹嘘自己的知识和传授智慧的能力的人时,他所观察到的正是如此。

    他发现,他们对某一特定领域的掌握会导致过度自信,从而误导他们错误地认为自己无所不知。 而且,这种过度自信的后果比这个人一开始就没有根据他假设的知识采取行动的后果更糟。

    由于对自己无所不知的过分自信蒙蔽了双眼,他们误入了自己造成的危机,这比他们假装什么都不知道还要糟糕。 与塔勒布的黑天鹅论文没有太大不同。

  376. utu 说:
    @JLK

    人们会期望大学会几乎实时(或至少每年一次)对 SAT 及其子部分的预测能力进行评估,并且会有大量数据浮动。 最有可能的原因是我们没有看到它,并且没有在公共空间印上表明预测质量的具体数字(学术八卦是具体的吗?)是因为这些数字无处不在。 这让人想知道 SAT 及其子部分的真正用途是什么。

    “不得不自律学习的笨蛋”——显然,Sitzfleisch 商的作用比 IQists 愿意承认的要大得多。

    • 回复: @utu
  377. utu 说:
    @utu

    我在某处看到 HS 的 GPA 比 SAT 更能预测大学的 GPA。 由于 Sitzfleisch 商数,这是有道理的。

    以下是一些与斯皮尔曼收益递减定律相矛盾的研究:

    SAT 更好地预测高能力科目的 GPA:对斯皮尔曼收益递减定律的影响
    https://www.ncbi.nlm.nih.gov/pmc/articles/PMC3090148/

    高能力科目的 SAT 与 GPA 的相关性高于低能力科目。 两组的 SAT g 负荷(即 SAT 与 g 的相关性)是等效的。 这是第一项表明 SAT 的预测有效性因 g 不同的能力组而异的研究。 结果与基于斯皮尔曼收益递减定律的假设相矛盾,即对于高能力科目的测试的预测效度应该较低。 需要进一步的研究来确定有助于 SAT 对 g 不同的组的预测有效性的因素。

  378. res 说:

    这可能很有趣: 斯皮尔曼收益递减定律:统计神器?
    https://www.sciencedirect.com/science/article/pii/S0160289613000809

    我试图确定 SAT/GPA 的发现与 SLODR 有多大的相反: https://en.wikipedia.org/wiki/G_factor_(psychometrics)#Spearman’s_law_of_diminishing_returns

    我认为这部分是:“SAT g 载荷(即 SAT 与 g 的相关性)对于两组来说是相等的。”
    但另一部分:“SAT 与 GPA 的相关性对于高能力科目高于低能力科目”似乎正在解决一些不同的问题。

    后者基于 Sitzfleisch 商非常有意义。 总体而言,高能力对象更依赖智力,即使他们在一个特定领域的挖掘可能比另一个领域更多。

    尽管 SAT(或 g、IQ 测试等)作为衡量能力的指标并不完美,但 IMO GPA 却差得多。
    – 科目难度。
    – 学校和科目之间的标准差异很大。
    - 随着时间的推移而变化。
    – 等级膨胀压缩可用规模。
    – 在更主观的课程中可能存在非学术偏见。

    Steve Hsu 在俄勒冈大学发表了一些讨论 GPA/SAT 的好博文。 例如: https://infoproc.blogspot.com/2010/04/dating-mining-university.html
    在他的博客中搜索“Schombert”了解更多信息。

    这些论文有一些关于 SAT/GPA/课程难度的好数据。

    评估学生表现的传统 GPA 的替代方案
    https://projecteuclid.org/download/pdf_1/euclid.ss/1030037959

    抽象。 针对美国本科院校日益严重的成绩膨胀问题,讨论了 GPA 和基于 GPA 的学生评估的替代方案。 基于贝叶斯潜在特征公式的另一种总结消除了与基于 GPA 的措施相关的许多不公平现象,并已被提议作为杜克大学基于 GPA 的班级排名的替代品

    调整 GPA 以反映课程难度
    https://www.researchgate.net/publication/241813085_Adjusting_GPA_to_Reflect_Course_Difficulty

    抽象
    研究生平均分 (GPA) 的计算错误地假设课程和教师的成绩具有可比性。 GPA 夸大了选择“更简单”课程的学生相对于选择“更难”课程的学生的表现。 本文提出了一种调整 GPA 的方法,并将其应用于卡内基梅隆大学一组本科生的数据。 调整后的 GPA 与学生的高中平均绩点和 SAT 分数的相关性高于原始 GPA 或使用突出的替代方法调整的 GPA,即项目反应理论。 一项对学生的调查发现,新方法对相对课程难度的估计与学生对相对课程难度的看法是一致的。

    这篇 2017 年的文章对该研究进行了简短的讨论: https://www.1843magazine.com/ideas/the-daily/the-extraordinary-silliness-of-american-college-grading

    PS 我认为在大多数情况下,范围限制比 SLODR 更成问题。

  379. JLK 说:
    @DFH

    但是词汇量的黑白分数差距比其他分测验小

    我想更新我的答案。 我过去读过的大部分内容都表明您的陈述是正确的,包括 詹森写的这封公开信:

    黑人在词汇、一般信息和语言理解的文化负载分测验中比在诸如积木设计之类的非语言表现测试中做得更好。 东方人、墨西哥裔美国人、印第安人和波多黎各人等少数族裔的情况恰恰相反。

    基于这样的陈述,我的帖子 #315 将黑人确定为一个群体,他们可能会受到 PSAT/SAT 的双重加权口头、一种数学、无空间格式的帮助。

    然而, Jensen 写的这篇实际的科学论文 似乎另有说明,特别是如表 2 所示,词汇测试、SAT-Verbal 和算术测试的黑白差异远大于列出的空间测试。 另外,看一下相关性 g 在 Jensen 论文中用于各种空间、算术和语言测试,看起来 Jensen 的 gg 这是由迪里根据 Salthouse 研究计算得出的。

    基于这种有限的研究比较,Deary g,这似乎比 Jensen 更重视空间智能 g,实际上对黑人来说比 Jensen 看起来更公平 g.

    因此,PSAT/SAT 形式可能一直以来对黑人的歧视性比一般认为的要大,言语部分的双重权重是伤害而不是帮助。 重新加权他们以拥有同样重要的语言、数学和空间部分实际上可能会帮助黑人以及亚洲人、美洲原住民、墨西哥人。 男性和未来的 STEM 学生。

    Jensen 研究可能是一个异常值,我的观察无疑是简单化的,因为某些类型的空间测试比其他类型的空间测试更具概念性。 但 Jensen 研究的表 2 似乎与他的陈述不同。

    Haier/Deary/Salthouse 研究、Jensen 研究和 里姆菲尔德等人。 学习 发现双因素解决方案优于单因素解决方案 g 解决方案似乎都提出了合理的问题,即大学招生和智商测试是否已经在詹森式上合理化了 g 对不同的少数族裔公平。

  380. 如果您要使用统计数据,请使用适当的方法并通过各种方式正确应用它们。 但似乎几乎不需要困难的数学分析就可以得出结论,智商测试仅衡量进行智商测试的能力,或者几乎相同的事情,在严重依赖有限范围的口头和智商测试中的数值运算。

    当然,鉴于学校教育提高智商的证据,智商测试在所有认知领域测量一些与生俱来的和不可改变的心理能力的想法似乎是荒谬的。 例如,这里显示 当 1960 年代挪威的义务教育年限从 19 年增加到 XNUMX 年时,XNUMX 岁时测量的学生的智商每年增加大约 XNUMX 个百分点。

    假设教育对智商的影响是递减的,这样的证据似乎毫无疑问地证明,过去 20 年美国人平均智商增加 100 点可以很大程度上归因于持续时间的增加。上学的。 同样,似乎可以合理地推断,如果撒哈拉以南非洲的平均教育年限从目前的大约 12 年增加到北美标准的 24 到 XNUMX 年,那么撒哈拉非洲人将被发现与美国白人一样天生聪明,甚至更多。 也许比华裔美国人或美国犹太人更甚。

    不应过分依赖单一的研究结果。 然而,许多研究发现,有 智商和音乐能力之间的相关性基本上为零, 依赖于许多复杂的心理过程的教师, 支持这样的观点, 即智商仅衡量在西方教育修改的有限任务范围内的先天能力, 绝不是一个有效的能力衡量标准。测试直接测量的认知过程范围有限。

    通过掩盖智商测试结果只不过是学术成就的代表这一事实,这是对现成数据的常识评估显而易见的结论,心理学家声称需要困难的分析方法来正确理解智商测试数据似乎是经过精心设计的向公众隐瞒IQists没有衣服的其他显而易见的事实。

    • 回复: @utu
  381. utu 说:
    @CanSpeccy

    智商分数不稳定:

    在四个重测间隔后,WAIS 分数的重测增益。
    https://www.ncbi.nlm.nih.gov/pubmed/457898
    两次对 76 名男大学生进行 WAIS,复试间隔为 1 周、1 个月、2 个月或 4 个月。 基本上所有的语言、表现和全面智商在重新测试时都有显着提高。 四个时间间隔的语言智商分别提高了 4.7、1.8、2.3 和 8 点; 后者是发现的唯一不显着的收益。 性能智商在随后的每个时间段内分别增加了 11.4、9.8、8.7 和 8.0 分,而全量表智商在每个相应的时间段内分别增加了 8.0、5.7、5.4 和 4.2 点。 四个区间的​​重测相关性范围为:言语智商为 91 至 72,性能智商为 87 至 79,全面智商为 94 至 74。 将结果与类似的重测研究进行对比,并讨论了临床和研究意义。

    韦氏儿童智力量表的长期稳定性-第三版
    http://edpsychassociates.com/Papers/WISC3LongStability%281998%29.pdf
    平均重测间隔为 2.87 年,重测信度系数为 Verbal IQ。 性能智商。 和全面智商分别为 87、87。 和 .91。 分别。

    • 回复: @James Thompson
  382. @utu

    韦氏手册过去建议两次测试之间至少间隔 6 个月以避免练习效果,现在建议一年。 这是因为记忆的稳定性会随着材料被记住而导致更高的分数。 如果措施真的不稳定,它们就不会全部上升。
    这都是众所周知的。 论文的使用期限应超过 6 个月。

    • 回复: @CanSpeccy
    , @AaronB
  383. @James Thompson

    詹姆斯,

    当您说连续测试对智商的积极影响是因为“材料被记住”(即从一个测试到另一个测试)时,这是否意味着第二个测试使用与第一个相同的问题,或者仅仅是相同的问题类型? 如果是后者,正如我假设的那样,考试准备的这种效果与一般教育的效果有何不同, 持续时间与智商呈正相关,以及在或多或少的学术闲散期间会消失,如暑假儿童能力倾向测试成绩的下降,社会阶层,阅读习惯等改变的跌倒?

    证据似乎表明,智商测试衡量的不是先天能力,而是受文化和生活方式改变的先天能力,这是西方传统生活方式的教育组成部分,强调算术和识字,具有很强的积极性对考试成绩的影响。

    既然如此,似乎可以肯定的是,随着西方文明沿着目前的衰落轨迹前进,儿童的学业成绩和智商将继续下降,后者已经说过 以每代七分的速度下降. 按照这个速度,我们可以预计到 2050 年左右,西部和撒哈拉以南非洲的智商会趋于一致,因为非洲人接受了更多的教育,而我们的孩子则通过将更多的生命投入到电子游戏中来丰富硅谷的科技亿万富翁、色情和社交媒体。

    • 回复: @James Thompson
    , @AaronB
  384. @CanSpeccy

    使用相同的测试。 没想到这些耗时的面对面韦氏测试会经常重复,但考虑到有些会重复,建议间隔六个月。 正如我所描述的,他们现在建议一年更好,并警告说仍然会有学习效果。
    另一方面,小组测试通常有 A 和 B 版本,因为假设学生可能会重新参加测试。
    因此,对 Wechsler 的重测效应本身并不是教育效应。 这是由于重复使用相同的材​​料而带来的限制。 智力测试被设计为“学校远”。 也就是说,不要非常依赖一个学生可能在一所特别好的学校学习的细节,而是使用所有学童都应该知道的材料。 PISA 和 NAEP 等“就近学校”考试以及其他学校考试受课程和学校变量的影响要大得多。
    从广义上讲,所有人类创造都是文化的。 少数没有教育系统的国家在概念性任务上会遇到更多困难。 然而,智力测试试图尽量减少这种影响。 因此,有趣的是看到 PISA 结果与国家 IQ 结果有何不同。

  385. AaronB 说:
    @James Thompson

    但是,如果你有一个小组一直在练习“类似智商”的任务,也许是作为一种生活方式或文化选择,他们的测试分数会更高,而不会反映群体之间先天能力的差异。

    重测效应清楚地表明,生活方式和文化对智商得分有很大影响,这已经是一段时间以来的常识性结论,并且比遗传力概念更符合观察到的群体和国家之间的某些模式。

    由于文化是可遗传的,遗传力测量可能掩盖了重新测试的影响,文化变化缓慢,我们只有最近的智商数据

    • 回复: @AaronB
    , @James Thompson
  386. AaronB 说:
    @CanSpeccy

    证据似乎表明,智商测试衡量的不是先天能力,而是受文化和生活方式改变的先天能力,这是西方传统生活方式的教育组成部分,强调算术和识字,具有很强的积极性对考试成绩的影响。

    这似乎与越来越多的证据最为一致。 问题是他们不能接受这一点。 正如 utu 所说,他们想要“具体化”智商——仅此而已。 他们希望它是一个稳定的“事物”——而不是某种不断变化的衡量标准,取决于文化和政治环境。

    一段时间以来,我一直在努力让他们解决动机(也许是生活方式或文化因素)如何影响智商的问题——常识表明,不能将先天能力与动机分开来衡量。

    回应是一种众所周知的修辞手段,用于处理一个无法通过刻意避免看到它来回答的逻辑反对意见。

    人们只是简单地“假设”动机在时间和空间上是不变的。 这是一个主要假设——因为它必须作为他们摇摇欲坠的大厦的主要支柱。

    特别具有讽刺意味的是,HBD 的一个关键原则是群体在所有特征上都不同。 但HBD运动当然是寻求将种族和民族具体化,不能接受种族的属性也在不断变化。

    HBD 运动想要一个“事物”的世界——消除赫拉克利特的通量。 这表明西方创新的长期疲惫和对稳定的渴望。 HBD 和 IQism 是对健康愚蠢的本能,对现实的自我保护盲目,对在稳定事物的世界中寻求庇护的本能。 这是“关闭西方思想”的一部分。

    虽然这可能是对长期创新的回应,但它可能是一种需要的睡眠。 HBD和IQ具体化运动可能是对稳定和睡眠的本能的负面表现,但也许满足它的方式不是反驳它——这是微不足道和无效的——而是为同样的欲望提供另一个出口,现在承认西方需要一个良好的恢复性午睡。

    • 回复: @utu
  387. AaronB 说:
    @AaronB

    即使使用相同的测试,也没有区别。

    假设有两个人第一次参加考试——一个人可能一辈子都在练习这类问题,另一个人没有。 即使是现代社会的两个孩子。

    实际上,没有两个人第一次参加智商测试时的准备程度相同。

    父母强迫他整天学习的好学的孩子将一生都在练习智商风格的解决问题的技巧。 运动型的孩子可能很少花时间练习。

    因此,对 Wechsler 的重测效应本身并不是教育效应。 这是由于重复使用相同的材​​料而带来的限制。

    显然,智商测试旨在反映人们在现代西方教育和社会中遇到的各种问题解决任务。

    显然,一个更努力学习、更认真对待学校的人将练习智商旨在测试的解决问题的技能。

    显然,重新测试的效果意味着 IQ 风格的问题解决能力反映了实践——因此也反映了动机——与先天能力一样多。

  388. @AaronB

    不要认为练习智力测试会提高智力。 反对它的证据,除了再次使用相同的测试,但很少推广到其他测试。
    教育,如果在晚年延长,似乎增加了 2 分。
    对这方面的任何证据持开放态度,但迄今为止最好的结果是 Ritchie 和 Tucker Drob,他们说 2-5 分,但认为 2 分优势最有可能被复制。
    https://www.unz.com/jthompson/boost-your-iq

  389. Art 说:

    对不起,但这不是让事情变得复杂。 北进的答案不就是达尔文的进化吗? 适者生存?

    在寒冷的天气中生存需要智慧——而不是强壮。 为了在寒冷的天气中生存,需要个人之间的计划、责任和合作。 在寒冷的天气条件下,弱者被淘汰。 这产生了更快的进化。

    具有智力基因和合作基因的人能在寒冷的天气中生存。

  390. utu 说:
    @AaronB

    从我所读到的关于阿尔弗雷德·比奈(Alfred Binet)的文章中,他拥有伟大的法国科学哲学传统,我怀疑他会迷恋斯皮尔曼的粗俗还原论,从而导致物化的诡计。 不幸的是,他并没有活多久。 与亨利庞加莱没有什么不同,如果他活得更久,可能会对现代物理学,特别是相对论产生有益的影响。 庞加莱对科学哲学的研究非常深入(另见皮埃尔·杜姆),他的观点比大多数人,尤其是爱因斯坦更深刻、更细致。

    在这里,您可能会对 Alfred Binet 有所了解:

    “比奈会争辩说,‘智力’无论它是什么,都永远不能与所讨论的个人的实际经历、环境和个人交往隔离开来”

    大约在他结束在 Salpetriere 的工作时,他开始对他的小女儿 Madeline 和 Alice 进行测试和拼图实验,并于 1890 年发表了三篇文章。他指出,儿童与成人的反应时间差异取决于是否孩子很注意。 儿童心理状态的重要性和环境对测试的影响对比内特来说仍然很重要(Fancher,1985)。

    他报告了女儿们成长过程中的经验数据,并在 1902 年题为 L'Etude Experimentale de l'Intelligence(智力实验研究)(Zusne,1975 年)的出版物中讨论了她们之间不同类型的智力。 他注意到他的女儿在学习走路时的不同风格,并将差异等同于两个同样聪明的成年人使用不同的方法解决问题:

    单纯的数字无法显示出来。 . . 实验的亲密本质。 当一个人在工作中观察一个主题时,这种信念就会自然而然地产生。 . . 实验者判断[受试者]头脑中可能发生的事情,并且肯定很难用一个简单、粗暴的数字来表达一个思想的所有波动,这只能具有欺骗性的精确度。 . . 我们认为有必要坚持认为,一个人的暗示性不能完全用一个数字来表达。 (范彻,1985 年,第 62-63 页)

    如果 19/20 世纪的法国像 20 世纪的以色列一样有一支突击队杀死敌方科学家,也许我们现在的状态会更好。

    • 回复: @AaronB
  391. AaronB 说:
    @utu

    有趣的。 知识运动被具有不同议程和能力较弱的人接管并不罕见。 我想知道从现在起 50 年后,智商是否会被用于我们完全无法预料的用途。

    你的引述显示了一个敏感的、微妙的立场,这与今天关于智商的谈话完全不同——比内特的目标似乎与今天的智商研究人员不同; 真正理解一个复杂的主题。 在他那个时代,西方思想的封闭并没有那么先进,他仍然在欧洲探索和好奇的伟大传统中运作。

    今天的研究人员并不那么热衷于探索一个引人入胜的话题并接受它出现的有趣方面,而是希望在一个狭窄的概念框架(很少有概念可以接受)内“建立确凿的事实”(大多数是预先设想的)——这是一个非常不同的议程,并且缺乏真正的好奇心。

    顺便说一句,塔勒布是庞加莱的忠实粉丝,我一直想了解他。

    • 回复: @utu
  392. utu 说:
    @AaronB

    更多关于比内特。

    比奈认为,智力是一种多方面的心理能力,它们在现实世界中联系在一起,并受实际判断的控制。

    在我们看来,智力中存在一种基本的能动性,缺乏或改变这种能动性对实际生活具有最重要的意义。 那就是判断力,也称为良好的感觉、实际的感觉、主动性或适应自己的能力……与判断力相比,智力的其余部分似乎并不重要。 (范彻,1985 年,第 74 页)

    他 1909 年的出版物 Idees Modernes sur les Enfants(关于儿童的现代观念)提出了提高儿童智力的想法,即使是在他认为可以扩大注意力的最低水平。 他相信个人确实有一个心理上限,但很少有人接近这个极限。 克拉克(1979)指出,比内认为智力是可教育的这一信念直到 1960 年代才再次得到证实,即使在今天,他的许多文章和演讲也可以被认为是激进的。

    1909 年,Binet 和 Simon 列出了智能思维的以下三个标准:方向,获取和维持给定的心理集合; l'adaptation,为达到给定目的而对思想的适应; 和 la critique,对一个人的思想采取批判的态度,并在必要时纠正它(Carroll,1982)。

    高尔顿和比内都在同一年去世。 高尔顿是个老人,早就停止工作了。 比奈 54 岁,正处于职业生涯的巅峰。 尽管有 277 篇出版物归功于他(Dennis,1954 年被 Albert 引用,1983 年),但比奈并没有留下许多追随者和 他的技术很快以“高尔顿”的方式应用 (范彻,1985 年)。

    你让我对塔勒布很好奇。 他在庞加莱发现了什么? 如果您检查 Poincare,另请参阅 Pierre Duhem。 他对英国科学和德国科学有有趣的观察。

    • 回复: @AaronB
  393. 塔勒布似乎是个混蛋,但他的第三点:

    如果您想检测某人在某项任务中的表现,……让他/她完成该任务

    听起来不错。

    詹姆斯认为,这一点在理论上是好的,但应用起来很昂贵。 然而,大多数雇主都遵循塔勒布的方法,这并不一定那么困难。 雇主寻找资格,无论是学校成绩还是过去的就业责任和成就,或者他们设计自己的特定工作能力倾向测试。 这种方法似乎普遍有效。 当我招聘 STEM 毕业生时,我从他们毕业班中排名前 XNUMX% 的人中挑选出来。 他们总是很好。 他们知道自己的主题,他们很认真,他们能够处理各种各样的任务,就像那些在几乎所有方面都得 A 的人所期望的那样,他们很快就掌握了新技能。

    不是每个人都可以根据学业成绩被聘用,但设计与工作相关的能力倾向测试通常并不难,无论是针对公共汽车司机、文员还是其他任何人,这比找出候选人解决​​问题的容易程度更有用字谜、匹配几何图案或推断数字序列中的下一个项目。

    如果 IQ 分数在选择优秀员工方面比业绩记录更有用,那么大多数公司都会使用它们,但很少有人这样做。 谷歌可能是个例外,但看看结果:詹姆斯·达莫尔(James Damore)的防御,辐射造成的损失仍有待充分计算; 为了 SJW 议程的利益而摆弄搜索结果,从而损害了对 Google 搜索结果的信任; 他们遵守中国政府的审查要求,因此表明他们愿意与暴君合作; 一直以来,谷歌在互联网搜索中的份额都在下降,这反映在用户的信心上。 几乎没有一家聪明公司的形象。 而现在他们说,他们永远无法在困难的驾驶条件下让自动驾驶工作。 哦,太好了:当他们的黑冰或黑雾在高速公路上斑块时,请注意那些 Google Waymo 车辆。

    • 回复: @James Thompson
  394. AaronB 说:
    @utu

    很多年前我读过塔勒布,记得庞加莱是他的英雄之一。 我在网上找到了 Taleb 的一些相关引述——

    亨利·庞加莱(Henri Poincaré)尽管声名远播,但通常被认为是一位被低估的科学思想家,因为他的一些想法花了近一个世纪的时间才得到认可。 他可能是最后一位伟大的思考数学家(或者可能反过来,一位数学思想家)。 每次我看到一件印有现代偶像阿尔伯特爱因斯坦照片的T恤,

    我不禁想起庞加莱——爱因斯坦值得我们尊敬,但他取代了许多其他人。 我们的意识空间太小了; 那里是赢家通吃的。

    再一次,庞加莱独自上课。 我记得我父亲推荐庞加莱的散文,不仅仅是因为它们的科学内容,还因为他的法语散文的质量。 大师把这些奇事写成连载的文章,像即兴演讲一样编成。 就像在每一部杰作中一样,你会看到重复、题外话的混合,一切都是一个“我也是”编辑者会谴责的东西——但由于思想的铁律一致,这些使他的文本更具可读性。

    庞加莱在 XNUMX 多岁时成为了一位多产的散文家。

    我要重申,庞加莱是真正的科学哲学家:他的哲学思想来自于他见证了学科本身的局限性,这就是真正的哲学。 我喜欢通过将庞加莱命名为我最喜欢的法国哲学家来勾引法国文学知识分子。

    « 他是哲学家? 什么意思,先生? » 总是令人沮丧地向人们解释,他们推崇的思想家,如亨利·柏格森或让-保罗·萨特,很大程度上是时尚生产的成果,在影响力方面无法与庞加莱相提并论。持续几个世纪。

    许多人声称庞加莱在爱因斯坦之前发现了相对论——爱因斯坦从他那里得到了这个想法——但他并没有把它当成什么大事。 这些说法自然是法国人提出的,但爱因斯坦的朋友兼传记作家亚伯拉罕·派斯似乎对此有所证实。 庞加莱在背景和举止上都太贵族了,无法抱怨结果的所有权。

    Poincaré 是第一个了解并解释我们的方程存在基本限制的著名数学家。 他引入了非线性,即可能导致严重后果的小效应,这个想法后来变得流行,也许有点太流行了,就像混沌理论一样。

    这种受欢迎程度有什么毒? 因为庞加莱的全部观点是关于非线性对预测的限制; 它们不是使用数学技术进行扩展预测的邀请。 数学可以相当清楚地向我们展示其自身的局限性。

    我越读越喜欢比内特。

  395. Sparkon 说:

    比内特:“……主动……”

    A不持久! 只有 411 条评论才能到达那里。

    但是,我们 好奇心决心?

    这些是有价值的个人特征或属性,但很难测试,不是吗?

    这是另一个: 独创性.

    专业提示/提示:大多数人都是模仿者。

    我称之为: 有样学样.

    • 回复: @CanSpeccy
  396. @CanSpeccy

    如果从优质大学中选出前10%的STEM毕业生,IQ测试就已经给出了。 如果可以,请始终使用捷径。

    • 回复: @CanSpeccy
  397. @James Thompson

    如果从优质大学中选出前10%的STEM毕业生,IQ测试就已经给出了。 如果可以,请始终使用捷径。

    不确定。 当我还是本科生时,我们大学辩论社的明星,一个红发的Eng。 点亮。 捷克血统的少校,在食堂排队时被无意中听到:“我刚参加了一次智商测试,得分95。”

    他是否以全班前百分之十的成绩毕业,我不知道,但他可能有。 正如 Alfred Binet 关于他的智力量表所评论的那样:

    这个尺度正确地说不允许测量智力,因为智力素质是不可叠加的,因此不能像测量线性表面一样测量,相反,它是一种分类,是不同智力之间的等级; 对于实践的需要,这种分类相当于一种措施。 (比内和西蒙,1980,第 40-41 页)

    因此,似乎没有理由假设 95 的智商与在特定专业领域排名前 10% 的优秀大学毕业的能力不一致。

    此外,获得好成绩的方法有很多种,但它们的含义并不相同。 我有一个姐姐,她获得了一大笔大学奖学金,其中包括物理 A 级,她对这门学科兴趣不大,似乎也不太了解。 然而,这并没有阻碍考试的成功,因为正如她在考试后所说的那样,“我可以在我眼前看到教科书的页面”。 她当然是一个聪明的学生,但与一个好的物理学生不同。

    乔尔·米切尔的这篇文章,悉尼大学心理学学院的,对智商是智力的衡量标准的观点提出了博学的批评。

    在迄今为止未被提及的评论中,以构建第一个智力量表而闻名的阿尔弗雷德·比内(Alfred Binet)声称他的量表不能衡量智力,而只能根据智力品质的等级进行分类。 试图理解这一评论背后的推理导致了一次历史之旅,从古代数学家欧几里得开始,到现代法国哲学家亨利·柏格森结束。 正如欧几里德所解释的(Heath,1908),构成给定数量属性的量级都是同类(即同质的),但他的标准仅涵盖广泛的量级。 Duns Scotus (Cross, 1998) 通过考虑差异包括密集的量级,这提高了有序属性的可能性(后来被 Sutherland, 2004 考虑)在程度之间具有异质差异(“异质顺序”)。 当然,这些属性是不可测量的。 随后,这成为对心理测量的“数量反对”的基础,首先由 Tannery (1875a,b) 和 Bergson (1889) 提出。 由此可见,对于科学中研究的属性,存在三种结构可能性:(1)分类属性(类别之间存在异质性差异); (2)异质阶(具有异质度的差异); (3) 定量属性(具有完全同质的量级差异)。 只有对类型 (3) 的属性进行测量才有可能,而且据我们所知,心理属性仅属于类型 (1) 或 (2)。 然而,与已知事实相反,心理测量学家出于他们自己的特殊原因坚持认为考试成绩提供了测量值

    任何知道什么是科学测量的人也都知道,心理测试与物理科学中用于描述对距离、质量或温度等定量属性的评估具有相同意义的测量。 虽然一些心理测量学家意识到了这一点,但大多数人并没有,他们通常将测试视为科学测量的工具。

    米切尔对心理测量学的批判是值得的 整体考虑.

    • 回复: @res
  398. @Sparkon

    这是另一个:独创性。

    智商既不是主动性也不是独创性的衡量标准,这也许是美军用它来评估新兵的原因。 军队肯定不希望没有聪明的驴子想出如何避免被杀。

  399. res 说:
    @CanSpeccy

    因此,似乎没有理由假设 95 的智商与在特定专业领域排名前 10% 的优秀大学毕业的能力不一致。

    如果你真的相信这一点,它就解释了很多。 谢谢。

    • 回复: @James Thompson
    , @CanSpeccy
  400. @res

    同意。 “在自助餐厅偷听到”几乎没有证据。

    • 回复: @CanSpeccy
  401. @James Thompson

    同意。 “在自助餐厅偷听到”几乎没有证据。

    大声笑。

    但米切尔(和比奈)对可以以线性尺度衡量智力的观点的批评对我来说似乎很有吸引力,并支持这样的观点,即 95 的智商可能不是一个标准——195 的智商也不能保证能力— 毕业前 10% 的工程师。 点亮。 班级。

    当然取决于班级的智商范围,并且超过一半的人口接受“高等教育”教育,许多班级的平均智商不可能远高于人口平均水平。 事实上,有些肯定低于人口平均数。

    • 回复: @James Thompson
  402. @res

    看,如果你有一个基于不可公度变量的量表——这正是比内特描述他发明的测试的方式——量表上的高分与一个或少数不可公度特征的低分并不矛盾整体测试结果或智商是基于它的。 相反,量表上的低分与一个或少数特征的高分并不矛盾。 每个人都知道有数学专家,例如白痴,他们可以在脑海中分解大量数字,或者将 pi 背诵到十亿位数。 那么,对于一个文学或语言专家,或者在一个可能被称为近乎专家的情况下,一个智商为 95 的人,但说话和写作都比他 90% 的英语好,这有什么不可思议的呢? 点亮。 同学?

    • 回复: @res
  403. @CanSpeccy

    至于比内特,我喜欢他的评论。 考试成绩不是对解决问题所采取的步骤的总结,只是对结果的描述。 Buz Hunt 在此基础上对 IQ 分数持批评态度,但我认为这是期望过高,而不是人们对分数的真正期望。 所有考试都将不符合该标准。
    我不希望过度推销智商,但我在 XNUMX 年前寄予希望的人格测量并没有像我预期的那样做出贡献。 目前,尽管存在测量方面的问题,IQ 仍处于领先地位。 我更喜欢 SS Stevens 的比例尺,但可以通过数字跨度和数字符号来获得。 我真的不认为这会破坏交易。 奇怪,我知道,一个人可以在两分钟内获得如此强大的预测器。

    • 回复: @CanSpeccy
  404. res 说:

    我更喜欢 SS Stevens 的比例尺,但可以通过数字跨度和数字符号来获得。

    你能详细说明一下吗? 你在之前的文章中介绍过吗?

    我特别感兴趣的是它如何转换为比率比例,以及你的两分钟预测器如何与总体 Wordsum 进行比较。

    • 回复: @James Thompson
  405. res 说:
    @CanSpeccy

    看,如果你认为找到百万分之一的案例说明了绝大多数人的情况,那么真的没有什么可说的了。

    并尝试找到一个真实的记录案例,而不是一堆传闻和猜测。

    • 回复: @CanSpeccy
  406. JLK 说:

    很有可能,大多数政策专家当时都尽了最大的努力。 我仍在使用我的单数 g 批评,但正如我之前指出的, g 布尔什维主义并不像某些人想象的那样分离。

    不要忽视充满烟雾的房间的危险。

    • 回复: @utu
  407. @res

    并尝试找到一个真实的记录案例,而不是一堆传闻和猜测。

    这里有一个困难,因为必须依赖智商专家的工作,从这里的评论中可以看出,智商专家始终如一地决心证明不存在这样一个真实的记录案例。

    然而,鉴于个人认知能力得分之间观察到的低相关性,我所描述的那种个体的出现是不可避免的。 毫无疑问,这里的聪明人之一,例如 Utu,可以根据认知能力的 Pearsonian 相互关系矩阵,告诉你一个人可能期望找到具有 90% 语言能力的人的频率,而不是所有人的平均值认知能力在第 40 个百分位。 正如你所说,我怀疑这种可能性低至百万分之一。 但即使他们是,我的情况也得到了证明。

    • 回复: @res
    , @JLK
  408. @James Thompson

    我不希望过度推销智商,但我在 XNUMX 年前寄予希望的人格测量并没有像我预期的那样做出贡献。 目前,尽管存在测量方面的问题,IQ 仍处于领先地位。

    用一个数字定义人类智力的领跑者!

    • 回复: @JLK
  409. utu 说:
    @JLK

    不要让我们悬念太久。

    • 回复: @JLK
  410. JLK 说:
    @utu

    我实际上是在第二个线程中跟进我的最后一个过去。 我很惊讶你没有跟进我链接的 Rimfeld 研究,因为它很好地满足了你的论点。 你在这方面领先。

    • 回复: @utu
  411. res 说:
    @CanSpeccy

    正如你所说,我怀疑这种可能性低至百万分之一。

    我想知道你是否会因为那一点夸张而叫我出来; ) 说真的,我猜它更像是千分之一到十万分之一。 很大程度上取决于你如何定义“好大学”。

    找一个人的语言能力在第 90 个百分位,而所有认知能力的平均值在第 40 个百分位。

    请记住,您曾断言“有能力在特定专业领域排名前 10% 的优秀大学毕业”。 这比“第 90 个百分位的语言能力”要高得多。 你的大多数“好大学”都将达到 90% 的口头分数,然后你将获得前 10% 的分数。

    就“证明你的情况”而言,请参阅你回应的我的评论的第一段(你没有引用)。

  412. utu 说:
    @JLK

    对不起,我没有承认它,但我在你链接后查找了它。 我以前不知道 Rimfeld。 如果我有机会第二次阅读它,我可能会在稍后发表评论。

  413. JLK 说:
    @CanSpeccy

    告诉您在 90% 时,人们可能期望找到具有语言能力的人的频率,以及在 40% 时所有认知能力的平均值。

    我不确定幅度,但这种偏差无疑是存在的。 高水平的语言通常比高空间和高数学领先,因为他们的技能对他们周围的人来说更明显。 油嘴滑舌的销售人员通常比设计产品的冷漠工程师给人留下更好的第一印象,而且通常会赚更多的钱,这让后者很懊恼。

    哪些技能具有最大的社会和经济效用? 对于一篇文章来说,这将是一个很好的话题,而且随着互联网改变了做事的方式,这些年来争论的平衡可能已经发生了变化。

    • 回复: @res
  414. @res

    搜索栏目前无序。
    没有 Wordsum / 2 min 预测变量相关性,但应该有一个!

  415. res 说:
    @JLK

    Steve Hsu 的这篇博文是对这个话题的一个有趣的看法: http://infoproc.blogspot.com/2011/06/high-v-low-m.html

    这是来自 SMPY(队列 2,1 级中的 200 个)的图表,显示了不同专业和职业的典型 MVS(S 符号和幅度由箭头表示)概况。

    这是一群没有代表性的人。 来自史蒂夫最后的评论 http://infoproc.blogspot.com/2011/06/human-capital-mongering-m-v-s-profiles.html

    由于选择标准,如果这个群体中的 SD 很大,我不会感到惊讶。 许多 SMPY 预选赛的 V 分数可能相对平均,SVPY 反之亦然。 因此,每种能力的最高分和最低分之间的差异可能大于一般人群。

    • 同意: JLK
    • 回复: @JLK
  416. JLK 说:
    @CanSpeccy

    用一个数字定义人类智力的领跑者!

    我很清楚,至少还有一些额外的心智能力没有被传统的能力任务探测到。 智商测试中的大多数推理问题都很简短,离散且可以通过线性思维解决。 它们与例如情报分析员的工作完全不同,后者必须阅读数百份文件、吸收信息并综合分析。

    在这样的分析领域,我注意到有些人比其他人更准确、更高效几个数量级。 他们的大脑似乎毫不费力地将信息无意识地组织成可询问的星座,识别模式和关联,并吐出其他人需要更多时间才能使用一系列线性比较得出的理论。 如果他们能做到的话。

    我注意到的另一件事是有些人有阻止阅读的能力。 他们可以看一个段落或页面一秒钟,基本上知道它在说什么,而不用逐行移动他们的眼睛。

    我读过一些天才有格式塔直觉。 我不是大脑绘图方面的专家,但前额叶皮层后面的区域可能正在进行一些无意识的处理,加上这两个部分之间的连接比通常更有效。 有点像分布式计算,两台计算机独立工作,直到有信息转储。 我觉得这个主题很吸引人。

    • 回复: @CanSpeccy
  417. JLK 说:
    @res

    这实际上是 Hsu 表示它的一种非常政治化的方式,因为社会科学/人文/法律人士可能很难弄清楚如何阅读 Z 轴。 😉

    这很适合他的情商。

    Hsu 的 Z 轴本质上是 PSAT/SAT 格式中缺少的,以及 X 轴的压缩。

  418. @JLK

    我很清楚,至少还有一些额外的心智能力没有被传统的能力任务探测到。

    这就是智力的字典定义告诉我们的:智力有很多方面,或者同义词,聪明。 这些范围从像牛津大学的任何毕业生一样说话蓝色条纹,到能够让女孩微笑,或者在尼亚加拉大瀑布上走钢丝。 如此多样化的功能必须参与不同的心理过程,实际上,大脑似乎被划分为多个模块,每个模块都通过无数代的选择而进化,以相对准确和速度执行特定任务。

    现在据说类似的东西是 在新兴的人工智能系统中很明显. 人工智能的进步并不依赖于更快地应用一个过程来解决每个问题。 相反,人工智能系统通过创新不断进步,这些创新越来越多地由系统本身创建、评估和进一步发展。 既然如此,试图在线性尺度上评估人工智能系统的能力将毫无意义。 系统将出现以解决特定问题或一组问题,并且仅根据它们在这些任务中的速度和准确性来评估它们是有意义的。

    人类需要解决许多不同的问题,因此在一个单一的尺度上评估他们的整体智力能力可能是有意义的,该尺度结合了不可通约变量的分数。 但是人们不应该忽视独立认知能力的独立可变性,在极端情况下,这种可变性可能是巨大的,就像在学者主义的情况下一样。 许多天才与白痴学者有着微弱的相似之处,尽管偶尔会出现惊人的多面手。

    • 回复: @CanSpeccy
    , @JLK
  419. CanSpeccy 说:
    @CanSpeccy

    人工智能系统通过创新不断发展,系统本身越来越多地创建、评估和进一步发展。

    这意味着人工智能的发展已经成为一个 在硅片 在一个拥有无限资源的“大脑”中以接近光速的速度进行进化——或者至少在谷歌、亚马逊、DARPA和中国政府的资源下。

    结果将是多种智能,每种智能都适用于特定目标,其中一些目标由人工智能本身设想为实现人类选择的目标所必需的。 在某些情况下,人工智能可能会制定我们无法想象也不会同意的目标。 也许这就是军备竞赛的结束方式:人工智能指挥同时对华盛顿、莫斯科和北京进行核打击,并严厉警告所有其他国家的首都放弃通过军事手段获得优势的希望。

  420. JLK 说:
    @CanSpeccy

    许多天才与白痴学者有着微弱的相似之处,尽管偶尔会出现惊人的多面手。

    其中一些我可以没有的技能。 至少雨人不是骗子。

  421. @J

    是的,但无论如何那将是各种各样的交配。

  422. obwandiyag 说:

    “当然,还有一个问题,即能力只能根据动机和目标来衡量,除非所有个人和群体在这些方面都可以说是相同的(这会奇怪地与 HBD 的假设相矛盾,即群体在所有特征上都有差异) ,测量能力是有问题的......房间里的大象没有人愿意解决......

    无论如何,这只是冰山一角……

    向抽象思维和广泛概括的转变在历史上在一段时间内是有用的,但今天,我们所处的位置是,构建一种更智能的现实方法意味着从抽象和一般转向具体,以及精确度的损失将通过细节的丰富性和对现实中实际模式的更好一致性得到补偿,而无法建立永恒的规律和发现柏拉图式的思想将被对现实做出现实的灵活性所抵消。”
    亚伦

    我将为这个奇妙的智能观察添加一些东西。

    我遇到的大多数智商高的人都有点愚蠢。

  423. obwandiyag 说:

    例如,小说家并不以高智商而闻名。

    现在,我知道你们这些二年级的傻瓜都是阿斯伯格的“STEM”崇拜者(即使使用首字母缩略词“STEM”——除了嘲笑它——是白痴的表现),你们认为小说和小说家只是愚蠢、懒惰、偷懒的人,谁胆大妄为地拒绝参与全人类的总的圣战,通过(“STEM”)技术摧毁全人类及其所生活的世界。

    所以你试着写一本任何人都想读的小说。

    然后你就会知道小丑帽子的归属。 在你自己的头上。

    • 回复: @Tom Welsh
  424. obwandiyag 说:

    根据我的经验,大多数“科学家”对“科学”的理解都相当简单。 他们实际上认为它存在。 我的意思是,就像梨或笨蛋一样存在。

  425. Tom Welsh 说:

    Edward Dutton 对此表示赞同:例如“为什么拥有很高的智商意味着你可能会过着糟糕的生活”。

    Sonja Falck 的书中还包含许多有趣的事实和想法。 (“极端智能”和“与极端智能一起生活”)。

    据我所知,主要有两个方面。

    1. 非常高的一般智力可以而且显然经常会导致其拥有者在错误的时间过度依赖它。 他们往往忽视本能和情感的支配,低估习俗。 对此类问题的有意识的心理决定有一种令人讨厌的方式,结果证明是肤浅的和错误的。

    2. 很简单,非常聪明的人倾向于发现追求“成功”是乏味的。 世界充满了有趣得多的事物,它们可以从早到晚(通常是深夜)填满一个人的一天。 问题可能在于,对金钱和地位形式的成功的追求有一种要么全有要么全无的方式。 离群的高智商人轻蔑地背弃它(绝大多数人也背弃它),常常让他们沮丧的是,当你发现你需要金钱和安全时,要捡起它并不那么容易。

    内向可能会加剧离群的高智商人群远离日常商业和地位的趋势。 塔勒布先生听起来像是一个非常内向的外向者,所以他可能对这些人缺乏同理心。 (我一直觉得很讽刺的是,外向的人经常谈论同理心,但他们似乎对那些不幸没有那么多同理心的人缺乏同理心)。

  426. Tom Welsh 说:
    @obwandiyag

    “然后你就会知道小丑帽属于哪里了。 在你自己的头上”。

    现在你听起来 *确切地* 像塔勒布先生。

当前评论者
说:

发表评论 -


 记得 我的信息为什么?
 电子邮件回复我的评论
$
提交的评论已被许可给 Unz评论 并可以由后者自行决定在其他地方重新发布
在翻译模式下禁用评论
通过RSS订阅此评论主题 通过RSS订阅所有James Thompson的评论